100% found this document useful (3 votes)
6K views

Electrostatic Potential and Capacitance

Sl Arora pdf of chapter 2 electrostatic potential and capacitance usefull for class 12th students in their study of physics

Uploaded by

Anjali Gupta
Copyright
© © All Rights Reserved
Available Formats
Download as PDF, TXT or read online on Scribd
100% found this document useful (3 votes)
6K views

Electrostatic Potential and Capacitance

Sl Arora pdf of chapter 2 electrostatic potential and capacitance usefull for class 12th students in their study of physics

Uploaded by

Anjali Gupta
Copyright
© © All Rights Reserved
Available Formats
Download as PDF, TXT or read online on Scribd
You are on page 1/ 94

C H A PT E R

ELECTROSTATIC POTENTIAL
AND CAPACITANCE

R
SI
IT
2.1 charge + q. We then calculate the potential difference
ELECTROSTATIC POTENTIAL AND
between points A and Bby the equation:
POTENTIAL DIFFERENCE
W
V-v
- B -V A--~ ...(2.1)
Introduction. The electric field around a charge can
qo
be described in two ways :
-t So the potential difference between two points in an
H
(i) by electric field (E), and electric field may be defined as the amount of work done in
(ii) by electrostatic or electric potential (V). moving a unit positive charge from one point to the other
-t
against the electrostatic forces.
The electric field E is a vector quantity, while In the above definition, we have assumed that the
electric potential is a scalar quantity. Both of these test charge is so small that it does not disturb the
O

quantities are the ~haracteristic properties of any point


distribution of the source charge. Secondly, we just
in a field and are inter-related.
apply so much external force on the test charge that it
just balances the repulsive electric force on it and hence
1. Develop the concepts of potential difference and does not produce any acceleration in it.
electric potential. State and define their 51 units.
M

SI unit of potential difference is volt (V). It has been


Potential difference. As shown in Fig. 2.1, consider named after the Italian scientist Alessandro Volta.
a point charge + q located at a point O. Let A and Bbe
1 joule
two points in its electric field. When a test charge qo is 1 vot=--"---
1
moved from A to B, a work WAB has to be done in 1 coulomb
moving against the repulsive force exerted by the or 1V = 1Nm CI = 1 JC
I

Source Test Hence the potential difference between two points


charge charge in an electric field is said to be 1 volt ifl joule of work has
+q +qo to be done in moving a positive charge of 1 coulomb from one
• ••• • point to the other against the electrostatic forces.
o B A
Electric potential. The electric potential at a point
Fig. 2.1 To define potential difference. located far away from a charge is taken to be zero.
(2.1)
2.2 PHYSICS-XII

r r
= 0, so
that
In Fig. 2.1, if the point A lies at infinity, then VA
W = f dW = - f 00
Fdx = - fl. 4rc
co EO
qqo dx
x2
w
V= VB=-
qo = _~
4rc
f
EO co
X-2 dx =_~
4rc EO
[_.!.]
X
r
co
where W is the amount of work done in moving the
test charge qo from infinity to the point B and VBrefers _~[~_~] __ 1_ q%
to the potential at point B. - 4rc EO r r . 00 -4rc EO·
So the electric potential at a point in an electricfield is
Hence the work done in moving a unit test charge
the amount of work done in moving a unit positive charge
from infinity to the point P, or the electric potential at
from infinity to that point against the electrostatic forces.
point Pis

R
. . Work done
Electric potential = ----
V= W or V=_l_.2.
Charge
qo 4rc EO r
SI unit of electric potential is volt (V). The electric
Clearly, V cc 1/ r. Thus the electric potential due to a
potential at a point in an electric field is said to be 1 volt if
point charge is spherically symmetric as it depends only
one joule of work has to be done in moving a positive charge

SI
on the distance of the observation point from the
of 1 coulomb from infinity to that point against the charge and not on the direction of that point with
electrostatic forces. respect to the point charge. Moreover, we note that the
potential at infinity is zero.
2.2 ELECTRIC POTENTIAL DUE TO
Figure 2.3 shows the variation of electrostatic
A POINT CHARGE potential (V cc 1/ r) and the electrostatic field (E o; 1/1)
2. Derive an expression for the electric potential at a with distance r from a charge q.
distance r from a point charge q. What is the nature of 5~-w--r--.---~--r-~--~---r--'---'
this potential ? 4.5
IT
Electric potential due to a point charge. Consider a 4
positive point charge q placed at the origin O. We wish 3.5 V=_l_. !!..
to calculate its electric potential at a point P at distance 3 41[1:0 r
r from it, as shown in Fig. 2.2. By definition, the electric t 2.5
potential at point P will be equal to the amount of work t<l
:::': 2
H
done in bringing a unit positive charge from infinity to 1.5
the point P.
I
q ~ qo F 0.5
(9~--------•-•---~.~--~.~--~.
__----oo
o P B A
00 0.5 L5 2 2.5 3 3.5 4 4.5 5
'1+--- r -----+t
O

r~
14 x -----.t~1
Fig. 2.3 Variation of potential V and field E
Fig. 2.2 Electric potential due to a point charge. with r from a point charge q.
Suppose a test charge qo is placed at point A at
distance x from O. By Coulomb's law, the electrostatic 2.3 ELECTRIC POTENTIAL DUE TO A DIPOLE
M

force acting on charge qo is 3. Derive an expression for the potential at a point


F __ 1_ qqo along the axial line of a short dipole.
- 4rc EO • x2 Electric potential at an axial point of a dipole. As
~ shown in Fig. 2.4, consider an electric dipole consisting
The force F acts away from the charge q. The small of two point charges - q and + q and separated by
work done in moving the test charge qo from A to B distance 2 a. Let P be a point on the axis of the dipole at
~
through small displacement dx against the electro- a distance r from its centre 0.
-q +q
static force is ee======t:1===:::::::eemummu-l
~ ~ A 0 B P
dW = F . dx = Fdx cos 180 = - Fdx0
I+--- a -~~1"4-- a -----+I

The total work done in moving the charge qo from 14 ~I


infinity to the point P will be Fig. 2.4 Potential at an axial point of a dipole.
ELECTROSTATIC POTENTIAL AND CAPACITANCE 2.3

Electric potential at point P due to the dipole is Let AP = r1 and BP = r2. P

1 - q 1 q Net potential at point P due to the dipole is ,":


V=V +V =--.-+--.-
1 2 41t EO AP 41t EO BP " I

V=V1+V2 rl,'" /
1 q 1 q
=---.--+--.--
41t~ r+a 41t~ r-a
=_1_.-q+_1_.!1. "" /r2
41t EO r1 41t EO r2 ,,' /

__ q
41t EO
[_1 __ 1 ]
r- a r+ a
q
= 41t
[1 1] ~.: L
EO ~ - ~
,rv!

",
"v"
e···-. /
:
I

A-q . 0 +q
B
=_q_[(r+a)-(r-a)]=_l_ qx2a - 41tQE \~2r2 I+-- 2a--.t

R
2 2 . 2 2 [ ]
41t EO r - a 41t EO r: - a o
Fig. 2.6

or V- ---'--2
1 P [.: p=qx2a] If the point P lies far away from the dipole, then
41t EO ? -a r1 -'2'" ABcos e =2a cos e and 'lr2 "'?

SI
For a short dipole, a2 < < ?, so V = _1_ E V=-q- 2acose
41t EO . ?. 41t EO . ?
4. Show mathematically that the potential at a point
on the equatorial line of an electric dipole is zero. or V- 1 pcose
- 41tEo .-?-
Electric potential at an equatorial point of a dipole.
--t ~ ~ 1\

As shown in Fig. 2.5, consider an electric dipole V- 1 p.r _ 1 p.r


consisting of charges - q and + q and separated by
distance 2a. Let P be a point on the perpendicular
or - 41tEo --r- 41tEo ~
" ~
IT
bisector of the dipole at distance r from its centre O. Here p = q x 2 a, is the dipole moment and '='/',
~ ~
is a unit vector along the position vector OP =r.
p
Special Cases
(i) When the point P lies on the axial line of the
dipole, e =0° or 180°, and
H
1 p
V=+-- -
- 41t EO . ?
i.e., the potential has greatest positive or the
greatest negative value.
O

Fig. 2.5 Potential at an equatorial point of a dipole. (ii) When the point P lies on the equatorial line of the
dipole, e =90°, and V =0, i.e., the potential at
Electric potential at point P due to the dipole is any point on the equatorial line of the dipole is
zero. However, the electric field at such points
1 -q 1 q
V=V +V =--.-+--.- is non-zero.
1 2 41t EO AP 41t EO BP
M

6. Give the contrasting features of electric potential of


= __ 1_ q +_1_ q =0 a dipole from that due to a single charge.
41t EO . ~? + a2 41t EO . ~? + a2 . Differences between electric potentials of a dipole
and a single charge.
5. Derive an expression for the electric potential at
any general point at distance rfrom the centre of a dipole. 1. The potential due to a dipole depends not only
on distance r but also on the angle between the position
Electric potential at any general point due to a ~
dipole. Consider an electric dipole consisting of two vector, of the observation point and the dipole moment
~
point charges - q and + q and separated by distance 2 a, vector p . The potential due to a single charge depends
as shown in Fig. 2.6. We wish to determine the potential
only on r.
at a point P at a distance r from the centre 0, the direc-
~ 2. The potential due to a dipole is cylindrically symmetric
tion OP making an angle e with dipole moment p. about the dipole axis. If we rotate the observation point
2.4 PHYSICS-XII

P about the dipole axis (keeping rand 8 fixed), the 2.5 ELECTRIC POTENTIAL DUE TO A
potential V does not change. The potential due to a CONTINUOUS CHARGE DISTRIBUTION
single charge is spherically symmetric.
8. Deduce an expression for the potential at a point
3. At large distance, the dipole potential falls off as due to a continuous charge distribution. Hence write the
1/ ? while the potential due to a single charge falls off expression for the electric potential due to a general source.
as 1/ r.
Electric potential due to a continuous charge
distribution. We can imagine that a continuous charge
2.4 ELECTRIC POTENTIAL DUE TO
distribution consists of a number of small charge
A SYSTEM OF CHARGES ~ ~
elements located at positions ~. If r is the position
7. Derive an expression for the electric potential at a
vector of point P, then the electric potential at point P

R
point due to a group of N point charges.
due to the continuous charge distribution can be
Electric potential due to a group of point charges. written as
As shown in Fig. 2.7, suppose N point charges
ql' q2' q3' ..... , qN lie at distances r1, r2, r3, ·····"N from a
V- 1 f dq
41tE ~ ~
point P. o 1 r - '; 1

SI
When the charge is distributed continuously in a
volume V, dq = P dV, where p is volume charge density.
The potential at point P due to the volume charge
distribution will be
V_I
v - 41tE
f ~
p dV
~
p~~----------~--------------~q4 oVlr-~1
When the charge is distributed continuously over
IT
an area 5, dq = o dS where o is surface charge density.
Fig. 2.7 Potential at a point due to a
system of N point charges.
V_I f cr dS
5 - 41tE ~ ~
o Sir - '; 1

Electric potential at point P due to charge ql is


When the charge is distributed uniformly along a
V1 =_1_ !ll line L, dq = 'AdL, where A is line charge density.
H
.
41t EO r1
V_I
L - 41tE
f ~
AdL
~
Similarly, electric potentials at point P due to other . oLlr-~1
charges will be
The net potential at the point P due to the conti-
_ 1 q2 _ 1 q3 _ 1 qN
V2 ---.-, V3---·-, ..., VN ---.- nuous charge distribution will be the algebraic sum of
O

41t EO r2 41tEo r3 41t EO rN


the above potentials.
As electric potential is a scalar quantity, so the total
V::ont = Vv + Vs + VL

~l
potential at point P will be equal to the algebraic sum
of all the individual potentials, i.e.,
V = VI + V2 + V3 + ...+ VN or Vcont =_l_[J
4 ~+J ---+ ---+- ~+J
~ ~ ~-t
11: EO vir - rjl sir -rjl L Ir - rjl
M

=_I_[ql + q2 + q3 + ...+ qN] Electric potential due to a general source. The


41t EO 11 '2 r3 rN potential due to a general source charge distribution,
which consists of continuous as well as discrete point
or V=_I_ ~ qi charges, can be written as
41tEo i=1 ';
V = V::ont + Vdiscrete
~ ~ ~ ~
If r1, r2, r3, ... ,rN are the position vectors of the N
point charges, the electric potential at a point whose
or V = __ 1__ [f p dV + f cr dS

~ 41t EO v 17 - ~ 1 5 17 - ~ 1
position vector is r , would be

V=_I_ ~ qi + f A dL
~~+L..
1 r - '; 1
" ~~qi
1r - ~ 1
1
41t EO i = 1 1 7 _~ 1
L All point
charges
ELECTROSTATIC POTENTIAL AND CAPACITANCE 2.5

2.6 ELECTRIC POTENTIAL DUE TO A


UNIFORMLY CHARGED THIN
SPHERICAL SHELL
9. Write expression for the electric potential due to a
r>
~
Electric ;::: :~r~:~'O:::::hil'pot~ti.~
gradient is a vector quantity.
The electric potential near an isolated positive charge is
uniformly charged spherical shell at a point (i) outside the
shell, (ii) on the shell and (iii) inside the shell. positive because work has to be done by an external
agent to push a positive charge in, from infinity.
Electric potential due to uniformly charged thin
spherical shell. Consider a uniformly charged spherical ~ The electric potential near an isolated negative charge is
negative because the positive test charge is attracted by
shell of radius R and carrying charge q. We wish to
the negative charge.
calculate its potential at point P at distance r from its

R
centre 0, as shown in Fig. 2.8. ~ The electric potential due to a charge q at its own
location is not defined - it is infinite.
p
v V=_l_.~ ~ Because of arbitrary choice of the reference point, the
Shell with ~ __ 4rc.•.•Eo R electric potential at a point is arbitrary to within an
charge Q additive constant: But it is immaterial because it is the

SI
potential difference between two points which is
physically significant.
~ For defining electric potential at any point, generally a
point far away from the source charges is taken as the
reference point. Such a point is assumed to be at infinity.
~ As the electrostatic force is a conservative force, so the
work done in moving a unit positive charge from one
Fig. 2.8 Potential due to Fig. 2.9 Variation of potential point to another or the potential difference between
IT
a spherical shell. due to charged shell with distance two points does not depend on the path along which
T from its centre. the test charge is moved. )

.
(i) When the point P lies outside the shell. We know
Examples based on
that for a uniformly charged spherical shell, the electric
.
field outside the shell is as if the entire charge is
H
concentrated at the centre. Hence electric potential at Formulae Used
an outside point is equal to that of a point charge Work done W
located at the centre, which is given by 1. Potential difference = or V =-
Charge q
V=_I_!i [Forr>R] 2. Electric potential due to a point charge q at
4m;0 r
distance r from it,
O

(ii) When point P lies on the surface of the shell. Here V = _1_.1
r = R. Hence the potential on the surface of the shell is 4rc EO r
V =_I_!L [For r= R] 3. Electric potential at a point due to N point charges,
4m;0 R
V = _1_ ~ !iL
M

(iii) When point P lies inside the shell. The electric field 4rc EO i = 1 1j
at any point inside the shell is zero. Hence electric
4. Electric potential at a point due to a dipole,
potential due to a uniformly charged spherical shell is
constant everywhere inside the shell and its value is V_I pcos e_ 1 P . ~r
equal to that on the surface. Thus, - 4rc EO -r-2- - 4rc EO ~

V =_I_!L [For r « R] Units Used


4m,0 R
Charge q is in coulomb, distance r in metre, work
Figure 2.9 shows the variation of the potential V done W in joule and potential difference V in volt.
due to a uniformly charged spherical shell with
distance r measured from the centre of the shell. Note Example 1. lfl00 J of work has to be done in moving an
that V is constant (= q / 4m;oR) from r =0 to r = R along electric charge of 4C from a place, where potential is -10 V
a horizontal line and thereafter V IX 1 / r for points to another place, where potential is V volt, find the value
outside the shell. ofV.
2.6 PHYSICS-XII

Solution. Here WAB = 100 J, qo = 4 C VA = -10 V, We can consider the circular wire to be made of a
large number of elementary charges dq. Potential due
VB=V,
to one such elementary charge dq at the centre,
As v:B -V A - WAB
- dV=_l_. dq
qo
4n EO r
V -(-10)= 100 =25
4 Total potential at the centre of the circular wire,
1
V= LdV=L- -. dq =_l_Ldq
or V = 25 -10 = 15 V.
4n EO r 4n EO r
Example 2. Determine the electric potential at the surface
of a gold nucleus. The radius is 6.6 x 10-15 m and the atomic 1 q 9 x 109 x 2 x 10-4 6

R
.- = = 18 x 10 V.
number Z =79. Given charge on a proton =1.6 x 1O-19c. 4n EO r 0.10
[Himachal 96)
Example 5. Electric field intensity at point 'B' due to a
Solution. As nucleus is spherical, it behaves like a
point charge 'Q' kept at point 'A' is 24 NC -1 and the
point charge for external points.
electric potential at point 'B' due to same charge is 12 fC-1.

SI
Here q = ne =79 x 1.6 x 10-19 C, Calculate the distance AB and also the magnitude of charge
r = 6.6 x 10-15 m Q. [CBSE OD 03C)

1 q 9 x 109 x 79 x 1.6 x 10-19 Solution. Electric field of a point charge,


.. V=--.-= V
4nEo r 6.6 x 10-15 E=_l_. Q =24 NC-1
4n EO ?-
= 1.7x 107 V.
Electric potential of a point charge,
Example 3. (i) Calculate the potential at a point P due to a
charge of 4 x 10-7 C located 9 em away. (ii) Hence obtain V = _1_. Q ~ 12 JC-1
IT
the work done in bringing a charge of 2 x 10-9 C from 4n EO r
infinity to the point P. Does the answer depend on the path The distance AB is given by
along which the charge is brought ? [NCERT) V 12
r=-=-=O.5m
Solution. (i) Here q = 4 x 10-7 C, r = 9 em = 0.09 m E 24
Electric potential at point P is The magnitude of the charge,
H
7
V = _1_ . !I = 9 x 109 x 4 x 10- = 4 x 104 V. Q = 4n EO Vr = _1-9 x 12 x 0.5 = 0.667 x 10-9 C
9 x 10
4n EO r 0.09
-7 Example 6. To what potential we must charge an insulated
q=4 x 10 C
••------- •• - - - - - - - - - --00
sphere of radius 14 em so that the surface charge density is
o P equal to III Cm' 2 ?
O

'4 9 m ---+I.'
Solution. Here r = 14 cm = 14 x 10-2 m,
Fig. 2.10 c = III Cm -2 = 10-6 Cm-2

(ii) By definition, electric potential at point P is 1 q 1 4n?-cr 1


.. V=--.-=--.--=--.4nrcr
equal to the work done in bringing a unit positive 4nEo r 4nEo r 4nEo
M

charge from infinity to the point P. Hence the


workdone in bringing a charge of 2 x 10- 9 C from =9 x109 x 4x 22 x 14x 10-2 x 10-6 V
7
infinity to the point P is
= 15840 V.
W = qo V =2 x 10-9 x 4 x 104 = 8 x 10-5 J
Example 7. A charge of241lC is given to a hollow metallic
No, the answer does not depend on the path along
sphere of radius 0.2 m Find the potential [CBSE D 95)
which the charge is brought.
(i) at the surface of the sphere, and
Example 4. A metal wire is bent in a circle of radius 10 em
(ii) at a distance of 0.1 em from the centre of the sphere.
It is given a charge of 200 IlC which spreads on it uniformly.
Calculate the electric potential at its centre. Solution. (i) q =241lC =24 x 10-6 C, R =0.2 m
[CBSE OD 9SC) Potential at the surface of the sphere is
Solution. Here q = 200 IlC = 2 x 10-4 C 9
V = _1_ . !L = 9 x 10 x 24 x 10--6 V = 1.08 x 106 V.
r=10 em =0.10 m 4nEo R 0.2
ELECTROSTATIC POTENTIAL AND CAPACITANCE 2.7

(ii) As potential at any point inside the sphere The other possibility is that x may also lie on OA
= Potential on the surface produced, as shown in Fig. 2.12.
:. Potential at a distance of 0.1 em from the centre q}=3x10
-8
C Q2=-2xlO
-8
C
= 1.08 x 10 6
v. o--------------------~o~------------~I
o A P
Example 8. Twenty seven drops of same size are charged at I.:.... 0._15
__- -_-_-_-x-~'Io-,
--x - 0.15 =1
220 Veach. They coalesce to form a bigger drop. Calculate
the potential of the bigger drop. [Punjab 01) Fig. 2.12
Solution. Let radius of each small drop =r
As VI + V2 =0
Radius of large drop =R

R
8 8
Then i1tR3=27xi1tr3 .. _1_ [3X 10- _ 2 x 10- ] =0
3 3 41t EO X x -0.15
or R =3r which gives x = 0.45 m = 45 em
Potential of each small drop, Thus the electric potential is zero at 9 ern and 45 em

SI
V=_l_.!1. away from the positive charge on the side of the
41t EO r negative charge.
:. Total charge on 27 drops, Example 10. Calculate the electric potential at the centre
of a square of side .J2 m, having charges 100 J.lc, - 50 J.lc,
Q = 27 q = 27 X 41t EO rV
20 J.lc, and - 60 J.lC at the four corners of the square.
Potential of large drop,
[CBSE OD 06C]
V' = _1_. Q = _1_ ._27_x_4_1t-,Eo,,-r_V_ Solution. Diagonal of the square
41t EO R 41t EO 3r
IT
= ~(.J2)2 + (.J2)2 =2 m
=9 V =9 x 220 = 1980 V.
Distance of each charge from the centre of the
Example 9. Two charges 3 x 10-8 C and - 2 x 10-8 Care
square is
located 15 em apart. At what point on the line joining the
two charges is the electric potential zero? Take the potential r = Half diagonal = 1m
at infinity to be zero. [ CERT) ., Potential at the centre of the square is
H
Solution. As shown in Fig. 2.11, suppose the two
point charges are placed on X-axis with the positive V=_l_[ql + q2 + q3 + q4]
41t EO r r r r
charge located on the origin 0.
-8 -8 6 6
q}=3 x 10 C q2=-2 x 10 C V = 9 x 109 [100 x 10- 50 x 10-
oo-----------+--------~o
O

1 1
P A
I--- X .,. 0.15 - x ---+l 20 x 10-6 60 x 10-6]
+ ----
Fig. 2.11 Zero of electric potential for two charges. I 1

Let the potential be zero at the point P and OP = x. = 9 x 109 x 10-6 x 10 = 9 x 104 v.
M

For x < 0 (i.e., to the left of 0), the potentials of the two Example 11. Four charges + q, + q, - q and - q are placed
charges cannot add up to zero. Clearly, x must be respectively at the corners A, B, C and D of a square of side
positive. If x lies between 0 and A then 'a' arranged in the given order. Calculate the electric
VI+V2=0 potential at the centre o. If
E and F are the midpoints
1 [ql + q2 ]- 0 of sides BC and CD respec- a/2
41t EO x 0.15-x tively, what will be the
8 8 work done in carrying a
or 9X109[3X10- _2XlO- ]=0 charge 'e' from 0 to E and a/2
x 0.15-x
from 0 to F ?
3 2 Solution. Let OA =
or -----=0
x 0.15-x OB= OC = OD=r.
which gives x = 0.09 m = 9 em Fig. 2.13
2.8 PHYSICS-XII

Then the potential at the centre 0 is dipole in broad-side-on position (ii) electric field and electric
potential at the same point after rotating the dipole through
V = _1_ [!l. + !l. _ !l. _ !l.] =0
90°.
o 4TCEO r r r r
Solution. Dipole moment,
Again, the potential at point E is
p = q x 21 =3 x 10-6 x 2 x 10-3 =6 x 10-9Cm
1 [q
q q q] (i) Electric field in broad-side-on position is
VE = 4TCE AE + BE - CE - DE =0
O 9 9
E= _1_. E = 9x 10 x6x 10- =250 Net
[.: AE = DE, BE = CEl 4TCEO r3 (0.6)3

Now, AF = BF =)a2 +(~r= ~a Electric potential in broad-side-on position, V = O.

R
(ii) When the dipole is rotated through 90°, the
.. The potential at point F is same point is now in end-on-position with respect to
the dipole.
1 [q q q q] E=_l_. 2p =500 Net
VF = 4TCEO AF + BF - CF - DF
4TCEO r3

SI
- .ss. [_1__ .2..] [.: AF = BF, CF = DFl 1 p 9 x 109 x 6 x 10-9
4TCEO AF CF V=--.-= =150V.
4TCEO ? (0.6l
2 [2 2]
q q
= 4TCE .Jsa - -;; = TCEa .Js-1
(1 ) Example 14. Two charges -q and +q are located at points
O O
A(O,O,-a) and B(O,O,+a) respectively. How much work is
Work done in moving the charge' e' from 0 to Eis done in moving a test charge from point P(7,O,0) to
W = e [VE - Vol = e x 0 = 0 Q( -3,0,0) ? [CBSE 0 09)
Solution. Points P and Q are located on the equa-
IT
Work done in moving the charge 'e' from 0 to F is
torial line of the electric dipole and potential of the
W=e[VF -vol=e[-q (~-1)-0] dipole at any equatorial point is zero.
TCEoa ,,5
:. Work done in moving a test charge qo from P to Q,
= TC::a Ck -1). W = qO(VQ- Vp)=qo(O-O)=O.
H
X
Example 12. A short electric dipole has dipole moment of
P(7, 0, 0)
4 x 10-9 Cm Determine the electric potential due to the
dipole at a point distant 0.3 m from the centre of the dipole
situated (a) on the axial line (b) on equatorial line and (c) on
O

a line making an angle of 60° with the dipole axis.


-q +q
Solution. Here p = 4 x 10-9 Cm, r = 0.3 m. ~--~~------~-------'----.Z
B(O, 0, a)
A(O, 0, - a)
(a) Potential at a point on the axial line is
Q(-3, 0, 0)
1 p 9 x 109 x 4 x 10-9
V=--.-= =400 V.
?
M

41tEo (0.3)2 y
(b) Potential at a point on the equatorial line = o. Fig. 2.14
(c) Potential at a point on a line that makes an angle
of 60° with dipole axis is
V __ 1_ pcosS
r-p roblems For Practice
- 4TCEO• ? 1. The work done in moving a charge of 3 C between
9
9 x 10 x 4 x 10-9 cos 60° two points is 6 J. What is the potential difference
2 =200 V. between the two points? (Ans. 2 V)
(0.3)
2. The electric potential at 0.9 m from a point charge is
Example 13. Two point charges of +3~C and-3 ~C are + 50 V. What is the magnitude and sign of the
placed 2 x 10-3 mapari from each other. Calculate (i) electric charge? [CBSE D 95C]
field and electric potential at a distance of 0.6 m from the (Ans. 5 x 10- 9 C, positive)
ELECTROSTATIC POTENTIAL AND CAPACITANCE 2.9

3. The electric field at a point due to a point charge is 11. The sides of rectangle ABCD are 15 cm and 5 ern, as
20 NC-1 and the electric potential at that point is shown in Fig. 2.16. Point charges of - 5 J.lC and
10 JC-1. Calculate the distance of the point from the + 2J.1Care placed at the vertices Band D respec-
charge and the magnitude of the charge. tively. Calculate electric potentials at the vertices A
[CBSED 06] and C. Also calculate the work done in carrying a
(Ans. 0.5 m, 0.55 x 1O-9q charge of 3 J.lCfrom A to C. (Ans. 2.52 J)
4. Two points A and B are located in diametrically A 0
opposite directions of a point charge of + 2 J.lCat
distances 2.0 m and 1.0 m respectively from it.
Determine the potential difference VA - VB'

R
(Ans. - 9 x 103 V)
5. A hollow metal sphere is charged with 0.4 J.lCof Fig. 2.16
charge and has a radius of 0.1 m. Find the potential
12. Charges of 2.0 x 10-6 C and 1.0 x 10-6 C are placed at
(i) at the surface (il) inside the sphere (iil) at a
the comers A and B of a square of side 5.0 cm as
distance of 0.6 m from the centre. The sphere is
shown in Fig. 2.17. How much work will be done in

SI
placed in air. (Ans. 36 kV, 36 kV, 6 kV)
moving a charge of 1.0 x 1O-6C from C to D against
6. Two point charges of + 10 J.lC and + 20 J.lC are the electric field? (Ans. 0.053 J)
placed in free space 2 cm apart. Find the electric
o 5cm C
potential at the middle point of the line joining the

,,0:,
two charges. (Ans. 27 MV)
7. Two point charges q and -2q are kept 'd' distance
apart. Find the location of the point relative to
charge 'q' at which potential due to this system of
IT
charges is zero. [CBSEOD 14C] A B
(Ans. At distance d / 3 from charge q)
Fig. 2.17
8. Two point charges, one of + 100 J.lCand another of
- 400 J.lC,are kept 30 cm apart. Find the points of 13. Calculate the potential at the centre of a square
zero potential on the line joining the two charges ABCD of each side .fi m due to charges 2, - 2, - 3
(assume the potential at infinity to be zero). and 6 J.lCat four comers of it. [Haryana 97]
H
(Ans.6 cm from + 100 J.lCcharge) (Ans. 2.7 x 104 V)
9. A charge q = + 1J.lCis held at 0 between the points 14. Charges of + 1.0 x 10-11C, - 2.0 x 10-11 C,
A and B such that AO = 2 m and 80 = Irn, as shown + 1.0 x 10-11 C are placed respectively at the comers
in Fig. 2.15(a). Calculate the potential difference B, C and D of a rectangle ABCD. Determine the
(VA - VB)' What will be the value of the potential
potential at the comer A. Given AB = 4 em and
O

difference (VA - VB) if position of B is changed as BC= 3cm.


shown in Fig. 2.15(b) ? (Ans. - 4500 V, - 4500 V) (Ans. 1.65 V)
15. ABCD is a square of side 0.2 m. Charges of 2 x 10-9,
q
• • • 4 x 10-9, 8 x 10-9 C are placed at the comers A, B
B 1m 0 2m A
and C respectively. Calculate the work required to
(a)
transfer a charge of 2 x 10-9 C from D to the centre a
M

B
of the square. [Kamataka 88]
1m (Ans. 6.27 x 10- 7 J)
16. Positive charges of 6, 12 and 24 nC are placed at the
q three vertices of a square. What charge must be
0 2m A placed at the fourth vertex so that total potential at
(b)
the centre of the square is zero? (Ans. - 42 nq
Fig. 2.15 17. Two equal charges, 2.0 x 10-7 C each are held fixed
at a separation of 20 cm. A third charge of equal
10. Two small spheres of radius 'a' each carrying magnitude is placed midway between the two
charges + q and - q are placed at points A and B, charges. It is now moved to a point 20 ern from both
distance'd' apart. Calculate the potential difference the charges. How much work is done by the electric
between points A and B. (Ans.2q/4TtEOd) field during the process? (Ans. - 3.6 x 10-3 J)
2.10 PHYSICS-XII

18. ABC is a right-angled triangle, where AB and BC


6. V= t'J + V2= _1_ [ ql + q2 ]
are 25 em and 60 em respectively; a metal sphere of 41tEO '1 r2
2 em radius charged to a P9tential of 9 x 105 V is
placed at B. Find the amount of work done in 9 [ 10 x 10-6 20 x 10-6 ]
= 9 x 10 + --:--:- _
carrying a positive charge of 1 C from C to A. 0.01 0.01
A (Ans. 0.042 J)
= 27 x 106 V = 27 MV.
7. Let the point P of zero potential lie at distance x
from the charge q.

q x d -x -2q
o~--~I--~~--------~o

R
~c A P B

Fig. 2.18 .. _1_.9 ..+ _1_. (-2q) =0 or.! =_2_ or x =:!..


41tEo X 41tEo(d - x) x d- x 3
HINTS
8. Suppose the point of zero potential is located at
w

SI
6J
1. V=-=-=2V. distance x metre from the charge of + 100 u C, Then
q 3C
6
V = _1_ [100 x 10- _ 400 x 10-6] = 0.
2. As V = _1_}. . . 50 = 9 x 109 x-.!L.
41tEo r 0.9 41tEo x 0.30 - x

_ 50 x 0.9 _ 5 lO-9 This gives x = 0.06 m = 6 em i.e., the point of zero


or q----9 - x C
9 x 10 potential lies at 6 em from the charge of + 100 f,lc.
As the potential is positive, the charge q must be _ 9 [ 1.0 x 10-6 1.0 x 10-6 ]
9. VA - V B - 9 x 10 - ---
positive. 2.0 1.0
IT
3. Electric field of a point charge, =-4500V
E= _1_ . !L = 20 NCI As potential is a scalar quantity, so change in position
41tEo r2
of the charge does not affect the value of potential.
Electric potential of a point charge,
10. VB- VA =_1_.~ __ l_.=.!L-.-3L
V = _1_ . .1 = 10 JCI 41teo d 41tEo d 41teo d
41tEo r
H
2 10-6 5 10-6]
Clearly, distance r = V = 10 = 0.5 m 11. VA = 9 x 109 x - x = - 7.8 x 105 V
[ 0.15 0.05
E 20
Magnitude of charge, 2 x 10-6 5 x 10-6]
10 x 0.5 -9 Ve = 9 x 109 - = 0.6 x 105 V
q=41tEo' V.r=---9 =0.55x10 C. [ 0.05 0.15
O

9 x 10
W = q (Ve - VA) = 3.0 x 10-6(0.6 x 105 + 7.8 x 105)
4. Here' q=2f,lC=2x10-6C,rA=2m,rB=lm
=2.52 J.
V
A
- V =
B
-q-[~_1.]
41tEo rA rB
12. V - _1_ [~
e - 41tEo AC
+
BC
.31...J

~J
M

6 9
= 2 x 10- x 9 x 10 [~- V 9 [2.0 x 10- 6 1.0 x 10- 6 ]
= 9 x10 + -----
= - 9 x 10 V. 3 -Ii x 0.05 0.05
5. (i) Potential at the surface,
=9000[ 2+.fi]V
1 q 4 x 10-7 x 9 x 109 .fix 0.05
V=--.-=-----
41tEO r 0.1
V
=
1 [ql q2
47tEO AD + BD
J
= 36000 V = 36 kV. D

(ir) Potential inside a hollow conductor is the same . 0:9 [2.0 x 10-6 1.0 x 10-6]
as on its surface. =9x1 +-;=--
0.05 .fi x 0.05
(iii) When r = 0.6 m,
9 x 109 x 4 x 10-7 = 9000 [ 2.fi + 1 ] V
V = = 6000 V = 6 kV. .fi x 0.05
0.6
ELECTROSTATIC POTENTIAL AND CAPACITANCE 2.11

W= q(Vo - Vc) c
= 1.0 x 10-6 x 9000 [2..fi + 1 - 2 - ..fi]
-Ii x 0.05
= 0.053 J.
13. Diagonal of the square = ~( ..fi)2 + (..fi)2 = 2 m

Distance of each charge from the centre,


r = Half diagonal = 1m
:. Potential at the centre of the square is
9 [2 x 10-6 2 x 10-6 3 x 10-6 6 x 10-6]

R
V=9x10 -----------+--- Fig. 2.19
1 1 1 1
W = q(Vc - Vo)
= 2.7 x 104 V. = - 2 x 10-7 x 1.8 x 104 = - 3.6 x 10-3 J.
14. AC = ~42 + 32 = 5crn = 0.05 m, AD= BC = 0.03 m 18. Potential of the charged sphere is

SI
V=_l_.!1.
1 [ 1.0 x 10-11 2.0 x 10-11 1.0 x 10-11 ] 41tEO r
V=-- - +----
41teo 0.04 0.05 0.03
. . 9 x 105 = 9 x 109 x -q-
0.02
=1.65 V.
or 0.02 2 -6 C
q = 104 = x 10 = 21-1
9 [2 x10-9 4 x 10-9 8 x 10-9 ]
15. V0 = 9 x 10 + + ---
0.2 0.2 Ji 0.2 Potential at A due to charge q is

= 577.26 V 1 q 9 x 109 x 2 x 10-6


VA=--.-= V
IT
9 [2 x 10-9 4 x 10-
9
8 x 10-9 ] 41tEO r 0.25
V =9xlO + +----r~
o 0.1..fi 0.1..fi 0.1..fi Potential at C due to charge q is
9 x 109 x 2 x 10-6
= 890.82 V Vc= V
0.60
9
W = q [V 0 - VoJ = 2 x 10- [890.82 - 577.26] Potential difference between A and C is
H
= 6.27 x 10-7 J. VA - Vc = 1.8 x 10-3 [_1 1_] V
16. Suppose a charge of q nC be placed at the fourth 0.25 0.60
vertex. 1.8 x 7
Let length of half diagonal be x metre. = --V = 0.042 V
300
6 x 10-9 12 x 10-9 24 x 10-9
O

9 [ Work done in moving a charge of + 1C from C to A


Vo = 9 x 10 + + ---
x x x W = q (VA - Vc) = 1 x 0.042 = 0.042 J.
9
q x 10- ]
+ =0
x 2.7 RELATION BETWEEN ELECTRIC FIELD
AND POTENTIAL
~ + 12 + 24 + 3.. = 0
M

or
x x x x 10. Show that the electric field at any point is equal to
3..= _ 42 the negative of the potential gradient at that point.
or
x x Computing electric field from electric potential. As
or q = -42 nC. shown in Fig. 2.20, consider the electric field due to
17. The situation is shown in Fig. 2.19. charge + q located at the origin O. Let A and B be two
adjacent points separated by distance dr. The two
" 1 [ 2 x 10-7 2 x 1O~7]"
V -V =-- +---
C 0 41tto 0.20' 0.20
7 7 +q
1 [2 x 10- 2 x 10- ]
- 41t to 0.10 + 0.10 a
• B A

= - 1.8 x 10-4 V
Fig. 2.20 Relation between potential and field.
2.12 PHYSICS-XII

~
points are so close that electric field E between them
remains almost constant. Let V and V + dV be the or
potentials at the two points.
The external force required to move the test charge ...• ~
~ where VI and V2 are the potentials at '1 and r2
qo (without acceleration) against the electric field E is
~ -
given by respectively. If we take 'i at infinity, then VI =0 and
~ ~ ~ ~
F = -qo E put r2 = r, we get
111ework done to move the test charge from A to B is ...•
= F . dr = - qo E . dr ~ f ~E.dr ~ r

R
W V(r)=-
Also, the work in moving the test charge from A to
B is Hence by knowing electric field at any point, we
W = Charge x potential difference can evaluate the electric potential at that point.
= qo (VB - VA) = qo dV 12. Show that the units volt/metre and newton/

SI
Equating the two works done, we get coulomb are equivalent. To which physical quantitlj do
- qoE . dr = qo . dV thelj refer ?
SI units of electric field. Electric field at any point is
or E=- dV equal to the negative of the potential gradient. It
dr suggests that the SI unit of electric field is volt per metre.
The quantity dV is the rate of change of potential But electric field is also defined as the force
dr experienced by a unit positive charge, so SI unit of
with distance and is called potential gradient. Thus the electric field is newton per coulomb. Both of these units
IT
electric field at any point is equal to the negative of the are equivalent as shown below.
potential gradient at that point. The negative sign shows volt joule / coulomb
that the direction of the electric field is in the direction metre metre
of decreasing potential: Moreover, the field is in the newton - metre newton
direction where this decrease is steepest. coulomb - metre coulomb
From the above relation between electric field and or 1Vm-1 =lNC1
H
potential, we can draw the following important con-
clusions:
(i) Electric field is in that direction in which the
potential decrease is steepest.
.•. -
-
. .. .
Exam /es based on
•.
•.
Electric F.ield
O

(il) The magnitude of electric field is equal to the


change in the magnitude of potential per unit Formulae Used
displacement (called potential gradient) normal 1. Electric field in a region can be determined from
to the equipotential surface at the given point. the electric potential by using relation,
E= _ dV
11. How can we determine electric potential if electric dr
M

field is known at any point ?


or E =_ av F =_ av r:- = _ av
Computing electric potential from electric field. x ax ' -y Dy , '-z az
The relation between electric field and potential is
2. Electric field between two parallel conductors,
~ dV ~ ~
E = - -- ~ or dV =- E. dr E= V
dr d
~ 3. Electric potential in a region can be determined
Integrating the above equation between points r1
from the electric field by using the relation,
~ r ~ ~
and r2, we get
V =- f E .dr
00

Units Used
E is in NC -lor Vm - \ V in volt, r in metre.
ELECTROSTATIC POTENTIAL AND CAPACITANCE 2.13

Example 15. Find the electric field between two metal But V-==3 x 106 V
plates 3 mm apart, connected to 12 V battery. . . 3 x 106 r ==3 x 106
Solution. Electric field, or r ==1m.
E ==V == 12 V ==4 x 103 Vm-1
d 3 x 10-3 m Example 19. A uniform electric field E of 300 NC 1
is
directed along negative X-axis. A, Band C are three points
Example 16. Calculate the voltage needed to balance an oil
in the field, having x and y coordinates (in metre), as shown
drop carrying 10 electrons when located between the plates
in Fig. 2,21. Find the potential differences to. VBA' to. VCBand
of a capacitor which are 5 mmuparitg« 10 ms" 2). The mass
to.VCA·
of oil drop is 3 x 10- 16 kg.
Solution. q ==ne ==10 x 1.6 x 10- 19 C y

R
C(-3,4) B (4, 4)
m==3 x 10-16 kg, d ==5mm ==5x 10-3 m
.~----~--------~--
E== V == V Vm-1
d 5 x 10- 3

For the charged oil drop to remain stationary in

SI
A (4, 1)
electric field, E'~----~--------~--
qE ==mg ------------~o+---------------~~x
10 x 1.6 x 10-19 x V 3 ==3x 10-16 x 10
5 x 10-
3 x 10-16 x 10 x 5 x 10-3
or V == ==9.47 V.
10 x 1.6 x 10-19 Fig. 2.21
IT
Example 17. An infinite plane sheet of charge densitJj Solution. (i) No work is done in moving a unit
10- 8 Cm 2 is held in air. In this situation how far apart are positive charge from A to B because the displacement
two equipotential surfaces, whose p.d. is 5 V ? of the charge is perpendicular to the electric field. Thus
Solution. Electric field of an infinite plane sheet of the points A and B are at the same potential.
charge, .. to.VBA==0
E==~ (ii) Work is done by the electric field as the positive
H
21:0 ~
charge moves from B to C ii.e., in the direction of E).
If M is the separation between two equipotential Thus the point C is at a lower potential than the point R
surfaces having potential difference to. V, then As E==_to.V
E ==to.V Sx
O

M to.VCB==- E f'o.x ==-300 Ne1 x 7m


(j to.V
==- 2100 V.
21:0 M
(iii) Points A and B lie on an equipotential surface.
M==21:0to.V ==2 xB,B5x 10-12 x 5 So VB ==VA
or
10-8
M

(j
to.VCA ==VC-VA ==VC-VB==to.VCB
==B.B5 x 10-3 m ==8.85 mm. ==-2100 V.
Example 18. A spark passes in air when the potential Example 20. Three points A, Band C lie in a uniform
gradient at the surface of a charged conductor is electric field (E) of5 x 103 NC1 as shown in the figure. Find
3 x 106 Vm-1. What must be the radius of an insulated the potential difference between A and C. [CBSE F 09]
metal sphere which can be charged to a potential of 3 x 106 V ~
before sparking into air ?
Solution. Potential gradient, ,, I
13cm
dV ==3x 106 Vm-1 5cm',
~E
dr I

'IC
or dV ==3 x 106 dr
or V ==3 x 106 r Fig. 2.22
2.14 PHYSICS-XII

Solution. Points B and C lie on an equipotential

"'
surface, so Vc = VB"
P'D. between A and C = P'D, between A and B 20cm:
,

9
=-Eill

= -Sx 103NC1 x 4x 10-2 m [.: E = - ~J 60~~~~


, '

= -200 V. [ill = AB = ~S2 _32 = 4 em]


30V
Example 21 . If the potential in the region of space around
the point (-1m,2 m,3 m) is given by 20V
V = (10x2 + Sy2 -3z2) volt, calculate the three components

R
Fig. 2.23
of electric field at this point.
Solution. Here x = -1 m, y =2. m, z =3 m HINTS
As V = lOx2 + Sl-3z2 1. L'1 V = - E L'1 r = - 20 x (-~) = 10 V.
100

SI
E =--=--(10x
av a 2
+Sy
2
-3z)
2
x ax ax 2. ~V = - So L'1x =- 30(10 - 4) = -180 V.
= -20x =-20 x (-1)=20 Vm- 1
. 3. ~ V =- So L'1 x - E,. L'1 Y = - 20 x 2 - 30 x 2 = -100 V.

av a 2 2 2 4. .: 51unit of electric field = NC-1


E =-=--(lOx +Sy -3z )=-10y
Y 8y dy :. 51unit of A = NC-1 x m3 = Nm3 C-1
(x,y,z) Ad A
=-10x2 =-20Vm-1
Potential, V =- f -;x = -2
2x
.

=_av 00
IT
E =-~(10x2+Sy2-3z2)=6z
Z az dz 5. For the equipotential surface of 60 V,
= 6 x 3 = 18 Vm -1. 60 V = kq =--'.:L
r 0.10 m
j2)roblems For Practice
or kq = 60 V x 0.10 m = 6 Vm
1. A uniform electric field of 20 NC-1 exists in the
E-- kq _£ V -1
H
vertically downward direction. Determine the •. 2 - 2 m
r r
increase in the electric potential as one goes up
through a height of 50 cm. (Ans. 10 V) Clearly, E decreases with r. The direction of electric
field will be radially outward because V decreases
2. A uniform electric field of 30 NC-1 exists along the
with r.
X-axis. Calculate the potential difference VB - VA
O

between the points A (4 m, 2 m)and B(10 m, 5 m).


(Ans. - 180 V) 2.8 EQUIPOTENTIAL SURFACES AND THEIR
----t 1\ 1'1 1 PROPERTIES
3. An electric field E = 20 i + 30 j NC - exists in free
13. What is an equipotential surface ? Give an
space. If the potential at the origin is taken zero, example.
determine the potential at point (2 m, 2 m).
M

Equipotential surface. Any surface that has same


(Ans. -100 V)
electric potential at every point on it is called an equipo-
4. The electric field in a region is given by E = x~ I, tential surface. The surface may be surface of a body or a
surface in space. For example, as we shall see later on,
Write the 51unit for A. Write an expression for the the surface of a charged conductor is an equipotential
potential in the region assuming the potential at
infinity to be zero. ( Ans. Nm 3C- 1,;:2) surface. By joining points of constant potential, we can
draw equipotential surfaces throughout the region in
which an electric field exists.
5. Figure 2.23 shows some equipotential surfaces. What
14. State and prove the important properties of
can you say about the magnitude and the direction of
equipotential surfaces.
the electricfield ?
Properties of equipotential surfaces: 1. No work is
(Ans. E = ~ Vm -1, radially outward) done in moving a test charge over an equipotential surface.
r
ELECTROSTATIC POTENTIAL AND CAPACITANCE 2.15

separated by a small distance, (iii) two point charges + q and


+ q separated by a small distance and (iv) a uniform electric
field.
Equipotential surfaces of various charge systems.
For the various charge systems, we represent equipo-
tential surfaces by dashed curves and lines of force by
Equipotential
full line curves. Between any two adjacent equipotential
surface
surfaces, we assume a constant potential difference.
(i) Equipotential surfaces of a positive point
Fig. 2.24 An equipotential surface.
charge. The electric potential due to a point charge qat

R
Let A and Bbe two points over an equipotential surface, distance r from it. is given by
as shown in Fig. 2.24. If the test charge qo is moved from V=_l_.!l.
A to B, the work done will be 41t EO r
WAB = Charge x potential difference This shows that V is constant if r is constant. Thus,
= qo (VB - VA) the equipotential surfaces of a single point charge are

SI
As the surface is equipotential, so VB - VA =0 concentric spherical shells with their centres at the
point charge, as shown in Fig. 2.25. As the lines of force
Hence WAB =0.
point radially outwards, so they are perpendicular to
2. Electric field is always normal to the equipo- the equipotential surfaces at all points.
tential surface at every point. If the field were not normal
to the equipotential surface, it would have a non-zero
component along the surface. So to move a test charge
against this component, a work would have to be done.
IT
But there is no potential difference between any two
points on an equipotential surface and consequently
no work is required to move a test charge on the
surface. Hence the electric field must be normal to the
equipotential surface at every point.
3. Equipotential surfaces are closer together in the
H
regions of strong field and farther apart in the regions
of weak field. We know that electric field at any point
is equal to the negative of potential gradient at that
Fig. 2.25 Equipotential surface of a +ve point charge.
point.
E=- dV dV (ii) Equipotential surfaces of two equal and opposite
i.e.,
O

or dr=--
dr E point charges : Electric dipole. Fig. 2.26 shows the
For the same change in the value of dV i.e., when equipotential surfaces of two equal and opposite charges,
dV = constant,we have + q and - q, separated by a small distance. They are close
1 together in the region in between the two charges.
dr «: -
E
M

Thus the spacing between the equipotential


surfaces will be smaller in the regions, where the
electric field is stronger and vice versa.
4. No two equipotential surfaces can intersect each
other. If they interesect, then there will be two values
of electric potential at the point of intersection, which
is impossible.

2.9 EQUIPOTENTIAL SURFACES OF


VARIOUS CHARGE SYSTEMS
15. Sketch and explain the equipotential surfaces for: Fig. 2.26 Equipotential surfaces for two equal and
(i) a point charge, (ii) two point charges + q and - q, opposite charges.
2.16 PHYSICS-XII

(iii) Equipotential surfaces of two equal positive done in moving one charge at infinite distance from
charges. Fig. 2.27 shows the equipotential surfaces of the other. But when they are brought closer to one
two equal and positive charges, each equal to + q, sepa- another, work has to be done against the force of
rated by a small distance. The equipotential surfaces repulsion. As electrostatic force is a conservative force,
are far apart in the regions in between the two charges, this work gets stored as the potential energy of the two
indicating a weak field in such regions. charges.
The electric potential energy of a system of point charges
may be defined as the amount of work done in assembling the
charges at their locations by bringing them in, from infinity.
18. Deduce expressions for the potential energy of a

R
system of two point charges and three point charges and hence
generalise the result for a system of N point charges.
Potential energy of a system of two
point charges. Suppose a point charge ql ,
is at rest at a point PI in space, as shown in ,
,r

SI
Fig. 2.27 Fig. 2.29. It takes no work to bring the first ,

charge ql because there is no field yet to


(iv) Equipotential surfaces for a uniform electric work against.
field. Fig. 2.28 shows the equipotential surfaces for a
uniform electric field. The lines of force are parallel
straight lines and equipotential surfaces are equidis-
tant parallel planes perpendicular to the lines of force.
Fig. 2.29 P.E. of two point charges.
Equipotential
surfaces Electric potential due to charge ql at a point P2 at
IT
.r>:
:---<..... r--- r--- distance r12 from PI will be
~ !1l
VI =_1_ .
~ 41t EO 'i2
'-~
'-~ If charge q2 is moved in from infinity to point P2, the

-- -- -- f-~ work required is


H
W2 = Potential x charge
Fig. 2.28 Equipotential surfaces for a uniform
electric field. = VIx q2 =_1_. qlq2
41t EO '12
16. Give the importance of equipotential surfaces.
O

Importance of equipotential surfaces. Like the lines As the work done is stored as the potential energy
of force, the equipotential surfaces give a visual picture U of the system (ql + q2)' so
~
of both the direction and the magnitude of field E in a U = WI + W = _1_. ql q2
2
region of space. If we draw equipotential surfaces at 41t EO r12
regular intervals of V, we find that equipotential
M

surfaces are closer together in the regions of strong Potential energy of


field and farther apart in the regions of weak field. a system of three point
~ charges. As shown in
Moreover, E is normal to the equipotential surface at
every point. Fig. 2.30, now we bring
in the charge q3 from
2.10 ELECTRIC POTENTIAL ENERGY infinity to the point P3.
Work has to be done
17. What is meant by electric potential energy of a
against the forces
charge system ?
exerted by ql and q2·
Electric potential energy. It is the energy possessed
by a system of charges by virtue of their positions.
When two like charges lie infinite distance apart, their Fig. 2.30 P.E. ofthree
potential energy is zero because no work has to be point charges.
ELECTROSTATIC POTENTIAL AND CAPACITANCE 2.17

Therefore ~ Positive potential energy implies that work can be


W3 = Potential at point P3 due to q1 and q2 obtained by releasingthe charges, while negative potential
x charge q3 energy indicates that an external agency will have to do
work to separate the charges infinite distance apart.
or W3 = _1_ [!!.L+ 31..] x q3 = _1_ [q1q3 + q2q3] ~ Electric potential is a characteristic of an electric field, it
41t EO r13 123 41t EO r13 r23 does not matter whether a charged object is placed in
Hence the electrostatic potential energy of the that field or not. It is measured in JC-1 or volt. On the
system q1 + q2 + q3 is other hand, electric potential energy is the energy of a
U = Total work done to assemble the three charges charged object in an external electric field. More
precisely, it is the energy of the system consisting of the
=W1 + W2 + W3
charged object and the external electric field (or charges

R
producing that field). It is measured in joule.
or U = _1_ [q1q2 + q1q3 + q2q3]
41t EO r12 r13 r23
Potential energy of a system of N point charges.
2.11 POTENTIAL ENERGY IN AN
EXTERNAL FIELD
The expression for the potential energy of N point
charges can be written as 19. Write an expression for the potential energy of a

SI
U=_l_ L qiqj =.!.._1_ f f fJiqj
single charge in an external field. Hence define electric
potential.
41t EO all pairs 'ij 2 41t EO i =1 j=1 'i;
Potential energy of a single charge. We wish to
i 7' j
determine the potential energy of a charge q in an
As double summation counts every pair twice, to ~
avoid this the factor 1/2 has been introduced. external electric field E at a point P where the corres-
ponding external potential is V. By definition, V at a
NOT E The potential at jth charge due to all other point P is the amount of work done in bringing a unit
charges can be written as positive charge from infinity to the point P. Thus, the
IT
V.
}
= I 3£
k= 1 r'k
work done in bringing a charge q from infinity to the
point P will be qV, i.e., W = qV
k7'j }
This work done is stored as the potential energy of
The expression for P.E. of N point charges can be written as ~
the charge q. If r is the position vector of point P
U=1. ~ q,[ _1_ I qk ]=1. I q.Y.
41t EO k= 1 rl
relative to some origin, then
2j=1 } 2j=1 } } ~ ~
H
.
k7' } } U(r)=qV(r)
P. E. of a charge in an external field
For Your Knowledge = Charge x external electric potential

~ Electric potential energy is a scalar quantity. While As V= U


O

finding its value, the value of various charges must be q


substituted with their proper signs. So we can define electric potential at a given point in
~ The potential energy of two like charges (ql q2 > 0) is positive. an external field as the potential energy of a unit positive
As the electrostatic force is repulsive, so a positive charge at that point.
amount of work has to be done against this force to 20. Write an expression for the potential energy of
bring the charges from infinity to a finite separation.
M

two point charges ql and Q2' separated by distance rin an


~ The potential energy of two unlike charges (ql q2 < 0) is
negative. As the electrostatic force is attractive, so a electric field E.
positive amount of work has to be done against this Potential energy of a system of two point charges in
force to take the charges from the given locations to ~ ~
infinity. Conversely, a negative amount of work is an external field. Let V (r1) and V (r2) be the electric
needed to bring the charges from infinity to the present
~
potentials of the field E at the points having position
locations, so the potential energy is negative. ~ ~
~ As electrostatic force is a conservative force, so the vectors r1 and r2 as shown in Fig. 2.31.
potential energy of a charge configuration is inde- ~
pendent of the manner in which the charges are Work done in bringing q1 from 00 to r1 against the
assembled to the present locations. The potential energy external field
is a characteristic of the present state of configuration,
not on how this state is attained.
2.18 PHYSICS-XII

2.12 POTENTIAL ENERGY OF A DIPOLE IN


A UNIFORM ELECTRIC FIELD

--
22. Derive an expression for the potential energy of a
dipole in a uniform electric field. Discuss the conditions of
stable and unstable equilibrium.

0
Potential energy of a dipole placed in a uniform
'I \ electric field. As shown in Fig. 2.32, consider an electric
I Source ~
dipole placed in a uniform electric field E with its
, charges
-='=- - ofE Q2V Cr;) ~
dipole moment p making an angle 8 with the field.

R
Fig. 2.31 P.E. of two charges in an external field. Two equal and opposite forces + q E and - q E act on
~ its two ends. The two forces form a couple. The torque
Work done in bringing q2 from OC! to '2 against the exerted by the couple will be
external field 't = qE x 2a sin 8 = pE sin 8

SI
~ where q x 2 a = p, is the dipole moment.
= q2V( '2)
Work done on q2 against the force exerted by q1 +q
.0....:.... ...•. +qE
->

_1_ q1q2
4m:o' '12 ->
-----------r~~~--~----~E
where '12 is the distance between q1 and q2' : 2a sin e
->
= The work -qE .•.••. --a:
IT
Total potential energy of the system -q
done in assembling the two charges

~ ~ 1 q1q2 Fig. 2.32 Torque on a dipole in a uniform electric field.


or U=q1V( 1J)+q2V('2)+--'-
41t1:o '12
1£ the dipole is rotated through a small angle d8
21. Define electron volt. Express it in joule. against the torque acting on it, then the small work
H
Units of electrostatic potential energy. Suppose an done is
electron (q = 1.6 x 10-19 C) is moved through a potential dW = 't d8 = pE sin 8 d8
difference of 1volt, then the change in its P.E. would be
The total work done in rotating the dipole from its
I:J.U = q I:J.V =1.6 x 10-19 ex 1 v =1.6 x 10-19 J orientation making an angle 81, with the direction of
O

This is a commonly used unit of energy in atomic the field to 82 will be


physics and we call it electron volt (eV).
82

= f dW = f pE
Thus electron volt is the potential energy gained or lost by
W sin 8 d8
an electron in moving through a potential difference of 1 volt.
~
1 eV = 1.6 x 10-19 J
M

= pE [- cos 8]~ = pE (cos 81 - cos 82)


Multiples and submultiples of eV
1meV (milli electron volt) This work done is stored as the potential energy U
= 10- 3 eV = 1.6 x 10-22 J of the dipole.
1keY (kilo electron volt) .. U = pE (cos 81 - cos 82)
= 103 eV = 1.6 x 10-16 J
1£ initially the dipole is oriented perpendicular to
1MeV (million electron volt)
the direction of the field (81 = 90°) and then brought to
= 106 eV = 1.6 x 10-13 J some orientation making an angle 8 with the field
1 GeV (giga electron volt) (82 = 8), then potential energy of the dipole will be
= 109 eV =1.6 x 10-to J U = pE (cos 90° - cos 8) = pE (0 - cos 8)
1TeV (tera electron volt) ~ ~
= 1012 eV = 1.6 x 10-7 J. or U = - pE cos 8 = - P . E
ELECTROSTATIC POTENTIAL AND CAPACITANCE 2.19

Special Cases Example 22


1. Position of stable equilibrium. When e =0°, (a) Determine the electrostatic potential energy of a
system consisting of two charges 7 ~C and - 2 ~C
u = - pE cos 0° = - pE (and with no external field) placed at (-9 em,O,O)
Thus the potential energy of a dipole is minimum and (9 em, a, 0) respectively.
when its dipole moment is parallel to the external (b) How much work is required to separate the two
field. This is the position of stable equilibrium. charges infinitely away from each other ?
2. Position of zero energy. When e = 90°, (c) Suppose the same system of charges is now placed in
U = - pE cos 90° =0. an external electric field E = A (1/ 1) ;
Thus the potential energy of a dipole is zero A =9 x 105 Cm-2. What would the electrostatic
when it is held perpendicular to the external energy of the configuration be ? [NCERT]

R
field. This can be explained as follows. If we Solution. (a) q1 = 7 ~C = 7 x 10-6 c,. q2 = - 2 x 10-6 C,
hold the dipole perpendicular to the electric r=18 em =0.18 m
field and bring it from infinity into the field,
Electrostatic potential energy of the two charges is
then the work done on charge + q by the
external agent is equal to the work done on U=_I_. q1 q2

SI
charge - q. The net work done on the dipole will 41t eo r
be zero and hence its potential energy is zero. 9 x 109 x 7 x 10-6 x (- 2) x 10-6
--------'-----'--- = -0.7 J.
3. Position of unstable equilibrium. When e = 180°, 0.18
U = - pE cos 180° = + pE (b) Work required to separate two charges infinitely
Thus the potential energy of a dipole is maximum away from each other,
when its dipole moment is antiparallel to the external W = U2 - U1 =0 - U =-(-0.7)=0.7 J.
field. This is the position of unstable equilibrium. (c) Energy of the two charges in the external electric field
IT
= Energy of interaction of two charges with the
Examples based on
external electric field
Electric Potential Energy
+ Mutual interaction energy of the two charges
Formulae Used = q1V (r1) + q2V (r2) + _1_ ql~2
1. Electric potential energy of a system of two point 4m;0 r:
charges,
H
U=_I_. q1q2 = q1 A + q2 A + _1_ q1q2
1. r2 4m;0 1
41t eo 'i2

2. Electric potential energy of a system of N point = [7~C + -2~C] x 9 x 105 Cm-2 -0.7 J
charges, 0.09 m 0.09m
O

U=_l_ L qjqk = (70 -20) -0.7 = 50 - 0.7 = 49.3 J.


41t eo all pairs rjk
Example 23. Three charges - q, + Q and - q are placed at
3. Potential energy of an electric dipole in a uniform equal distances on a straight line. If the potential energy of
electric field, the system of three charges is zero, find the ratio Q / q.
u= - pE(cos 82 -cos~) Solution. As shown in Fig. 2.33, suppose the three
M

If initially the dipole is perpendicular to the field charges are placed at points A, Band C respectively on
E, ~ = 90° and 82 = 8 (say), then a straight line, such that AB = BC = r.
-4 -4
-q +Q -q
U =- pE cos 8=- P . E •
A
•B •
C
If initially the dipole is parallel to the field E, I--- r '1' r ------I
~ = 0° and 82 = 8 (say), then
Fig. 2.33
U= - pE(cos 8-1) = pE(I-cos 8)
As the total P.E. of the system is zero, so
Units Used
Charges are in coulomb, distances in metre,
_1_ [- qQ + (- q)(-q) + Q(-q)] =0
energy in joule or in electron volt (eV) and dipole 41t EO r 2r r
moment in coulomb metre (Cm). or - Q + !t. - Q = a or 2 Q = !t. or Q =.!. = 1 : 4.
leV = 1.6 x 10-19 C, 1 MeV = 1.6 x 10-13 C. 2 2 q 4
2.20 PHYSICS-XII

Example 24. Two positive point charges of 0.2 /lC and Example 27. Four charges /~q -.qD
0.01 /lC are placed 10 em apart. Calculate the work done in are arranged at the corners of a
reducing the distance to 5 em. square ABCD of side d as
Solution. Here q1 = 0.2 x 1O-6C, q2 = 0.01 x 10-6C shown in Fig. 2.34. (i) Find the
d ,
work required to put together
Initial separation (r) = 10 em = 0.10 m this arrangement. (ii) A charge ,
Final separation (r ) = 5 em = 0.05 m qo is brought to the centre E of B __._----- .• C
f
Work done = Change in potential energy the square, the four charges -q +q
being held fixed at its corners. Fig. 2.34
= Final P. E. - Initial P. E.

__ 1_ q1q2 __ 1_ q1q2 - q1q2 [~-.!l How much extra work is


needed to do this ? [NCERT ; CBSE F 15]

R
- 4m,0· r 4m,o '; - 41tEo r '; Solution. (i) Given AB = BC = CD = AD = d
f f

= 0.2 x 10-6 x 0.01 x 10-6 x 9 x 109 [_1


0.05
l_J
0.10
.. AC= BD=~d2 + d2 =..fi d
Work required to put the four charges together
= 1.8 x 10-4 J.

SI
= Total electrostatic P.E. of the four charges
Example 25. Two electrons, each moving with a velocity of
106 ms-l, are released towards each other. What will be the =_I_[qAqB + qAqC+ qAqO+ qBqC+ qBqO+ qcqo]
47t EO AB AC AD BC BD CD
closest distance of approach between them ?
Solution. Let ro be the distance of closest approach _ 1 [q2 q2 q2 q2 q2 q2]
- 41tEO - d + J2d - d - d + J2 d - d
of the two electrons. At this distance, the entire K.E. of
the electrons changes into their P.E. Therefore,
1 2 1 2 1 ee
=-L(4-J2).
-mv +-mv =--- 47t Eo
IT
2 2 41tEo ~
(ii) Extra work needed to bring charge qo to centre E
e2 9 x 109 x (1.6 x 10-19)2
1
r, = --. -- = ----'-,;-;---~;O-'--- W = qo x Electrostatic potential at E due to the
o 41tEO mv2 9.1 x 10-31 x (106)2 four charges
= 2.53 x 10-tO m.
-q[ q + -q
Example 26. Two particles have equal masses of5.0 g each - 0 41tEo(d I ..fi) 41tEo(d I ..fi)
H
and opposite charges of +4 x 10-5 C and -4.0 x 10-5 C. They
are released from rest with a separation of 1.0 m between
+ q + -q ]-0-.
o(dl..fi) o(dl..fi)
41tE 41tE
them. Find the speeds of the particles when the separation is
reduced to 50 em. Example 28. Three point
Solution. Here m = 5.0 g = 5 x 10-3 kg. charges, +Q,+2Q and -3Q A(+Q)
O

are placed at the vertices of an


q =±4x 1O-5C, r1 =1.0 m, r2 =50 em =0.50 m
equilateral triangle ABt of side I
Let v = speed of each particle at the separation of (Fig. 2.35). If these charges are
50 cm. displaced to the midpoints ~,
From energy conservation principle, 1\ and C1 respectively, find the
M

K.E. of the two particles at 50 em separation amount of the work done in B(+ 2Q) B1 q- 3Q)
+ P.E. of the two particles at 50 em separation shifting the charges to the new
locations. [CBSE OD 2015] Fig. 2.34
= P.E. of the two particles at 1.0 m separation
.! mv2 + .! mv2 + _1_. q1 q2 = _1_. q1 q2 Solution. ~ 1\ = 1\ C1 = ~ C1 = AB =.i
2 2
2 2 47tEO r2 41tEO r1
Initial P.E. of the system is
mv2 = q1q2 [~_
41tEO r1
~Jl
r2
or v2 = q1q2 [r2 -
41tEOm
r1]
r1r2 u. = _1_[QX2Q + 2Qx (-3Q) + QX(-3Q)]
I 41tEo I I I
v2 = 4 x 10-5 x (-4 x 10-5) x 9 x 109 [0.50 -1.0]
5 x 10-3 1.0 x 0.50 __ 1_7Q2
41tEO' I
= 2880 or v = 53.67 ms-t.
ELECTROSTATIC POTENTIAL AND CAPACITANCE 2.21

Final P.E. of the system is Estimate the heat released by the substance in aligning its
dipoles along the new direction of the field. For simplicity
U ==_1_[QX2Q + 2Qx(-3Q) + QX(-3Q)] assume 100% polarization of the sample. [NCERT]
f 41tEo 1/2 1/2 1/2
Solution. Here p ==10-29 Cm, E ==106 Vm -1,
1 14Q2 8 ==60°, N ==6 x 1023
- 41tEo ·-1-
Work required to bring one dipole from position
Work done == Uf - U, 8 ==0° to position e is
W == pE - pE cos 8 ==pE(1-cos 8)
1 14Q2 1 7Q2 1 7Q2
==---.-- + ---- ==---.-- . ==10-29 x 106(1- cos 60°) J ==0.5 x 10-23 J
41tEo I 41tEo I 41tEo I
Work required for one mole of dipoles

R
Example 29. An electric dipole of length 2 em is placed
==W x N ==0.5 x 10-23 x 6 x 1023 ==3.0 J
with its axis making an angle of 60° to a uniform electric
field of 105 uc:'. If it experiences a torque of 8J3 Nm, Heat released = Loss in P.E.= Work done = 3.0 J.
calculate the
jOrOblems For Practice
(i) magnitude of the charge on the dipole, and

SI
1. Two point charges + 10 ~C and - 10 ~C are
(ii) potential energlJ of the dipole. [CBSE OD 2000]
separated by a distance of 2.0 em in air. (i) Calculate
Solution. Here 2a ==2 em ==0.02 m, 8 ==60°, the potential energy of the system, assuming the
E ==105 NC\ 1: ==8J3 Nm zero of the potential energy to be at infinity.
(i) T == pE sin 8 ==q x 2a x E sin 8 (ii) Draw an equipotential surface of the system.
[CBSE D 04] (Ans. - 45 J)
8J3 == q x 0.02 x 105 x sin 60°
2. Two point charges A and B of values + 15 IlC and
+ 9 IlC are kept 18 em apart in air. Calculate the
== 8J3 x 2 ==8 x 10-3 C.
IT
or work done when charge B is moved by 3 cm
q 0.02 x 105 x J3
towards A. [CBSE OD 2000] (Ans. 1.35 J)
(ii) P.E. of the dipole is
3. Two point charges 20 x 1O-6C and -4 x 1O-6C are
U ==- pE cos e ==- q x 2a x E cos 8 separated by a distance of 50 cm in air. (i) Find the
==- 8 x 10-3 x 0.02 x 105 x cos 60° ==- 8 J. point on the line joining the charges, where the
electric potential is zero. (ii) Also find the electro-
Example 30. An electric dipole of length 4 em, when placed
H
static potential energy of the system. [CBSE OD OS]
with its axis making an angle of 60° with a uniform electric
field experiences a torque oj 4 J3 Nm. Calculate the (i) magni- [Ans. (i) 41 em from the charge of 20 x 10-6 C
tude of the electric field, (ii) potential energy of the dipole, if (ii) - 144 Jl
the dipole has charges of ± 8 nC. [CBSE OD 04 ; D 06C, 14] 4. Two charges, of magnitude 5 nC and - 2 nC, are
placed at points (2 em, 0, 0) and (x em, 0, 0) in a
2 a ==4 cm ==0.04 m,
O

Solution. Here 8 ==60°,


region of space, where there is no other external
T ==4 J3 Nm, q ==8 nC ==8 x 10- 9 C
field. If the electrostatic potential energy of the
Dipole moment, system is - O.5IlJ, what is the value of x ?
p == q x 2a ==8 x 10-9 x 0.04 ==0.32 x 10-9 Cm. [CBSE D OSC] (Ans. x = 4 em)
5. Three point charges are arranged as shown in
(i) As == pE sin 8
M

1:
Fig. 2.36. What is their mutual potential energy ?
E==__ 1:_ Take q ==1.0 x 10--4 C and a ==10 em. (Ans. 0.27 J)
P sin 8 0.32 x 10-9 x sin 60°

D
9
== 4 J3 x 10 x 2 ==2.5 x 1010 NCl.
0.32 x J3
(ii) U ==- pE cos e
==-0.32 x 10-9 x 2.5 x 1010 x cos 60° ==- 4 J. q a q
Example 31. A molecule of a substance has permanent Fig. 2.36 Fig. 2.37
electric dipole moment equal to 10-29 Cm. A mole of this
substance is polarized (at low temperature) applying a t:y 6. Determine potential energy of the charge configu-
ration shown in Fig. 2.37. ( q2 r; J
strong electrostatic field of magnitude (10 Vm-l). The Ans.--(-.,,2)
direction of the field is suddenly changed by an angle of 60°. 41t Eo a
2.22 PHYSICS-XII

7. Find the amount of work the field on the dipole (ii) the work which the
done in arranging the . external agent will have to do in turning the dipole
three point charges, on through 180° starting from the position e = 0°.
the vertices of an equi- [Ans. (i) 2 x 10-3 Nm (ii) 4 x 10-3 JJ
lateral triangle ABC, of
side 10 em, as shown in HINTS
the adjacent figure. 8e------,e C 1. U=_l_. qlq2
6~C - 6~C
[CBSE Sample Paper 2011] 41tEO r
(Ans. - 3.24 J ) 9 10 x 10-6 x(-10) x 10-6
=9xlO x 2 =-45J.
8. Calculate the work done to dissociate the system of 2.0 x 10-
three charges placed on the vertices of a triangle as

R
For equipotential surface, see Fig. 2.26 on page 2.15.
shown in Fig. 2.38. Here q = 1.6 x 10-10 C.
2. W = Final P.E. - Initial P.E.
[CBSE D 08; OD 13] (Ans. 2.304 x 10-8 J)

4~q:J~-{]
ql~:4
q
=

SI
= 9 x 109 x 15 x 10-6 x 9 x 10-6 [100 _ 100]
15 18
= 1.35 J.
3. (i) Suppose the point of zero potential is located at
- 4q ~----- __ + 2q
q,~q, distance x metre from the charge of 20 x 10-6 C.
lOan
Fig. 2.38 Fig. 2.39 Then, V = _1_ [20 x 10-6 _ 4 x 10-6] = 0
41tEo x 0.50 - x
9. What is the electrostatic potential energy of the
IT
charge configuration shown in Fig. 2.39 ? Take This gives x = 0.41 m = 41 em.
ql = + 1.0 x 10-8 C, q2 = - 2.0 x 10-8 C, (ii) U = _1_. qlq2
41tEo r
q3 = + 3.0 x 10-8 C, q4 = + 2.0 x 10-8
C
and a = 1.0 metre. (Ans. - 6.36 x 10-7 J) 9 x 109 x 20 x 10-6 x(-4) x 10-6
-------'--'--- = -1.44 J.
10. Three point charges + q, + 2q and Qare placed at the 0.50
H
three vertices of an equilateral triangle. Find the value 4. U = _1_ qlq2
of charge Q (in terms of q), so that electric potential 41tEo r
energy of the system is zero. (Ans. Q = - 2q / 3) -6 9 x 109 x 5 x 10-9 x(-2) x 10-9
11. An electron (charge = -e) is placed at each of the :. - 0.5 x 10 = 2
(x-2)xl0
eight comers of a cube of side a and an a-particle •
O

(charge = + 2e) at the centre of the cube. Calculate On solving, x = 4 em


the potential energy of the system. 3 q2
(Ans. 3.89 x 10lOe2 / a joule) 5. U=---
41tEo a
12. Two identical particles, each having a charge of
2.0 x 10-4 C and mass of 10 g, are kept at a 3 x9 x 109 x(1.0 x 10-4)3 = 0.27 J.
M

separation of 10 em and then released. What would 0.10


be the speeds of the particles when the separation
becomes large? (Ans. 600 ms ") 7. W =_l_[qAqB + qAqC + qBqC]
41tEo AB AC BC
13. Find the amount of work done in rotating an electric
dipole, of dipole moment 3.2 x 10-8 em, from its =_1_[!Li+ q(-q) + q(-q)]
position of stable equilibrium, to the position of 41tEo r r r
unstable equilibrium, in a uniform electric field of
-intensity 104 N / C. [CBSE Sample Paper 2011]
= - --
1
=-
l 9 x 109 x(6x10-6)2
J = - 3.24 J.
(Ans. 6.4 x 10-4 J) 41tEo r 0.10 ..
14. An electric dipole consists of two opposite charges 8. Initial P.E. of the three charges,
each of magnitude 11lC separated by 2 em. The
dipole is placed in an external electric field of U. = _1_ [ Ihq2 + q2q3 + qlq3]
1cPNC-1. Find (i) the maximum torque exerted by I 41tf.:o r r r
ELECTROSTATIC POTENTIAL AND CAPACITANCE 2.23

=_1_[q(-4q) + (-4q)x2q + qX2q] 13. Here '\ = 0°, 82 = 180°, P = 3.2 x 10-8 Cm,
41tEo r r r E = 104N/C

1 10q2 9x109x10x(1.6xlO-10)2 W = pE(cos,\ -cos82)


41tEo -r- = - 0.10 J =3.2xlO-8 x 104(cos OO-cos180°)
= 3.2 x 10-4 x(l + 1)= 6.4 x10-4 J.
= - 2.304 x 10-8 J
14. P = q x 2a = 10-6 xO.02 = 2 x 1O-8Cm
Final P.E., Uf = 0
Work required to dissociate the system of three (i) "max = pEsin 90° = 2 x 10-8 x 1ef xl
charges, = 2 x 10-3 Nm.

u, J.

R
W = U - = 2.304 x 10-8 (ii) W = pE(cos '\ -cos 82)
f
= 2 x 10-8 x 105(cos OO-cos 180°)
9. U = _1_ [q1q2 + q~3 + %q4 + q2q3 + q~4 + q3q4]
41tEo a v2a a a v2a a = 2 x 10-3(1+ 1) = 4 xlO-3 J.
9
=; 9 x 10 [(1)(-2) + (1)f]) + (1)(2) + (-2)(3)
-n 2.13

SI
1.0 CONDUCTORS AND INSULATORS
+ (-2) x (2) + (3)(2)] x 10-16 J 23. What are conductors and insulators? Why were
12 insulators called dielectrics and conductors non-electrics ?
9 x 109 x 10-16 7 Conductors and insulators. On the basis of their
12 J = -6.36 x 10- J.
behaviour in an external electric field, most of the
10. Suppose the charges + q, + 2q and Q are placed at materials can be broadly classified into two categories:
the comers A, Band C of an equilateral MBC of side 1. Conductors. These are the substances which allow
a. Then large scale physical movement of electric charges through
IT
_1_ [q x 2q + q x Q + 2q Q] = 0 x
them when an external electricfield is applied. For example,
silver, copper, aluminium, graphite, human body, acids,
41tEo r r r
alkalies, etc.
or 2q + Q + 2Q = 0 or Q = - 'lq / 3. 2. Insulators. These are the substances which do not
11. U = 9 x 109 [12 (- :)( - e) + 12 (- jt e)
allow physical movement of electriccharges through them when
an external electricfield is applied. For example, diamond,
H
glass, wood, mica, wax, distilled water, ebonite, etc.
+ 4 (-e)(-e) +8 (-e)(2e)]
.s; ../3a/2 The rubbed insulators were able to retain charges
placed on them, so they were called dielectrics. The
9 2
9 x 10 x 4 x e [3 + 2. + ~ _~ lJ rubbed conductors (metals) could not retain charges
12J3J3
O

a placed on them but immediately drained away the


36 x 109 e2 e2 charges, so they were called non-electrics.
---- [3 + 2.12 - 4.04] = 3.89 x 1010 - joule.
a a
12. Here q = 2.0 x 10-4 C, m = 10 g = 10-2 kg I
2.14 FREE AND BOUND CHARGES
r = lOcm = 0.10 m 24. Discuss the various free and bound charges
M

present in conductors and insulators.


Let v be the speed of each particle at infinite
separation. By conservation of energy, Free and bound charges. The difference between
the electrical behaviour of conductors and insulators
P.E. of two particles at the separation of 10 em can be understood on the basis of free and bound
= K.E. of the two particles at infinite separation charges.
_1_ . ql q2 = .!. mv2 + .!. mv2 In metallic conductors, the electrons of the outer
41t£o r 2 2 shells of the atoms are looselybound to the nucleus.
2 1 ql q) They get detached from the atoms and move almost freely
or v =--.---
41tEo rm inside the metal. In an external electric field, these free
9 x 109 x 2.0 x 10-4 x 2.0 x 10-4 4.
electrons drift in the opposite direction of the electric
0.10 x 10-2 = 36 x 10 field. The positive ions which consist of nuclei and
electrons of inner shells remain held in their fixed posi-
v= 600 ms-1• tions. These immobile charges constitute the boundcharges.
2.24 PHYSICS-XII

In electrolytic conductors, both positive and charges are induced on the right end of the conductor.
negative ions act as charge carriers. However, their ~
The process continues till the electric field Eind set up
movements are restricted by the external electric field
and the electrostatic forces between them. by the induced charges becomes equal and opposite to
~ ~ ~ ~
In insulators, the electrons are tightly bound to the the field Eext' The net field E (= Eext - Eind) inside the
nuclei and cannot be detached from the atoms, i.e., conductor will be zero.
charges in insulators are bound charges. Due to the
2. Just outside the surface of a charged conductor,
absence of free charges, insulators are poor conductors of
electric field is normal to the surface. If the electric
electricity.
field is not normal to the surface, it will have a
For Your Knowledge component tangential to the surface which will

R
immediately cause the flow of charges, producing
~ A third important category of materials is the surface currents. But no such currents can exist under
semiconductors which we shall discuss in chapter 14. static conditions. Hence electric field is normal to the
surface of the conductor at every point.
~ In metallic conductors, electrons of outer shells of the
atoms are the free charges while the immobile 3. The net charge in the interior of a conductor is

SI
positive ions are the bound charges. zero and any excess charge resides at its surface. As
~ In electrolytic conductors, both positive and negative
shown in Fig. 2.41, consider a conductor carrying an
ions are the free charges. excess charge q with no currents flowing in it. Choose a
Gaussian surface inside the conductor just near its
~ In insulators, both electrons and the positive ions are ~
the bound charges. outer boundary. As the field E =0 at all points inside
~ There is no clear cut distinction between conductors the conductor, the flux <It through the Gaussian surface
and insulators - their electrical properties vary must be zero. According to Gauss's theorem,
continuously within a very large range. For example, I. ~ ~ q
IT
the ratio of the electrical properties between a metal <It =j E. dS=-
EO
and glass may be as high as 1020.
As <It = 0, so q=0
2.15 BEHAVIOUR OF CONDUCTORS IN ->
E
ELECTROSTATIC FIELDS
+ + + +
H
25. State and prove the various electrostatic properties + + - - -- - - -: q +
shown by conductors placed in electrostatic fields. + / ", +
-> + I -> \ +
Electrostatic properties of a conductor. When E +-----1, E =0 '
placed in electrostatic fields, the conductors show the + \ I +
+ " ",,/ +
following properties :
O

+ +
1. Net electrostatic field is zero in the interior of a Gaussian
conductor. As shown in Fig. 2.40, when a conductor is surface
~
placed in an electric field Eext' its free electrons begin to
Fig. 2.41
~
move in the opposite direction of Eext' Negative
M

Hence there can be no charge in the interior of the


charges are induced on the left end and positive conductor because the Gaussian surface lies just near
the outer boundary. The entire excess charge q must
reside at the surface of the conductor.
..• +
Eind 4. Potential is constant within and on the surface
- +-+ of a conductor. Electric field at any point is equal to the
+ -> + negative of the potential gradient,
E =0 + . E= _ dV
t.e.,
dr
+ But inside a conductor E = 0 and moreover, E has
no tangential component on the surface, so
Conductor
dV =0 or V = constant
Fig. 2.40 Electric field inside a conductor is zero. dr
ELECTROSTATIC POTENTIAL AND CAPACITANCE 2.25

Hence electric potential is constant throughout the inside the cavity. Imagine a Gaussian surface inside the
volume of a conductor and has the same value (as conductor quite close to the cavity. Everywhere inside
inside) on its surface. Thus the surface of a conductor is an the conductor, E =0. By Gauss's theorem, charge
equipoteniial surface. enclosed by this Gaussian surface is zero (E = 0 =>
If a cond uctor is charged, there exists an electric q =0). Consequently, the electric field must be zero at
field normal to its surface. This indicates that the every point inside the cavity (q = 0 => E = 0). The entire
potential on the surface will be different from the excess charge + q lies on its surface.
potential at a point just outside the surface. E
5. Electric field at the surface of a charged conductor
is proportional to the surface charge densitq. Consider
a charged conductor of irregular shape. Let c be the

R
surface charge density at any point of its surface. To
+ +
E'---(
+ +
+ +

SI
E=O
+ +

Surface of
conductor E

Fig. 2.43 Electric field vanishes in the cavity


of a conductor.

Fig. 2.42 A small pill box as a Gaussian surface 2.16 ELECTROSTATIC SHIELDING
IT
of a charged conductor.
26. What is electrostatic shielding? Mention its few
determine E at this point, we choose a short cylinder applications,
(pill box) as the Gaussian surface about this point. The Electrostatic shielding. Consider a conductor with a
pill box lies partly inside and partly outside the con- cavity, with no charges placed inside the cavity.
ductor. It has a cross-sectional area L'lS and negligible Whatever be the size and shape of the cavity and
height.
H
whatever be the charge on the conductor and the
Electric field is zero inside the conductor and just external fields in which it might be placed, the electric
outside, it is normal to the surface. The contribution to field inside the cavity is zero, i.e., the cavity inside the
the total flux through the pill box comes only from its conductor remains shielded from outside electric
outer cross-section. influence. This is known as electrostatic shielding.
O

~ = E »s Such a field free region is called a Faraday cage.


Charge enclosed by pill box, q = c L'lS The phenomenon of making a region free from any
electricfield is called electrostatic shielding. It is based on
By Gauss's theorem,
the fact that electric field vanishes inside the cavity of a
~=l.. hollow conductor.
M

EO
Applications of electrostatic shielding
E L'lS = cr L'lS or
1. In a thunderstorm accompanied by lightning, it
EO
is safest to sit inside' a car, rather than near a tree
~
As E points normally outward, so we write or on the open ground. The metallic body of the
~ cr" car becomes an electrostatic shielding from
E =-n lightning.
EO
2. Sensitive components of electronic devices are pro-
where;; is a unit vector normal to the surface in the tected or shielded from external electric distur-
outer direction. bances by placing metal shields around them.
6. Electric field is zero in the' cavity of a hollow 3. In a coaxial cable, the outer conductor connec-
charged conductor. As shown in Fig: '2.43, consider a ted to ground provides an electrical shield to
charged conductor having a cavityi-with no charges the signals carried by the central conductor. •
2.26 PHYSICS-XII

For Your Knowledge 28. Define the unit of capacitance for a conductor.
Give its dimensions.
~ In the interior of a conductor, the electric field and the Units of capacitance. The 51 unit of capacitance is
volume charge density both vanish. Therefore, farad (F), named in the honour of Michael Faraday.
charges in a conductor can only be at the surface.
The capacitance of conductor is 1farad if the addition
~ Electric field at the surface of a charged conductor
of a charge ofl coulomb to it, increases its potential by 1volt.
must be normal to the surface at every point.
~ For a conductor without any surface charge, electric :. 1 farad = 1 coulomb or 1F = 1C = 1 cv '
field is zero even at the surface. 1 volt 1Y
~ The entire body of each conductor, including its One farad is a very large unit of capacitance. For
surface, is at a constant potential. practical purposes, we use its following submultiples:

R
~ If we have conductors of arbitrary size, shape and
charge configuration, then each conductor will have a 1 millifarad = 1 mF = 10-3 F
characteristic value of constant potential which may 1 microfarad = IIlF = 10-6F
differ from one conductor to another. 1 picofarad = 1 pF = 10-12F
~ A cavity inside a conductor is shielded from outside

SI
electrical disturbances. However, the electrostatic Dimensions of capacitance.The unit of capacitance is
shielding does not work the other way round. That is, _ 1 C _ 1 C _1 C2 _ 1(As)2
1 F ------------
if we place charges inside the cavity, the exterior of 1Y 1J / C 1J 1 Nm
the conductor cannot be shielded from the electric
fields of the inside charges. .. Dimensions of capacitance
2 2
A T =[~lL-2y4A2]
2.17 ELECTRICAL CAPACITANCE OF A MLr2.L
CONDUCTOR
2.18
IT
27. Define electrical capacitance of a conductor. On CAPACITANCE OF AN ISOLATED
which factors does it depend? SPHERICAL CAPACITOR
Electrical capacitance of a conductor. The electrical 29. Obtain an expression for the capacitance of an
capacitance of a conductor is the measure of its ability to hold isolated spherical conductor of radius R.
electric charge. When an insulated conductor is given Capacitance of an iso-
some charge, it acquires a certain potential. If we
lated spherical conductor.
H
increase the charge on a conductor, its potential also
Consider an isolated sphe-
increases. If a charge Q put on an insulated conductor
rical conductor of radius R.
increases its potential by V, then
The charge Q is uniformly
Qoc V or Q= CV distributed over its entire
O

The proportionality constant C is called the surface. It can be assumed


capacitance of the conductor. Thus to be concentrated at the
. Charge centre of the sphere. The
Capacitance = --"""- potential at any point on the
Potential Fig. 2.44 Capacitance of a
surface of the spherical
Hence the capacitance of a conductor may be defined spherical conductor.
M

as the charge required to increase the potential of the conductor conductor will be
by unit amount. V=_I_. Q
The capacitance of a conductor is the measure of its 41t EO R
capacity to hold a large amount of charge without running Capacitance of the spherical conductor situated
a high potential. It depends upon the following factors : in vacuum is
1. Size and shape of the conductor.
2. Nature (permittivity) of the surrounding medium.
3. Presence of the other conductors in its neigh-
bourhood.
or
It is worth-noting that the capacitance of a con-
ductor does not depend on the nature of its material Clearly, the capacitance of a spherical conductor is
and the amount of charge existing on the conductor. proportional to its radius.
ELECTROSTATIC POTENTIAL AND CAPACITANCE 2.27

Let us calculate the radius of the spherical Solution. Let rand R be the radii of the small and
conductor of capacitance 1 F. bigger drops, respectively.
Volume of the bigger drop
R = _1_. C =9 x 109 mF-1.1 F
4rc So = 27 x Volume of a small drop
6
= 9x 109m =9 x 10 km .
t.e., A3 rc R3 = 27 x A3 rcr3
This radius is about 1500 times the radius of the
or R =3r=3 x 3 mm=9 x 10-3 m
earth (-6 x 103 km). So we conclude:
1. One farad is a very large unit of capacitance. .'. Capacitance of the bigger drop is

2. It is not possible to have a single isolated C = 4rcso R = _1- .9 x 10-3 F


conductor of very large capacitance. 9 x 910

R
= 10-12 F = 1 pF
For Your Knowledge Charge on bigger drop
~ The formula: C = 41t EO R is valid for both hollow and q = 27 x Charge on a small drop
solid spherical conductors. =27x 10-12 C

SI
C
As E =-- .'. Potential of bigger drop is
o 41t R
q 27 x 10-12
So the 51 unit of EO can be written as farad per metre V= - = =27 V.
(Fm-1). From Coulomb's law, the 51 unit of EO comes C 10-12
out to be C2N-1m-2. Both of these units are equivalent. Example 34.Eight identical spherical drops, each carrying
~ The farad (1F = 1 Cy-l) is an enormously large unit a charge 1nC are at a potential of900 Veach. All these drops
of capacitance because the coulomb is a very big unit combine together to form a single large drop. Calculate the
of charge while the volt is the unit of potential having potential of this large drop. (Assume no wastage of any kind
IT
reasonable size. and take the capacitance of a sphere of radius r as
proportional to r). [eBSE Sample Paper 15]

Solution.
Capacitance of each small drop, C ex: r => C = kr
Formulae Used Charge on each small drop, q = CV = (krx 900)C
H
1. Capacitance of a spherical conductor of radius R, Charge on large drop, q =8q = 7200kr C
C =4rcso R Volume of a large drop = Volume of 8 small drops
. Charge ArcR3=8xArcr3 => R=81/3r=2r
2. Capacitance = -----'~ or 3 3
Potential Capacitance of large drop, C' = kR = 2 kr
O

Units Used Hence, the potential of the large drop is


Charge is in coulomb, potential in volt and V' =!L = 7200kr = 3600 V.
capacitance in farad (F). C' 2 kr
Example 3S.A charged spherical conductor has a surface
Example 32.An isolated sphere has a capacitance 50 pF.
charge density of 0.07 C em- 2. When the charge is increased
M

(i) Calculate its radius. (ii) How much charge should be


by 4.4 C, the surface charge density changes by
placed 011 it to raise its potential to 104 V ?
0.084 C em- 2. Find the initial charge and capacitance of the
Solution. Here C = 50 pF = 50 x 10-12 F, V = 104 V spherical conductor.
(i) R =_1_. C =9x 109mF-1 x SOx 10-12 F Solution. Let q be the charge on the spherical con-
4rcSo
ductor and r its radius. Its surface charge density is
= 45 x 1O-2m = 45 em. -q- = 0.07 C ern -2 (i)
47t? ...
q = CV = 50 x 10-12 x 104 = 5 x 10-7 C = 0.5 1lC.
(ii)
Example 33. Twenty seven spherical drops of radius 3 mm When the charge is increased by 4.4 C, the surface
12 charge density becomes
and carrying 10- C of charge are combined toform a single
drop. Find the capacitance and the potential of the bigger q + 4.4
2 = 0.084 C cm-
2
••• ( II
.. )
drop. [Haryana 01] 4rc r:
2.28 PHYSICS-XII

Dividing equation (ii) by (I), we get Potential of the big drop,


q + 4.4 0.084 C V'- 1 q' _ 1 Nq
--=--
q 0.07
or q=22 - 41t EO • R- 41t Eo . NI/3 r

From equation (i), we get = N2/3 . __ ~ . !1 = N2/3 V


47t Eo r
22x7
r- - =5 cm =0.05 m or
- 41t x 0.07 - 4 x 22 x 0.07
~
Capacitance,
2.19 CONCEPT OF A CAPACITOR AND
C = 41t EOr = __ 1-9 x 0.05 = 5.56 x 10-12 F.
9 x 10
ITS PRINCIPLE

R
30. An isolated conductor cannot have a large
j2)roblems ForPractice capacitance, why ?
The capacitance of an isolated conductor is small.
1. Find the capacitance of a conducting sphere of
radius 10 cm situated in air. How much charge is When a conductor holds a large amount of charge, its
required to raise it to a potential of 1000 volt? potential is also high. If the associated electric field

SI
(£ = a/Eo) becomes high enough, the atoms or
(Ans. 11 pP, 1.1 x 10-8 C)
molecules of the surrounding air get ionised. A
2. Assuming the earth to be a spherical conductor of breakdown occurs in the insulation of the surrounding
radius 6400 km, calculate its capacitance. medium and the charge put on the conductor gets
[Himachal 98C; Haryana 98C]
neutralised or leaks away. For air, the breakdown
(Ans.711IlF) point occurs at fields of the order of3 x 106 Vm -1. This
3. N drops of mercury of equal radii and possessing puts the limit on the capacitance of a conductor.
equal charges combine to form a big drop. Compare Moreover, if we tend to have a single conductor of
IT
the charge, capacitance and potential of bigger drop large capacitance, it will have practically inconvenient
with the corresponding quantities of individual drops. large size.
[Punjab 01]
31. Why does the capacitance of a conductor increase,
(Ans. N, N1/3, N2/3)
when an earthed connected conductor is placed near it ?
HINTS Briefly explain.
1. C = 41t EoR = _1-9 x 0.10 = 11x 10-12 F = 11 pF. Principle of a capacitor. Consider a positively
H
9xlO charged metal plate A and place an uncharged plate B
q = CV = 11 x 10-12p xl00G V = 1.1 x 10-8 C. close to it, as shown in Fig. 2.45. Due to induction, the
closer face of plate B acquires negative charge and its
1 6
farther face acquires a positive charge. The negative
2. C=41tEoR=--9 x6.4xl0
9 x 10 charge on plate Btends to reduce the potential on plate
O

= 0.711 x 1O-3p = 711 IlF. A, while the positive charge on plate B tends to
increase the potential on A As the negative charge of
3. Let q be the charge on each small drop and r its
plate B is closer to plate A than its positive charge, so
radius.
the net effect is that the potential of A decreases by a
Capacitance of each small drop, C = 41t EOr small amount and hence its capacitance increases by a
M

small amount.
Potential of each small drop, V = _1_ !1
41t Eo r A B A B
+ + - + + +
If R is the radius of the big drop, then + + - + + +
+ + - + + +
4 3 4 3 1/3 + + - + + +
- nr x N = - 1tR or R=N r + + - + + +
3 3 + + - + + +
+ + - + + +
-i- + - + + +
Charge on the big drop, q' = Nq or + + - + + +
+ + - + + +
+ + - + + +
Capacitance of the big drop. +"
+
+
+
-
-
+
+
+ +
+ + -
+ + - + + +
C' = 41t EO R = 41t Eo N1/3 r = NI/3 C + + - + + +
+ + - + + +
+ + - + + +
or e = N1/3.
C Fig. 2.45 Principle of a capacitor.
ELECTROSTATIC POTENTIAL AND CAPACITANCE 2.29

Now if the positive face of plate B is earthed, its The proportionality constant C is called the capa-
positive charge gets neutralised due to the flow of citance of the capacitor. Clearly,
electrons from the earth to the plate B. The negative
C=Q
charge on B is held in position due to the positive V
charge on A. The negative charge on B reduces the . Charge on either conductor
potential of A considerably and hence increases its or C apaatance= ---~---------
P.O. between the two conductors
capacitance by a large amount.
Hence we see that the capacitance of an insulated The capacitanceof a capacitormay be defined as the charge
conductor is considerably increased when we place an required to be supplied to either of the conductors of the
earthed connected conductor near it. Such a system of two capacitor so as to increase the potential difference between

R
conductors is called a capacitor.
them by unit amount.
The capacitance of a given capacitor is a constant
32. What is a capacitor ? Define capacitance of a
and depends on the geometric factors, such as the
capacitor. On what factors does it depend ?
shapes, sizes and relative positions of the two cond-
Capacitor. A capacitor is an arrangement of two uctors, and the nature of the medium between them.

SI
conductors separated by an insulating medium that is used
SI unit of capacitance is farad (F). A capacitor has a
to store electric charge and electric energy.
capacitance of 1 farad if 1 coulomb of charge is transferred
A capacitor, in general, consists of two conductors from its one conductor to another on applying a potential
of any size and shape carrying different potentials and difference of 1 volt across the two conductors.
charges, and placed closed together in some definite
positions relative to one another. 2.20 PARALLEL PLATE CAPACITOR
Pictorial representation of a capacitor. The pictorial 33. What is a parallel plate capacitor ? Drive an
symbol for a capacitor with fixed capacitance is as expression for its capacitance. On what factors does the
IT
shown in Fig. 2.46(a) and for that with a variable capa- capacitance of a parallel plate capacitor depend ?
citance is as shown in Fig. 2.46(b).
Parallel plate capacitor. The simplest and the most
--111--- widely used capacitor is the parallel plate capacitor. It
consists of two large plane parallel conducting plates,
(a) (b)
separated by a small distance.
Fig. 2.46 Symbols for a capacitor with Let A = area of each plate,
H
(a) fixed, (b) variable capacitance. d = distance between the two plates
Capacitance of a capacitor. As shown in Fig. 2.47, ± c = uniform surface charge densities on the
usually a capacitor consists of two conductors having two plates
charges + Q and - Q. The potential difference between ± Q=± oA = total charge on each plate.
O

I 1 1 ftl r=~~·
them is V = V+ - V_. Here Q is called the charge on the Area=A E=O
~I~~_~_~~~ ~~I
capacitor. Note that the charge on capacitor does not
mean the total charge given to the capacitor which is
+ Q-Q=O.
+ + + +

'-- --:::--:::-- ---'1 density


M

- cr
E=O

Fig. 2.48 Parallel plate capacitor.

In the outer regions above the upper plate and


below the lower plate, the electric fields due to the two
charged plates cancel out. The net field is zero.
E=~-~=O
Fig. 2.47 Two conductors separated by an
2Eo 2Eo
insulator form a capacitor.
In the inner region between the two capacitor
For"a given capacitor, the charge Q on the capacitor plates, the electric fields due to the two charged plates
is proportional to the potential difference V between add up. The net field is
the two conductors. Thus,
E=~+~=~
QocV or Q=CV 2Eo 2Eo EO
2.30 PHYSICS-XII

The direction of the electric field is from the The potential difference (caused by the inner
positive to the negative plate and the field is uniform sphere alone) between the two shells will be
throughout. For plates with finite area, the field lines b b b

bend at the edges. This effect is called fringing of the V=- f a


E.;t = fa
Edr = f ~4n
a EO'
dr
field. But for large plates separated by small distance
(A» d2), the field is almost uniform in the regions far
from the edges. For a uniform electric field, =, 4n
Q
EO ~
b
r
-2
dr = 4n
Q
EO
[l]b
-; a = 4n
Q
EO
[1 b1]
-;; -
P'D. between the plates
-t -t

= Electric field x distance between the plates [.: E points radially inward and dr points
-t -t
V = Ed = ad = Edr 180 = - Edr]

R
or outward so E . d r 0

EO
The capacitance of the spherical capacitor is
Capacitance of the parallel plate capacitor is
Q Q or 4n ab .
C= Q
V
=~
ad / EO
or C = EoA
d
C=V= Q [1 1] C= EO
b-a

SI
4n EO a b
Factors on which the capacitance of a parallel
plate capacitor depends
2.22 CYLINDRICAL CAPACITOR*
1. Area of the plates (C oc A).
35. What is a cylindrical capacitor ? Derive an
2. Distance between the plates (C oc 1 / d).
expression for its capacitance.
3. Permittivity of the medium between the plates
C} lind.ica capacitor. A cylindrical capacitor consists
(COCE).
of two coaxial conducting cylinders of inner and outer radii a
and b. Let the two cylinders have uniform linear charge
2.21 SPHERICAL CAPACITOR*
IT
densi ties of ± A. Cm -1. The length L of the capacitor is
34. What is a spherical capacitor ? Derive an so large (L» radii a or b) that the edge effect can be
expression for its capacitance. neglected. The electric field in the region between the
Spherical capacitor. A spherical capacitor consists of two cylinders comes only from the inner cylinder, the
two concentric spherical shells of inner and outer radii a and outer cylinder does not contribute due to shielding. To
b. The two shells carry charges - Q and + Q calculate the electric field E at any point P in between
H
respectively. Since the electric field inside a hollow the two cylinders at a distance r from the central axis,
-t . we consider a coaxial Gaussian cylinder of radius r.
conductor is zero, so E = 0 for r < a. Also the field is Using Gauss's theorem, the flux through Gaussian
-t
zero outside the outer shell, i.e., E =0 for r > b. A radial surface must be
~
O

field E exists in the region between the two shells due


to the charge on the inner shell only.
b a
To determine the electric field at any point P at dis-
tance r from the centre, consider a concentric sphere of
radius r as the Gaussian surface. Using Gauss's theorem,
M

4t = E.4n? = Q or E=~
+1-
Charge
EO 4n Eor density + A.
1

£=0 Charge+Q +1- Charge


1

+ +1 density - A.
1
1
+1 Gaussian
1
+1- cylinder
1
+ 1 --J..-I---L-

Charge-Q

Gaussian
surface
Fig. 2.49 Spherical capacitor. Fig. 2.50
ELECTROSTATIC POTENTIAL AND CAPACITANCE 2.31

or E .21tr L= ')...L Example 36. When 1.0 x 1012 electrons are transferred
eo from one conductor to another of a capacitor, a potential
E=_J..._ difference of 10 V develops between the two conductors.
Calculate the capacitance of the capacitor.
21t eo r
Solution. Here q = ne = 1.0 x 1012.x 1.6 x 10-19
:. Potential difference between the two cylinders is
=1.6 x 10-7 C
b-+-+ b
V =- f E. dr =f E dr [.: E and d -; are in V=lOV
a a opposite directions] 7
.. C= ..i = 1.6 x 10- = 1.6 x 10-8 F.
b J... J... b 1 V 10

R
= f --dr=-f -dr Example 37. A capacitor of unknown capacitance is
a 21t eo r 21t eo a r connected across a battery of V volts. The charge stored in it
J... b J... is 360 ~c.When potential across the capacitor is reduced by
=-[lnr] =-[lnb-lna]
21t eo a 2 1t eo 120 V, the charge stored in it becomes 120 ~c.Calculate:
(i) The potential y and the unknown capacitance C

SI
or V=~ln~
21t eo a (ii) What will be the charge stored in the capacitor, if
the voltage applied had increased by 120 V ?
Total charge on each cylinder is Q= LA [CBSE D 13]
.. Capacitance of cylindrical capacitor is Solution. (i) Let C be the capacitance of the capa-
C=Q= LA or C=21teoL citor and V the potential drop across the plates. Then
V _J..._ln~ ln~ q= CV=360~C
21t eo a a When the potential difference is reduced by 120 V,
IT
if
- -. .
Exam /es based on
- .•. .. . ... .. _V_=36O
V -120
= C(V -120)=120

120
=3
~C

V=180 V
Formulae Used
C= ..i = 360 ~C = 2 F.
1. Capacitance, C = 3.. V 180V ~
V
H
2. Capacitance of a parallel plate capacitor, C = BodA (it) When the voltage is increased by 120 V,
l' = C(V + 120) =2~Fx (180 + 120) = 600 ~C
3. P.D. between the two plates of a capacitor having
Example 38. A parallel plate capacitor has plate area of
charges % and q2' 25.0 art and a separation of2.0 mm between its plates. The
V = q1 - q2 capacitor is connected to 12 V battery. (j) Find the charge on
O

2C
the capacitor. (ii) If the plate separation is decreased by
4. Capacitance of a spherical capacitor, C = 41tSo ..!!!!..... 1.0 mm what extra charge is given by the battery to the
b-a
positive plate ?
Here a and b are the radii of inner and outer shells Solution. A =25.0 cm2 =25 x 10-4 m2,
of the spherical capacitor.
M

S. Capacitance of a cylindrical capacitor, d = 2.0 mm = 2 x 10- 3 m, V = 12 V


L L 12
C= eo A = 8.85 x 10- x 25 x 10-4 =1.1 x 10-11 F
C = 21tSo --b = 21tSo b
loge - 2303 log10 - d 2 x 10-3
a a
(i) q = CV = 1.1 x 10-11 x 12 = 1.32 x 10-10 C
Here a and b are the radii of inner and outer coaxial
cylinders and L is the length of the capacitor. (ii) Here d' = 2.0 -1.0 = 1.0 mm = 1 x 10-3 m
12
C' = 8.85 x 10- x 25 x 10-4 =2.2 x lO-11F
Units Used .. Ix 10-3
Capacitance C is in farad, charge q in coulomb,
potential difference V in volt, thicknesses d and t if = cv =2.2 x 10-11 x 12 =2.64 x 10-10 C
in metre. Extra charge given by the battery to the positive
plate is
Constant Used
q' - q = (2.64 -1.32)x 10-10 = 1.32 x 10-10 C.
Permittivity constant, EO = 8.85 x 10-12 C2N-1m-2
2.32 PHYSICS-XII

Example 39. Two parallel plate air capacitors have their Example 42. The negative plate of a parallel plate capacitor
plate areas 100 and 500 err?- respectively. If they have the is given a charge of - 20 x 10-8 C. Find the charges
same charge and potential and the distance between the appearing on the four surfaces of the capacitor plates.
plates of the first capacitor is 0.5 mm what is the distance Solution. As shown in Fig. 2.51, let the charge
between the plates of the second capacitor ? [Punjab 97C] appearing on the inner surface of the negative plate be
Solution. As capacitance, C = q / V and the two - Q. Then the charge on its outer surface will be
capacitors have the same charge q and potential V, so Q-20 x 10-8 C
they have the equal capacitances, i.e.,
1 2 3 4
C1 = C2

R
EO~= EO ~
or
d1. d2
• p
or d=~d
2 ~ 1

=500 crn2,

SI
But ~ =100 cm2,'~ -8
-Q +Q -Q Q - 20 x 10 C
d1 = 0.5 mm = 0.05 ~m

d - 500 x 0.05 -025 - 25 Fig. 2.51


.. 2 - 100 -. em - . nun.
The induced charge on the inner surface of the
Example 40. A sphere of radius 0.03 mis suspended within positive plate will be + Q and that on the outer surface
a hollow sphere of radius 0.05 m. If the inner sphere is will be - Q, as the positive plate is electrically neutral.
charged to a potential of 1500 volt and outer sphere is earthed, To find Q, we consider the electric field at a point P
find the capacitance and the charge on the inner sphere. inside the negative plate.
IT
Solution. Here a = 0.03 m, b = 0.05 m, V = 1500 V
Field due to surface 1= ~, towards left
The capacitance of the air-filled spherical capacitor is 2Eo A
41t EO ab 0.03 x 0.05
C=--"-- Field due to surface 2 = ~, towards right
b- a 9 x 109 x (0.05 -0.03) 2Eo A
= 8.33 x .10-12 F = 8.33 pF.
H
Field due to surface 3 = ~, towards left
Charge, q= CV =;8.33 x 10-12 x 1500 2Eo A
= 1.25 x 10-8 C. . 20x 10-8C
Field due to surface 4 = , towards left
Example 41. The thickness of air layer between the two 2Eo A
coatings of a spherical capacitor is 2 em The capacitor has
O

As the point P lies inside the conductor, the field


the same capacitance as the sphere ofl.2 m diameter. Find here must be zero.
the radii 0/ its surfaces. 8
41t E ab ~_~+~+ Q-20x10- =0
Solution. Here = 41t EO R
0
b-a ", ~ .. 2Eo A 2Eo A 2Eo A 2EoA
or 2Q-20x10-8 =0'
M

·or ~=R
b-a Q = + 10 x 1O-8C
Now b - a =2 cm and R =.!2 m =60 ern :. Charge on surface 1 = -10 x 10-8 C
2
ab =60 Charge on surface 2 = + 10 x 10-8 C
2 Charge on surface 3 = =: 10 x 10-8 C
or ab·=!120 Charge on surface 4 = - 10 x 10-8 C.
(b + a)2 = (b - a)2 + 4ab
= 22 + 4 x 120 = 484 problems For Practice
or b + a =22 1. A capacitor of 20 J.1Fis charged to a potentialof
or 2+a+a=22 [':b-a=2crn] 10 kV. Find the charge accumulated on each plate
a = 10 em and b = 12 em. of the capacitor. (Ans, 0.2 C)
ELECTROSTATIC POTENTIAL AND CAPACITANCE 2.33

2. Calculate the capacitance of a parallel plate capa-


4.
c- EoA _ 8.85 x 10-12 x 200 x 10-4
citor having circular discs of radii 5.0 cm each. The - d - Lx 10-3

separation between the discs is 1.0 mm.


=0.177 x 10-9 F = 0.177 nF
(Ans. 0.69 x 10- 10 F)
. 1nC q
3. A parallel plate air capacitor consists of two circular (I) V = - = = 5.65 V.
C 0.177 nF
plates of diameter 8 cm. At what distance should
the plates be held so as to have the same capa- (ii) When the plate separation increases from
citance as that of a sphere of diameter 20 cm ? 1.0 mm to 2.0 mm, the capacitance decreases by
(Ans. 4 mm) a factor of 2. For the same charge, the potential
difference will increase by a factor of 2.
4. A parallel-plate capacitor has plates of area 200 em2

R
and separation between the plates 1.0 mm. (i) What :. V' = 2 V = 2 x 5.65 =11.3 V.
potential difference will be developed if a charge of 5. Charge on the capacitor,
1.0 nC is given to the capacitor? (ii) If the plate q = 70 pC = 70 x 10-12 C
separation is now increased to 2.0 mm, what will be 12
the new potential difference? (Ans. 5.65 V, 11.3 V) C = .i = 70 x 10- C = 3.5 pF.

SI
V 20V
5. Two metallic conductors have net charges of
+ 70 pC and - 70 pC, which result in a potential 6. Here a = 9em = 0.09 m, b = 10cm = 0.10 m
difference of 20 V between them. What is the C = 41tEoab= _1_. 0.09 x 0.10 F
capacitance of the system? (Ans. 3.5 pF) b- a 9 x 109 (0.10 - 0.09)
6. A spherical capacitor has an inner sphere of radius
= 0.01xO.10xlO-9F = 0.lx10-9F = 0.1 nF.
9 em and an outer phere of radius 10em. The outer 0.01
sphere is earthed and the inner sphere is charged.
7. Here
What is the capacitance of the capacitor?
IT
a = radius of the earth = 6.4 x 106 m
(Ans. 0.1 nF)
b = distance of the stratosphere layer from the
7. The stratosphere acts as a conducting layer for the centre of the earth
earth. If the stratosphere extends beyond 50 krn
= 6400 + 50 = 6450 km = 6.45 x 106 m
from the surface of the earth, then calculate the
capacitance of the spherical capacitor formed ab 1 6.4 x 106 x 6.45 x 106
between stratosphere and earth's surface. Take radius C = 4n Eo a _ b = 9 x 109 x (6.45 _ 6.4) x 106
H
of the earth as 6400 krn. (An . 0.092 F) = 0.092 F.
8. A charge of + 2.0 x 10-8 C is placed on the positive 8 8
8. V = ~ - q2 = 20 x 10- + 1.0 x 10- = 12.5 V.
plate and a charge of -1.0 x 10- C on the negative 2C 2 x 1.2 x 10 9
plate of a parallel plate capacitor of capacitance
O

1.2 x 10-3 ~F. Calculate the potential difference


developed between the plates. (Ans. 12.5 V) 2.23 COMBINATION OF CAPACITORS IN
SERIES AND IN PARALLEL
HINTS
36. A number of capacitors are connected in series.
1. C = 20 ~F = 20 xlO-6F, V = 104 V
= 10 kV
Derive an expression for the equivalent capacitance of the
M

Charge, q = CV = 20 x 10-6 x 104C = 0.2 C series combination.


2. Here r = 5.0em = 0.05 m, d = 1.0 mm = 10- 3 m Capacitors in series. When the negative plate of one
Capacitance, capacitor is connected to the positive plate of the second, and
the negative of the second to the positive of third and so on,
EoA eonr2 the capacitors are said to be connected in series.
C=-=--
d d Figure 252 shows three capacitors of capacitances
n x (0.05)2 -10 . C1, C2 and C3 connected in series. A potential
= 9 3 = 0.69 x 10 F.
4n x 9 x 10 x 10 difference V is applied across the combination. This
sets up charges ± Q on the two plates of each capacitor.
eA Eonri
3. _0_ = 4nl'.~R or --- =4ne R What actually happens is, a charge + Q is given to the
d -u 4d 0
left plate of capacitor C1 during the charging process.

or d = .s:
16 R
= (0.08)2
16 x 0.10
= 4 x 10- 3 m = 4 mm. The charge + Q induces a charge - Q on the right plate
of C] and a charge - Q on the left plate of C2, etc.
2.34 PHYSICS-XII

+Q -Q +Q -Q +Q -Q
37. A number of capacitors are connected in parallel.
Derive an expression for the equivalent capacitance of the

: =H: =H:
+ -
+ -
+
+
-
-
+
+
parallel combination.
Capacitors in parallel. When the positive plates of all
capacitors are connected to one common point and the
negative plates to another common point, the capacitors are
CI Cz C3 said to be connected in parallel.
~ VI ~I~ V2 ~I~ V3 --+I Figure 2.53 shows three capacitors of capacitances
Cl' C2 and C3 connected in parallel. A potential
L..--------o V o-------~
difference V is applied across the combination. All the

R
(+) H
capacitors have a common potential difference V but
Fig. 2.52 Capacitors in series. different charges given by
Ql = c, V, Q2 = C2V, Q3 = C3V
The potential differences across the various
capacitors are

SI
Q Q Q
VI =-, V2 =-, 3 V =-
CI C2 C3

For the series circuit, the sum of these potential diffe-


rences must be equal to the applied potential difference.

V=V +V +V =Q+Q+Q
1 2 3 C C C
l 2 3
IT
VII 1
or -=-+-+- ...(1)
QCl C2 C3
+ -
Clearly, the combination can be regarded as an
effective capacitor with charge Q and potential dif- V
ference V. If Cs is the equivalent capacitance of the (+) (-)
series combination, then
H
Fig. 2.53 Capacitors in parallel.
C=Q
s V
Total charge stored in the combination is
1 V Q = Q1 + Q2 + Q3 =(C1 + C2o+C3) V ...(1)
or ...(2)
Cs Q If C is the equivalent capacitance of the parallel
O

r,
com bimation, th en
From equations (1) and (2), we get °

111 1 Q = Cp V ...(2)
-=-+-+-
c, C1 C2 C3 From equations (1) and (2), we get
CpV":(C1 +C2+C3)V
For a series combination of n capacitors, we can
M

write or Cp = Cl + C2 + C3
111 1 For a parallel combination of n capacitors, we can write
-=-+-+ ..... +-
c, Cl C2 c, Cp = Cl -+ C2 + .....+ Cn
For series combination of capacitors For parallel combination of capacitors
1. The reciprocal of equivalent capacitance is equal 1. The equivalent capacitance is equal to the sum
to the sum of the reciprocals of the individual of the individual capacitances.
capacitances. 2. The equivalent capacitance is larger than the
2. The equivalent capacitance is smaller than the largest individual capacitance.
smallest individual capacitance. 3. The potential difference across each capacitor is
3. The charge on each capacitors is same. same.
4. The potential difference across any capacitor is 4. The charge on each capacitor is proportional to
inversely proportional to its capacitance. its capacitance.
ELECTROSTATIC POTENTIAL AND CAPACITANCE 2.35

Solution. Let the two capacitances be CI !IF and C2 !IF.


iliiiiiiiiiiiiiiiiiii~LE~xampleS_b_a.s__e_d_O_n -----' In parallel, Cp = CI + C2 = 5 !IF
Formulae Used CC
In series, Cs = 1 2 = 1.2 !IF
1 1 1 1 CI + C2
1. In series combination, -=-+-+-+ ...
Cs c,. CZ C;
CI(5 - CI) = 1.2
or
2. In parallel combination, Cp = c,. + CZ + C; + ... 5
2
3. In series combination, charge on each capacitor is or CI - 5CI + 6 = 0
same (equal to the charge supplied by battery) but
Hence, CI = 2 or 3!lF
potential differences across the capacitors may be

R
different. :. The capacitances are of 2!lF and 31lF.
4. In parallel combination, potential difference on Example 46. Three capacitors of equal capacitance, when
each capacitor is same but the charges on the capa- connected in series have net capacitance CI, and when
citors may be different. connected in parallel hque net capacitance C2. What is the

SI
Units Used value of CI / C2 ?

Capacitances are in farad, potential differences in Solution. Let C = capacitance of each capacitor.
volt and charges in coulomb. For series combination,
1 1 1 1 3
Example 43. Two capacitors of capacitance of 6 !IF and -=-+-+-=- or
CI C C C C
12 !IF are connected in series with a battery. The voltage
across the 6 !IF capacitor is 2 V. Compute the total battery For parallel combination,
voltage. [CBSE 00 06] CI_C 1_1
C2 = C + C + C =3C
IT
Solution. As the two capacitors are connected in C -3"'3C-9
2
series, the charge on each capacitor must be same.
Example 47. In Fig. 2.54, each of the uncharged capacitors
Charge on 6!lF = Charge on 12 !IF has a capacitance of25 !IF. What charge will flow through

or
capacitor capacitor

6 !IF x 2 volt = 12 !IF x V volt


r~
the meter M when the switch 5 is closed ?
H
6x2
:. PD. across 12 !IF capacitor = -- = 1volt
12 4200 V
Battery voltage = VI + V2 =2 V + 1 V = 3 V.
Example 44. Two capacitors of capacitances 3!lF and
l~_I J Ie
O

6 !IF, are charged to potentials of2 V and 5 V respectively. Fig. 2.54


These two charged capacitors are connected in series. Find
Solution. As the three capacitors are connected in
the potential across each of the two capacitors now.
parallel, their equivalent capacitance is
[CBSE Sample Paper 04]
Cp = C + C + C =3C =3 x 25!lF =75!lF
Solution. Total charge on the two capacitors
M

V= 4200 V
=CIVI + C2V2 =(3x2 +6x 5)!lC=36!lC
Charge, q = Cp V = 75 x 10-6 x 4200
In series combination, charge is conserved.
= 315 x 10- 3 C = 315 me
.. Charge on either capacitor,
Example 48. Calculate the charge supplied by the battery
q = 36!lC in the arrangement shown in Fig. 2.55.
Potential on 3 !IF capacitor =!L = 36 !lC = 12 V
CI 3!lF

Potential on 6 !IF capacitor =!L = 36 !lC = 6 V.


C2 6!lF

Example 45. Two capacitors have a capacitance of 5!lF


10V
when connected in parallel and 1.2 !IF when connected in
series. Calculate their capacitances. Fig. 2.55
2.36 PHYSICS-XII

Solution. The given arrangement is equivalent to Clearly, C2 and C3 are in parallel. Their equivalent
the arrangement shown in Fig. 2.56. capacitance is
c1 = 5 J.1f C' = C2 + C3 = 5 + 5 = 10 ~F
II Now C1 and C' form a series combination, as
+11 -
shown in Fig. 2.58(b). Their equivalent capacitance is
II C = C1 C = 10 x 10 = 5 ~F
+11 C1 + C 10 + 10
Cz = 6 J.1f
Charge drawn from the battery,
+ I - q= CV = 5 ~F x 6 V =30 ~C
I

R
10V Charge on the capacitor C1 = q = 30 ~C
Charge on the parallel combination of C2 and
Fig. 2.56
C3 =q=30 ~C
Clearly, the two capacitors are connected in parallel. As C2 and C3 are equal, so the charge is shared

SI
Their equivalent capacitance is equally by the two capacitors.
30
C = C1 + C2 = 5 + 6 = 11 ~F Charge on C2 = charge on C3 = - = 15 ~C
2
Charge supplied by the battery is
q= CV = 11 ~F x 10 V = 110 J..Ic.
Example 50. Find the equivalent capacitance of the
combination of capacitors between the points A and B as
Example 49. Three capacitors C1, C2 and C3 are connected shown in Fig. 2.59. Also calculate the total charge that flows
to a 6 V battery, as shown in Fig. 2.57. Find the charges on in the circuit when a 100 V battery is connected between the
the three capacitors. points A and B. [CBSE D 02]
IT
40 J.1f 60 J.1f
A o-------i ~

I I ~J.1f
H
lOp!' ~ 6Op!'

Fig. 2.57
L---~----------~---oB
Solution. The given arrangement is equivalent to Fig. 2.59
the arrangement shown in Fig. 2.58(a).
Solution. Here three capacitors of 60 ~F each are
O

connected in series. Their equivalent capacitance C1 is


given by
1 1 1 1 3 1
-=-+-+-=-=-
• C1 60 60 60 60 20
M

or C =20 ~F
6V The given arrangement now reduces to the
+ - equivalent circuit shown in Fig. 2.60(a)
(a) 40 J.1f
C1 = 10 IlF C' = 10 IlF

(,)Ao-i::-Lo ,I 1,0,'
C:J 6V
(b) (b)
A
T T T 40 IlF
OI----IIf---IIf---O
40 IlF
B

Fig. 2.58 Fig. 2.60


ELECTROSTATIC
POTENTIALAND CAPACITANCE 2.37

Clearly, the three capacitors of IOIlF, IOIlF and


20 IlF are in parallel. Their equivalent capacitance is
C2 = 10 + 10 + 20 = 40 IlF
Now the circuit reduces to the equivalent circuit
shown in Fig. 2.60(b). We have two capacitors of 40 IlF
each connected in series. The equivalent capacitance
between A and B is
40x 40
C= = 20 IlF. L-------~-----------+----~B
40 + 40
Fig. 2.62
Given V =100 V

R
.'. Charge, q = CV =20 ~F x 100 V Solution. Capacitors C2 and C3 form a parallel com-
= 2000 ~C = 2 mC bination of equivalent capacitance,

Example 51. If C1 =3 pF and C; =2


pF, calculate the Cs = C2 + C3 = 2 + 2 = 4 ~F
equivalent capacitance of the given network between points Capacitors C4 and Cs form a series combination of

SI
A and B. capacitance C9 given by
1111131
--= -- +-- =-+ - =- =-
C9 C4 c, 12 6 12 4

C9 = 41lF
The equivalent circuit can be shown as in Fig. 2.63(a)
IT
Fig. 2.61

Solution. Clearly, capacitors 2, 3 and 4 form a


series combination. Their total capacitance C' is
H
given by
11111117
-=-+-+-=-+-+-=- Fig. 2.63 (a)
C' C1 C2 C1 3 2 3 6
Capacitors C1 and Cs form a series combination of
C' =~ pF
7 capacitance CIO given by
O

The capacitance C' forms a parallel combination _ C1Cs _ 8 x 4 _ 32 _ ~ F


C10 - - - - ~
with capacitor 5, so their equivalent capacitance is C1 + Cs 8 +4 12. 3

C" = C' + C2=-+2=-p


6 20 F Capacitors C6 and C9 form a parallel combination
7 7 of capacitance.
M

The capacitance C" forms a series combination with Cn = C6 + C9 = 4 + 4 = 8 ~F


capacitors 1 and 6. The equivalent capacitance C of the The given network reduces to the equivalent circuit
entire network is given by Fig. 2.63(b).
1 1 1 1 7 1 1 61
-=-+-+-=-+-+-=-
C C" C1 C1 20 3 3 60

C= 60 F.
61 P

Example 52. From the network shown in Fig. 2.62, find


the value of the capacitance C if the equivalent capacitance
between points A and B is to be 1 ~F. All the capacitances are
in ~F.
Fig. 2.63 (b)
2.38 PHYSICS-XII

Again, capacitors C7 and Cn form a series combi- Solution. Suppose a parallel combination of n
nation of capacitance C12 given by capacitors is connected in series with a series
combination of (7 - n) capacitors.
_ C7 x Cn _1 x 8 _ 8 \IF
C12 - - -- r
C7 + Cn 1 + 8 9 Capacitance of parallel combination, C1 =2n JlF
2
Now ClO and C12 form a parallel combination of Capacitance of series combination, C2 = -- JlF
7-n
capacitance C13 as shown in Fig. 2.63(c).
As these two combinations are in series, so
8 8 32
C13 = ClO + C12 = 3" + 9' = 9 JlF C
5
= 10 F
11 Jl
C
1 1 1 11 1 7-n
But -=-+- -=-+--

R
A~~ ~ ~ ~ 10 2n 2

clOLL~3 C12 9
~
Multiplying both sides by 10 n, we get
11n = 5 + 3Sn - 5n2
B or Sn2 -24n-S =0

SI
Fig. 2.63 (c) (n - S)(5n + 1) = 0
Finally, the capacitors C and C13 form a series com- or n=5 [Rejecting -ve value]
bination of capacitance 1 JlF as shown in Fig. 2.63(d). Hence parallel combination of 5 capacitors must be
C C13 connected in series with the other 2 capacitors.
A~~~B Examp e 55 Find the equivalent capacitance between the
32 points P and Q as shown in Fig. 2.64. Given C = 18 JlF and
9 C1 = 12 JlF. REC 97]
IT
Fig. 2.63 (d)

1 1 9 32

:&lliDc
-=-+- or C=-JlF.
1 C 32 23
Example 53. Connect three capacitors of3JlF,3 JlF
and 6 JlF such that their equivalent capacitance is 5 JlF.
H
Solution. Capacitors connected in parallel have c E C B C
maximum equivalent capacitance. Fig 2.64
Cmax = 3 + 3 + 6 = 12 JlF
Equivalent capacitance between points F and B is
Capacitors connected in series have minimum
18 x 18 + 18 = 27 F
O

equivalent capacitance. 18 + 18 Jl
1 1 1 1 5 Equivalent capacitance between points A and B is
--=-+-+-=-
Cmin 3 3 6 6 18 x 27
12 + -- = 12 + 10.8 =22.8 "'-23 JlF
6 18 +27
or Cmin = - = 1.2 JlF
M

5 Equivalent capacitance between points A and E is


The required equivalent capacitance of 5 JlF lies 23 x 18 + 18 = 28 F
23 + 18 Jl
between Cmax and·Cmin. So
3x6 Equivalent capacitance between points 0 and E is
5 JlF = 3 JlF + 2 JlF = 3 JlF + - JlF
3+6 28 x 18 + 12 = 23 F
28 + 18 Jl
So we should connect the series combination of3 JlF
and 6 JlF capacitors in parallel with the third capacitor Equivalent capacitance between points 0 and Q is
of 3 JlF. 23 x 18 + 18 = 28 F
23 + 18 Jl
Example 54.Seven capacitors, each of capacitance 2 JlFare
to be connected in a configuration to obtain an effective Equivalent capacitance between points P and Q is
capacitance of 10 / 11 JlF. Suggest a suitable combination to 28 x 18 = 11 F.
achieve the desired result. [lIT 90] 28 + 18 Jl
ELECTROSTATIC POTENTIAL AND CAPACITANCE 2.39

Example 56. Four capacitors are connected as shown in Solution. The given circuit can be redrawn in the
the Fig. 2.65. Calculate the equivalent capacitance between form of a wheatstone bridge as shown in Fig. 2.69.
the points X and Y. [CBSE D 2000]

c'4~
B

1 ,1!~~hj;i~Y
xe

Fig. 2.65
A~~rc
C
Solution. Clearly, the first plate of 2 IlF capacitor,

R
Fig. 2.69
the second plate of 3 IlF capacitor and the first plate of
5 IlF capacitor are connected to the point A On the As C1 = C2 = C4 = Cs '
other hand, the second plate of 2 IlF capacitor, the first
Therefore, -.l
C = --.!
C .
plate of 3 IlF capacitor and the second plate of 5 IlF
capacitor are connected to the point B.Thus the capa- C2 c,

SI
citors of 2 IlF, 3 IlF and 5 IlF are connected in parallel Thus the given circuit is a balanced wheatstone
between points A and B, as shown in the equivalent bridge. So the potential difference across the ends of
circuit diagram of Fig. 2.66. capacitor C3 is zero. Capacitance C3 is ineffective. The
21lF given circuit reduces to the equivalent circuit shown in
Fig. 2.70(a).

Xo---A+----II----+-B---I~y
IT
10 JlF

Fig. 2.66

Total capacitance of the parallel combination of


capacitances 21lF, 3 IlF and 5 IlF is Fig. 2.70 (a)
H
C=2+3+5=IOIlF
As shown in Fig. 2.67, this parallel combination is Capacitors C1 and C2 form a series combination of
in series with capacitance of 10 1lF. equivalent capacitance C6 given by

x o>-----II--~II--Oy C = C1 x C2 = 10 x 10 = 5 IlF
O

6 C1 + C2 10 + 10

Fig. 2.67 Similarly, C4 and Cs form a series combination of


equivalent capacitance C7 given by
Equivalent capacitance between X and Y
= 10 x 10 =5 F. C = C4 x Cs = 10 x 10 = 5 IlF
10 + 10 C4 + Cs 10 + 10
M

Il 7

Example 57 .Five capacitors of capacitance 10 IlF each are As shown in Fig. 2.70(b), C6 and C7 form a parallel
connected with each other, as shown in Fig. 2.68. Calculate combination. Hence the equivalent capacitance of the
the total capacitance between the points A and C. network is given by
C4 C = C6 + C7 = 5 + 5 = 10 IlF.
C6

A$~~c A~~C

Fig. 2.68 Fig. 2.70 (b)


2.40 PHYSICS-XII

Example 58. There are infinite number of capacitors, each Solution. Let C be the equivalent capacitance of the
of capacitance I/-1F. Thelj are connected in rows, such that infinite network. It consists of repeating units of two
the number of capacitors in the first row, second row, third capacitors of I/-1F and 2 /-IF.The addition of one such
row, fourth row, are respectively 1, 2, 4, 8, ..... The rows of more unit will not affect the equivalent capacitance.
these capacitors are then connected between points A and B, But then the network would appear as shown in
as shown in Fig. 2.71. Determine the equivalent capacitance Fig. 2.73.
of the network between the points A and B.
1 IlF

R
Fig. 2.73

The equivalent capacitance of the new arrangement

SI
must be equal to C.
2xC
A B
C=I+
2+C
Fig. 2.71
or C2 - C -2 =0
Solution. Let Cl, C2, Cy C4, ..... be the effective or C = 2 /-IF or - 1 /-IF
capacitances of the capacitors of first row, second row, As the capacitance cannot be negative, so the
third row, fourth row, ..... respectively. Then equivalent capacitance of the ladder is 2/-1F.
IT
Cl = IIJF Example 60. If Cl =20 /-IF, C2 =30 /-IF and C3 =15 /-IF
1xII and the insulated plate of Cl be at a potential of90 V, one
C2 = -- =- /-IF
1+ 1 2 plate of C3 being earthed. What is the potential difference
11111 between the plates of C2' three capacitors being connected in
-=-+-+-+-=4 series? [CBSE OD 15]
C3 1 1 1 1
H
1 Solution. Here Cl = 20/-lF, C2 =30 ~lF, C3 = 15 /-IF,
C3 = 4" /-IF V=90V

Similarly, C4 = .!. /-IF,and so on.


8
As these rows are connected in parallel between
O

points A and B, so the equivalent capacitance between


points A and B is
1 1 1 Fig. 2.74
C = Cl + C2 + C3 + C4 + ..... =1 + - + - + - + .....
2 4 8 The equivalent capacitance C of the series com-
This is an infinite geometric progression with first bination is given by
M

term a = 1 and common ratio r = 1 /2. Hence 11111113


a 1 -=-+-+-=-+-+-=-
C=-=--=2/-1F. C Cl C2 C3 20 30 15 20
l-r 1-1/2
C = 20 /-IF
Example 59. Find the equivalent capacitor of the ladder 3
(Fig. 2.72) between points A and B.
Total potential difference =90 -0 =90 V
21lF 21lF
:. Total charge,

A~T,~T,~~-l"~ q= CV = 20 x 10-6.90 =600 x 10-


3
6 C

Bo
Fig. 2.72
T T T Ihh_
PD. between the plates of capacitor C2 is
V
2
=..!L
C2
6
= 600 x 10- C = 20 V.
30 x 10-6 F
ELECTROSTATIC POTENTIAL AND CAPACITANCE 2.41

Example 61. In the circuit shown in Fig. 2.75, if the point' Solution. (a) In the given network, Cl' C2 and C3 are
C is earthed and point A is given a potential of + 1200 V, find' connected in series. Their equivalent capacitance C is I

the charge on each capacitor and the potential at the point B. given by
C2 11111113
-==-+-+-==-+-+-=-

12000V
A
cb-[J
I
3~F B
4~F
3
v=O
c
or
C' Cl C2 C3 10 10 10 10

C'== 10 J.lF
3
Now C' and C4 form a parallel combination. There-
2 ~F fore, the equivalent capacitance of the whole network is
Fig. 2.75 10 40

R
I
C= C + C4 ==-+ 10 =-J.lF =13.3 J.lF.
3 3
Solution. Capacitors C; and C3 form a parallel com-
(b) It is clear from Fig. 2.76 that the charge on each
bination. Their equivalent capacitance is
of the capacitors Cl, C2 and C3 is same. Let it be q. Let
C' = C2 + C3 = (4 + 2) J.lF= 6 J.lF the charge on C4 be q'.

SI
Now Cl and C' form a series combination, there-
•. P.D. across AB,
fore, the equivalent capacitance of the entire network is
CC' 3x 6
C = C + C' = 3 + 6 = 2 J.lF P.D. across BC,

The charge on the equivalent capacitor is


P.D. across CD,
q = CV =2 x 10-6 x 1200 C =2.4 x 10-3 C
This must be equal to the charge on C, and also the But
sum of the charges on C2 and C3• Thus
IT
V _ v: = -.:L = 2.4 x 10-3 =800 V
A B Cl 3 x 10-6

VA = 1200 V or
.. VB = 1200 -800 = 400 V
H
Hence Ve - VB = 400 -0 = 400 V or
q2 = C2 (Ve - VB) = 4 x 10-6 x 400 C ==1.6 x 10-3 C
q3 ==C3 (Ve - VB) =2 x 10-6 x 400 C = 0.8 x 10-3C
ql ==q = 2.4 x 10-3C. ==1.7x 10-3 C
O

Example 62. A network of four 10 J.lF capacitors is Also, P.D. across AD='£ =500 V
connected to a 500 V supply as shown in Fig. 2.76. Deter- C4
mine (a) the equivalent capacitance of the network, (b) the
•. q'=500xC4==500xl0J.lC
charge on each capacitor. [NCERT]
= 5000 x 10-6 C ==5 x 10-3 C.
+q -q
M

Example 63. Four capacitors Cl' C2' C3 and C4 are con-


Brlh c nected to a battery of12 V, as shown in Fig. 2.77. Find the

_q .L. + c,-" ~+q


potential difference between the points A and B.
CI = 8 ~F C2 = 4 ~F

+q ++++
CI C3
-----q I 1 I
+q' _q'
+
A
+ D
+
+
C4
+ -
SOOV 12 V
Fig. 2.76 Fig. 2.77
2.42 PHYSICS-XII

Solution. Let VA be the potential at point A and VB So charges on various plates are
that at B. Then _+ EO AV __ 2 EO AV
ql - d' q2 - d
P.D. across C1 = 12 - VA
P.D. across C2 = VA -0 = VA _ 2 EO AV _ 2 EO AV _ EO AV
q3 - + d ' q4 - - d ' qs - + -d- .
P.D. across C3 = 12 - VB
P.D. across C4 = VB -0 = VB Example 65. For the network shown in Fig. 2.80, find the
As the capacitors C1 and C2 are connected in series, so potential difference between points A and B, and that bet-
ween Band C in the steady state.
ql = q2 3flF IJlF
or C1 (12 - VA) = C2VA

R
or 8 (12 - VA) = 4 VA
or VA =8 V 3~~

Again, the capacitors C3 and C4 are connected in


series, so

SI
q3 = q4 Ion
or C3 (12 - VB) = C4 VB 20n
A c>----'V\J'\r--+-:-I1111 r---<lC
or 3 (12 - VB) = 6 VB
100V
or VB = 4V
The potential difference between the points A and Bis Fig. 2 80

VA - VB = 8 - 4 = 4 V. Solution. The two capacitors of 3 flF and 3 flF on the


Example 64. Five identical capacitor plates, each of area A left side of the network are in parallel, their equivalent
IT
are arranged such that the adjacent plates are at distance d capacitance = 6 flF
apart. The plates are connected to a source of emf V, as The two capacitors of 1flF and 1flF on the other side
shown in Fig. 2.78. Find the charges on the various plates. of the network are also in parallel, their equivalent
[lIT 84] capacitance = 2 flF. So the given network reduces to the
equivalent circuit shown in Fig. 2.81.
H
~
6 flF 2 flF

1 2 3 4 5 ..;;;;..v
+
O

Ion
A . 20 n 100V
A<r--'vvv--+-:-11111 _
C
Fig. 2.78
Fig. 2.81
Solution. As shown in Fig. 2.79, the given network
M

In the steady state, when all the capacitors are


is equivalent to three parallel-plate capacitors con-
charged, there is no current in the circuit. So there is no
nected in parallel.
potential drop across any resistance. Hence
Their capacitances are p.d. across 1uf capacitor
A
EO A 2 EO A and EO A
= p.d. between points A and C = 100 V
-d-' d d As 6 flF and 2 flF capacitances are in series, the p.d.
of 100 V is divided between them in the inverse ratio of
The p.d. across each V their capacitances i.e., in the ratio 2 : 6 or 1 : 3.
capacitor is V. + - 1
VAB = p.d. across 6 flF = 4" x 100 = 2S V
As Fig. 2.79
Charge = Capacitance x p.d. VBC = p.d. across 2 flF= 4"3 x 100 = 7S V.
ELECTROSTATIC POTENTIAL AND CAPACITANCE 2.43

8. In the combination of four identical capacitors shown


~rOblems For Practice in Fig. 2.84, the equivalent capacitance between
1. Two capacitors have a capacitance of 51lF when points P and Q is 1 IlF. Find the value of each
connected in parallel and 1.2IlFwhen connected in separate capacitance. (Ans. 41lF)
series. Calculate their capacitances.
(Ans. 21lF, 31lF)
2. Two capacitors of equal capacitance when
connected in series have net capacitance C;, and
when connected in parallel have net capacitance c;..
What is the value of C; / c;. ? [CBSE D 93CI

R
Fig. 2.84
(Ans. C; / c;. = 1/4)
3. Three capacitors of capacity 1, 2 and 31lF are 9. Find the equivalent capacitance of the combination
connected such that second and third are in series shown in Fig. 2.85 between the points A and B.
and the first one in parallel. Calculate the resultant
capacity. (Ans. 2.2 IlF) L l (Ans.c;+~J

SI
4. The capacities of three capacitors are in the ratio A~~B ~+~
1 : 2 : 3. Their equivalent capacity in parallel is
greater than the equivalent capacity in series by
60/11 pF. Calculate the individual capacitances. C1-r= C2=r-
(Ans. 1 pF, 2 pF, 3 pF) Fig. 2.85
5. The equivalent capacitance of the combination 10 For the network shown in Fig. 2.86, calculate the
between A and B in Fig. 2.82 is 41lF. equivalent capacitance between points A and B.
20 f.1F
IT
(Ans.6IlF)
~----~II---H--;
C

Fig. 2.82
(i) Calculate capacitance of the capacitor C.
(ii) Calculate charge on each capacitor if a 12 V
H
Fig. 2.86
battery is connected across terminals A and B.
(iii) What will be the potential drop across each 11. Calculate the capacitance of the capacitor C in
capacitor? [CBSE D 091 Fig. 2.87. The equivalent capacitance of the combi-
[Ans. (i) 51lF (ii) 48 IlC (iii) 2.4 V, 9.6 Vj nation between P and Q is 30 1lF. [CBSE OD 95]
O

6. How would you connect 8, 12 and 241lF capacitors (Ans. 60IlF)


to obtain (i) minimum capacitance (ii) maximum
capacitance? If a potential difference of 100 volt is
applied across the system, what would be the C 20 f.1F
charges on the capacitors in each case?
P~~~--4~--~S--~Q
[Ans. (i) In series, Cmin = 41lF,q = 41lC,
M

(ii) In parallel, Crnax = 44IlF, ql = 800 1lC,


q2 = 1200IlC, q3 = 24001lC]
Fig. 2.87
7. Calculate the capacitance of the capacitor in Fig. 2.83,
if the equivalent capacitance of the combination bet-
12. Calculate the equivalent capacitance between points
ween A and B is 151lF. [CBSE D 941
A and B of the combination shown in Fig. 2.88.

A~~~"~:F)
(Ans. 60 1lF)

IIlF IIlF
Fig. 2.83
Fig. 2.88
2.44 PHYSICS-XII

13. Find the equivalent capacitance between points A


and B for the network shown in Fig. 2.89.

C1 Cz ( Ans. ~ JlF)

A "T"
C3 1 J!F B

Fig. 2.93

18. Find the resultant capacitance between the points X

R
and Y of the combination of capacitors shown in
Fig. 2.89 Fig. 2.94. [Haryana 01]

14. Calculate the equivalent capacitance between the (Ans. 2.5 JlF)
points A and B of the circuit given below.
C C [CBSE F 95]

SI
z 28
Ao----1~~ 1
(Ans.-JlF)
4J!F r~ •..
:J!Fr~ ~

C6I'"' C'W'
"lB
5J!F

Fig. 2.94
C,1'"' 19. The outer cylinders of two cylindrical capacitors of
IT
capacitance 2.2 JlFeach are kept in contact and the
Fig. 2.90 inner cylinders are connected through a wire. A
battery of emf 10 V is connected, as shown in
15. A network of six identical capacitors, each of value Fig. 2.95. Find the charge supplied by the battery to
Cis made, as shown in Fig. 2.91. Find the equivalent the inner cylinders. (Ans. 44 JlC)
capacitance between the points A and B.
(Ans. 4C/ 3)
H
A 0---,:--1

lOV
I----'--<l B
O

Fig. 2.91

16. Find the equivalent capacitance between the points Fig. 2.95
A and B of the network of capacitors shown in
Fig. 2.92. (Ans. 1JlF) 20. In Fig. 2.96, C1 = 1JlF, C2 = 2 JlFand c,
= 3 JlF.Find
M

the equivalent capacitance between points A and B.


(Ans.6JlF)

A 0---< >---QB

Fig. 2.92
Fig. 2.96
17. Find the capacitance between the points A and B
of the assembly shown in Fig. 2.93. 21. Four capacitors of equal capacitances are con-
(Ans. 2.25 JlF) nected in series with a battery of 10 V, as shown in
ELECTROSTATIC POTENTIAL AND CAPACITANCE 2.45

Fig. 2.97. The middle point B is connected to the 25. A variable capacitor has n plates and the distance
earth. What will be the potentials of the points A between two successive plates is d.. Determine its
and C? (AnS. VA = + 5 V, Vc = - 5 V) capacitance. (Ans. C = (n - ~ Eo A J
lOV

J;~:l
26. A network of four capacitors each of 12 JlF
capacitance is connected to a 500 V supply as
shown in Fig. 2.101. Determine
(a) equivalent capacitance of the network, and
(b) charge on each capacitor. [eBSE OD 10]
" [Ans. (a) 16 JlF (b) ql = q2 = q3 =2000 JlC,
Fig. 2.97 q4 =6000 flq

R
22. Determine the potential difference across the plates
of each capacitor of the network shown in Fig. 2.98.
Take Ez > fl·
Ans. v: = (Ez - fl) c; v:
r = (Ez - fl) c;. J

SI
( 1 c;.+c; 2 c;.+c;
Fig. 2.101
27. For the network shown in Fig. 2.102, compute
3J.lF 3J.lF 3J.lF

'~~,~~
IT
Fig. 2.98
bo--1~~~
23. Find the potential difference between the points A 3J.lF 3J.lF 3J.lF
and B of the arrangement shown in Fig. 2.99.
Fig. 2.102
(Ans. - 8 V)
(i) the equivalent capacitance between points a
H
and b.
(ii) the charge on each of the capacitors nearest to a
and b when Vab = 900 V.
(iii) V cd when Vab = 900 V.
r
O

[Ans. (i) 1 JlF (ii) 900 JlC (iii) 100 Vj


Fig. 2.99
HINTS
24. Determine the potential difference VA - VB bet- 1. Proceed as in Example 45 on page 2.35.
ween points A and B of the circuit shown in
2. Proceed as in Example 46 on page 2.35.
Fig. 2.100.Under what condition is it equal to zero? 2x3
M

3. C = 1+ -- = 2.2 JlF.
2+ 3
4. Let the capacitances be C, 2 C and 3 C. Then
Cp = C + 2 C + 3 C = 6 C
1 1 1 1 11 6C
-=-+-+-=- or C =-
Cs C 2C 3C 6C s 11
Po--f---l 1-----4t--- I---t----o Q 60 6C 60
Given Cp - Cs = 11 pF or 6 C - 11= 11 pF
or C= 1pF
v So the individual capacitances are 1 pF, 2 pF and
Fig. 2.100 3 pF.
2.46 PHYSICS-XII

5. (i) As 20 /IF capacitor and capacitor C are in series, 13. C1and ~ are in parallel between points A and D. So
their equivalent capacitance is the equivalent capacitance between A and D is
C = Cx20 C'=C1+~=1+1=2/lF
AB C+ 20
The given network now reduces to the equivalent
20C circuit shown in Fig. 2.103.Between points A and B,
or 4/lF=--
C+20 now C and C; are in series and C4 in parallel. Hence
or 4C+ 80 = 20C the equivalent capacitance between A and B is
or C = 5/lF. CZ

(ii) Charge on each capacitor,


q = CAB V = 4/lF x 12V = 48 /lC H~

R
C'~C~B
...) P .D . on 20/lF capacitor
(III . =-- q =--48/lC = 2.4 V
20/lF 20/lF Fig. 2.103
2 !iF
q
48/lC CC; 2x1 8
P.D. on capacitor C = - = -- = 9.6 V.
C 5/lF C = -- + C = -- + 2 = - /IF.
C+C; 4 2+1 3

SI
6. (i) For minimum capacitance, the three capacitors 14 Capacitors C;, ~ and C4 are connected in series,
must be connected in series. Then their equivalent capacitance ~ is given by
1 1 1 1 1
-- =- + - + - =- or Cmin = 4/lF. 11111111
Cmin 8 12 24 4 -=-+-+-=-+-+-=-
c, C; c, C4 4 2 4 1
(ii) For maximum capacitance, the three capacitors
~ =l/lF
must be connected in parallel. Then
Also, CSand C6 are in series, the equivalent capa-
Cmax = 8 + 12 + 24 = 44/lF.
citance is
IT
(iii) In series combination, charge is same on all
CSxC6 2x4 4
capacitors. Cs =--=-=-/IF
CS+C6 2+43
q = CV = 4/lF x 100 V = 4/lC.
~ and Csform a parallel combination of capacitance,
In parallel combination, charges on the
4 7
capacitors are ~=~+Cs=I+-=-/lF
3 3
= C1V = 8/lF x 100 V = 800/lC
1ft
H
Now C1and ~ form a series combination. The equi-
q2 = C;V = 12/lF x 100 V = 1200/lC
valent capacitance C between A and B is given by
q3 = C;V = 24/lF x 100 V = 2400/lc.
1 1 1 1 3 19 28
7. The combined capacitance of the parallel com- - = - + - = - + - = - or C =-/IF.
CC1 ~4728 19
bination of two 10 /IF capacitors is 20 /IF.This com-
O

bination is connected in series with capacitance C. 15. The equivalent network is shown in Fig. 2.104.
1 1 1 1 1 1 4-3 1
.. -+-=- or -=---=--=-
20 C 15 C 15 20 60 60
or C = 60/lF.
8. All capacitors are in series.
M

4 1
- -- or C = 4 /IF.
C l/lF
Fig. 2.104
9. C=~+~+~=~+ 2C1C;.
C1+C; C1+C; C1+C; Clearly, the equivalent capacitance
1 1 1 1 1 = [2 C and C in series] II [Cand 2 C in series]
10. C ="'9+ "'9+ "'9= '3' C = 3/lF
2C x C C x2C 4C
=---+---=-.
C = 3/lF + C = 3/lF + 3/lF = 6/lF. 2C+ C C+ 2C 3
1 1 1 16. Two 2/lF capacitors at the left side of the network
11. - + - = - :. C = 60/lF.
C 60 30 are in series. Their equivalent capacitance is
1 1 112 2x2
12. - = -- + - + -- =- .. C = 0.5 /IF. C =-- =l/lF
C 1+1 1 1+1 1 s 2+2
ELECTROSTATIC POTENTIAL AND CAPACITANCE 2.47

The capacitance Cs and the next capacitor of 1J..lFare


in parallel. Their equivalent capacitance is
Cp = 1+ 1= 2J..lF
Proceeding in this way, we finally get two 2 J..lF
capacitors connected in-series.
·. Equivalent capacitance between A and B
Fig. 2.105
2x2
=--=lJ..lF.
2+ 2 Proceeding as in the above problem 22, we get
_ (E2 - f1) ~Cz
17. The given arrangement is a balanced wheatstone
q- ~+Cz

R
bridge. Proceed as in Example 57 on page 2.39.
18. The arrangement between the points A and B is a P.D. across the plates of ~,
balanced wheatstone bridge. Proceeding as in v =!L=(E2-f1)Cz=(12V-24V)4J..lF =-SV
Example 57, we find that the equivalent capacitance 1 ~ ~+Cz 2J..lF+4J..lF
between A and Bis

SI
C' = 5J..lF 24. Suppose the charge ql flows in the upper branch
and 'h in the lower branch. Then
Now the capacitor C' and the left out capacitor of
5 J..lFare in series. The equivalent capacitance
between points X and Y will be
V = ql [~ + ~ J or
C'x5 5x5
C = -,-
C +5
= --
5+5
= 2.5IlF. Also, V = q2 [.2.c, + .2.J
C4
19. The two capacitors are connected in parallel
V~C4
·. C = 2.2 + 2.2 = 4.4 J..lF or q2 = +C
IT
~
4
Charge, q = CV = 4.4 J..lFx 10 V = 44 J..lc.
20. The three capacitors are connected in parallel :. VA - VB = (VQ - VB) -(VQ - VA) = i£. _!!L
C4 Cz
between points A and B.
·. C = ~ + Cz + ~ = 1+ 2 + 3 = 6 J..lF.
Putting the values of ~ and q2' we get

21. Here VB = O. As the capacitances are equal on the V -V -~-~


H
two sides of point B, A B-~+C4 ~+Cz
.. VA - VB = VB - Vc
_ V [ Cz~ - ~C4 ]
or VA + VC = 2 VB = 0 (~ + Cz)(~ + C4)
But VA - Vc = 10 V
For VA - VB = 0, we have
O

.. VA = + 5 V and Vc = - 5 V.
22. Let charge q flow across the capacitor plates until or £L= ~
Cz C4
the current stops. In a closed circuit,
L 6V = 0 25. The given arrangement is equivalent to (n -1)
capacitors joined in parallel.
or f1 +!L-E2+!L=0 .. C = (n - 1) Eo A .
M

~ Cz
d
or q [ \+CzCz J = E2 - f1 26. (a) ~23
12J..lF
= -3- = 4 J..lF

(E2-f1)~Cz Ceq = ~23 + C4 = 4 + 12 = 16 J..lF.


or
q= ~+Cz
(b) ql = q2 = q3 = ~23 V = 4 J..lFx500V = 2000 J..lC
P.D. across plates of ~ =!L = (E2 - f1) Cz q4 = C4V = 12 J..lFx 500 V = 6000 uc
~ ~+Cz
27. (i) Three 3J..lFcapacitors in series have equivalent
PD. across plates of Cz =!L = (E2 - f1) ~ . capacitance = 1J..lF. The combination is in
Cz ~+Cz parallel with 2 JlF capacitor.
23. The given arrangement is equivalent to the circuit •. Equivalent capacitance between.c and d
shown in Fig. 2.105. =1+2=3J..lF
2.48 PHYSICS-XII

The situation is repeated for points e and f Suppose now a small additional charge dQ be trans-
Hence there are three 31lF capacitors in series ferred from plate 2 to plate 1. The work done will be
between points a and b. Equivalent capacitance
between a and b = 11lF.
dW '" V' . dQ = Q . dQ
, C
(ii) Potential drop of 900 V across a and b is equally The total work done in transferring a charge Q from
shared by three 31lF capacitors. plate 2 to plate 1 [Fig. 2.105(b)] will be
Hence charge on each capacitor nearest to a
and b
W=f dW=f
Q Q
-.dQ=
[Q,2]Q
-
1 Q2
= 300 x 3 = 900 IlC o C 2C 0 2' C
(iij) Potential drop of 300 V across e and f is equally

R
shared by 31lF capacitors. This work done is stored as electrical potential
energy U of the capacitor.
Hence Vcd = 100 V.
1Q21 21
U= - .- =- . CV =- QV [.: Q = CV]
2.24 ENERGY STORED IN A CAPACITOR 2 C 2 2

SI
38. How does a capacitor store energy ? Derive an 39. If several capacitors are connected in series or
expression for the energy stored in a capacitor. parallel, show that the energy stored would be additive in
either case.
Energy stored in a capacitor. A capacitor is a device
to store energy. The process of charging up a capacitor Energy stored in a series combination of capacitors.
involves the transferring of electric charges from its For a series combination, Q = constant
one plate to another. The work done in charging the Total energy,
capacitor is stored as its electrical potential energy. This
energy is supplied by the battery at the expense of its U = Q2 . .!. =Q2 . [~ +~ +~ +...
]
IT
stored chemical energy and can be recovered by
2 C Z C1 C2 C3

allowing the capacitor to discharge. Q2 Q2 Q2


=--+-+--+ ...
Expression for the energy stored in a capacitor. 2C1 2C2 2C3
Consider a capacitor of capacitance C. Initially, its two
plates are uncharged. Suppose the positive charge is or U = U1 + U2 + U3 + ...
transferred from plate 2 to plate 1 bit by bit. In this
H
Energy stored in a parallel combination of
process, external work has to be done because at any capacitors. For a parallel combination, V = constant
stage plate 1 is at higher potential than the plate 2.
Total energy,
Suppose at any instant the plates 1 and 2 have charges
Q and - Q respectively, as shown in Fig. 2.106(a). Then U = -1 CV 2 = -1 [C1 + C2 + C3 + ...] V
2

the potential difference between the two plates will be 2 2


O

V,=Q = -1 C1 V 2 + -1 C2 V 2 + -1 C3 V 2 + ...
C 222
or U = U1 + U2 + U3 + ...
Q' -Q'-dQ' Q -Q
+ + Hence total energy is additive both in series and parallel
••
M

+ + combinations of capacitors.
+ + •
+ dQ' + -
+ + -> ~ - 2.25 ENERGY DENSITY OF
+ ~ - +E~ AN ELECTRIC FIELD
+ + •
+ + •• 40. Where is the energy stored in a capacitor? Derive
+ + • an expression for the energy density of an electric field.
+ + •
Energy density of an electric field. When a
1 (a) 2 1 (b) 2
capacitor is charged, an electric field is set up in the
region between its two plates. We can say that the
Fig. 2.106 (a) Work done in transferring charge dQ' from plate 2 work done in the charging process has been used in
to plate ~. (b) Total work done in charging the capacitor may be creating the electric field. Thus the presence of an
considered as the energy stored in the electric field between electric field implies stored energy or the energy is stored
the plates. in the electric field.
ELECTROSTATIC POTENTIAL AND CAPACITANCE 2.49

Consider a parallel plate capacitor, having plate from the conductor at higher potential to that at lower
area A and plate separation d. Capacitance of the potential till their potentials become equal. Thus the
parallel plate capacitor is given by charges are redistributed. But the total charge still
E A remains Q1 + Q2'
C=_o_
d If the capacitance of the thin connecting wire is
If c is the surface charge density on the capacitor negligible and the conductors are a sufficient distance
plates, then electric field between the capacitor plates apart so that do not exert mutual electric forces, then
will be their combined capacitance will be C1 + C2.

E=~ . Total charge


Common potential = ---------'=------
Total capacitance

R
EO

Charge on either plate of capacitor is


or V = gl + Q2 = C1 VI + C2 V 2
Q = c A= EoEA C1 + C2 C1 + C2
Energy stored in the capacitor is
If after redistribution charges on A and Bare Q'1 and

SI
U=Q2 =(EoEA)2 =..!.E E2Ad Q'2 respectively, then
2C 2. EoA 2 0
Q'1 = C1 V
d
But Ad = volume of the capacitor between its two Q'1 = C1
plates. Therefore, the energy stored per unit volume or the Q'2 C2
energy density of the electric field is given by Thus, after redistribution, the charges on the two
U 1 2 conductors are in the ratio of their capacitances.
U = Ad =2 EO E
IT
42. When two charged conductors having different
Although we have derived the above equation for a capacities and different potentials are joined together,
parallel plate capacitor, it is true for electric field due to show that there is always a loss of energy.
any charge configuration. In general, we can say that Loss of energy in redistribution of charges. Let C1
an electric field E can be regarded as a seat of energy with and C2 be the capacitances and VI and V2 be the poten-
energy density equal to.3.. EO E2. Similarly, energy is also tials of the two conductors before they are connected
H
2 together. Potential energy before connection is
associated with a magnetic field.

2.26 REDISTRIBUTION OF CHARGES


v, = 21 C1VI 2 + 21 C2"2 2 r

41. If two charged conductors are touched mutually After connection, let V be their common potential.
O

and then separated, prove that the charges on them will be Then
divided in the ratio of their capacitances. V = Total charge = Q1 + Q2 = C1VI + C2V2
Redistribution of charges. Consider two insulated Total capacitance C1 + C2 C1 + C2
conductors A and B of capacitances C1 and C2, and Potential energy after connection is
carrying charges Q1 and Q2 respectively. Let VI and
M

V2 be their respective potentials. Then 1 2 1 2 1 2


Uf = 2 C1V + 2 C2V = 2 (C1 + C2) V
Q1 = C1VI and Q2 = C2 V2
=..!.(C + C ) [C1V1 + C2V2]2
2 1 2 C +C
1 2

=..!. (C1 VI + C2V2)2


2 (C1+C2)
Loss in energy,

Fig. 2.107 Redistribution of charges.


U= u, - U
f

Now, if the two conductors are joined by a thin = ..!. C V 2 + ..!. C V 2 _..!. . (C1 VI + C2v2l
conducting wire, then the positive charge will flow 2 1 1 2 2 2 2 (C + C )
1 2
2.50 PHYSICS-XII

1 2 2 2 2 Example 68. Two parallel plates, separated by 2 mm of air,


---- [C1 VI + C1C2 VI + C1C2 V2
2 (C1 + C2) have a capacitance of 3 x 10-14 F and are charged to a
+ C2 V2
2 2
-
2 2
C1 VI - C2 V2
2 2
- 2 C1C2 VI V2]
potential of 200 V. Then without touching the plates, they
are moved apart till the separation is 6 mm. (i) What is the
.! C1C2 [V 2 +V 2 -2 V V] potential difference between the plates ? (ii) What is the
2 (C + C ) 1 2 1 2 change in energy ?
1 2
Solution. Charge, q = CV =3 x 10-14 x 200 =6 x 10-12 C
_ 1 C1C2 (VI - V2)2
- 2· c +C 1 2
When the separation increases from 2 mm to 6 mm,
the capacitance becomes
This is always positive whether VI > V2 or VI < V2.

R
So when two charged conductors are connected, C' =!£. C =~x3x 10-14 =10-14 F
charges flow from higher potential side to lower d' 6
potential side till the potentials of the two conductors (i) P.D. between the plates becomes
get equalised. In doing so, there is always some loss of 6 10-12
potential energy in the form of heat due to the flow of V' = ~ = x = 600 V.
C 10-14

SI
charges in connecting wires.
(ii) Initial energy stored in the capacitor,
Examples based on
U = .! CV 2 =.! x 3 x 10-14 X (200)2 = 6 x 10-10 J
"iff' Energ StoreCiin Ca acitors 2 2
Formulae Used Final energy stored in the capacitor
1. Energy stored in a capacitor, U' = .! CV,2 =.! x 10-14 x (600l = 18 x 10-10 J
2 2 2
U =]. cv 2 =]. . .L =]. qV
= U' - U = 12 x 10-10 J.
Increase in energy
IT
2 2 C 2
2. Energy stored per unit volume or the energy Example 69. Two capacitors of capacitances C1 = 3 JlF and
density of the electric field of a capacitor, C2 = 6 JlF arranged in series are connected in parallel with a
1 2 third capacitor C3 = 4 JlF The arrangement is connected to a
u =- EO E
2 6.0 V battery. Calculate the total energy stored in the
3. Electric field between capacitor plates, E = ~ capacitors, [CBSE Sample Paper 98]
H
EO Solution. Equivalent capacitance of the series
Units Used combination of C1 and C2 is given by
Capacitance is in farad, charge in coulomb, C = C1C2 = 3 x 6 = 2 JlF
electric field in NC-1 or Vm -1, energy in joule and C1 + C2 3+6
energy density in Jm-3.
O

Combination C' is in parallel with C3.


Example 66. How much work must be done to charge a :. Total capacitance,
• . , -6
24 JlF capacitor when the potential difference between the C = C + C3 =2 + 4 =6 JlF =6 x 10 F
plates is 500 V ? [Haryana 02]
Energy stored,
Solution. Here C = 24 JlF = 24 x 10-6 F, V = 500 V
M

Work done,
U =.! cv 2 =.! x 6 x 10-6 x 62 = 1.08 x 10-4 J.
2 2
W =.! CV2 =.! x 24 x 10-6 x (500)2 = 3 J. Example 70. Three identical capacitors C1' C2 and C3 of
2 2
capacitance 6 JlF each are-connected to a 12 V battery as
Example 67. A capacitor is charged through a potential shown. Find:
difference of200 V, when 0.1 C charge is stored in it. How (i) charge on each capacitor.
much energy will it release, when it is discharged ?
(ii) equivalent
[ISCE 98]
capacitance of the
Solution. Here V =200 V, q =0.1 C network.
1 1 (iii) energy stored in
Energy stored, U =-qV =- x 0.1 x 200 =10 J
2 2 the network of
When the capacitor is discharged, it releases the capacitors.
same amount of energy i.e., 10 J. [CBSE D09]
Fig. 2.108
ELECTROSTATIC POTENTIAL AND CAPACITANCE 2.51

Solution. (i) C1 and C2 are connected in series across Energy stored in C1'
12 battery while C3 is in parallel with this combination. U = L= 49.5 x 2 x 11 x 10- 6
= 544.5 J
Equivalent capacitance of C1 and C2 is 1 2 C1 2 x 1 x 10-6
C C 6x6
C = 1 2 = -- = 3 /IF Total energy stored in the arrangement
12 C +C 6+6
1 2
= 544.5 + 49.5 = 594.0 J.
Charge on either of the capacitors C1 and C2 is same.
Example 72. In a camera-flash circuit (Fig. 2.110), a
q1 = q2 = C12 V =3/lFx 12 V = 36/lC 2000 /IF capacitor is charged by a 1.5 V cell. When aflash is
Charge on C3' required, the energy stored in the capacitor is discharged by
means of a trigger T through a discharge tube in 0.1

R
q3 = 6/lFx 12 V = n/lC
millisecond. Find the energy stored in the capacitor and the
(ii) Equivalent capacitance of the network, power of the flash. [ISCE 97]

C = ~2 + ~ = 3 IlF + 6/lF = 9 /IF. Electronic trigger

(iii) Energy stored in the network,

SI
U= .!CV2 =.! x9xlO-6 x(12)2 = 6.48x10-4 J. Discharge
2 2 tube
Example 71. In Fig. 2.109, the energy stored ill C4 is 27 J.
Calculate the total energy stored in the system.
2!!F Fig. 2.110

Solution. Here C = 2000 /IF = 2 x 10-3 F, V = 1.5 V

~IlI-F_Qo--+-_3_IlF--I ~ Energy stored in the capacitor,


IT
C1 61lF ~ U = .! cv 2 =.! x 2 x 10-3 x (1.5)2 = 2.25 x 10-3 J
2 2
C4
Time during which capacitor is discharged for
Fig. 2.109 producing flash,
Solution. Energy stored in C4 is t =0.1 millisecond =0.1 x 10-3 s = 10-4 s
H
3
1 2 Power of flash, P = U = 2.25 x 10- = 22.5 W.
U4 = 2" C4 V = 27 J t 10-4

or .! x 6 x 10-6 x V 2 =27 Example 73. A 800 pF capacitor is charged by a 100 V


2 battery. After some time the batten) is disconnected. The
O

or V 2 = 27 x 2 = 9 x 106 capacitor is then connected to another 800 pF capacitor.


6 x 10- 6 What is the electrostatic energy stored ? [CBSE F 09]

Energy stored in C2 ' Solution. Here C1 = C2 = 800 pF =8 x 10-10 F,

U2 = .! x 2 x 10-6 x 9 x 106 = 9 J VI = 100 V, V2 = a


2
M

Common potential,
Energy stored in C3'
10
V=~V1+c;V2= 8xlO- xlOO+0 =50V
U3 = .! x 3 x 10-6 x 9 x 106 = 13.5 J ~ + c; 8 x 10-10 + 8xlO-10
2
Energy stored in C2' C3 and C4 1 2
Uf = 2(~ + C;)V
= U2 + U3 + U4 =9 + 13.5 + 27 = 49.5 J
Equivalent capacitance of C2' C3 and C4 connected = .!(8xlO-10 + 8xlO-10) x(50)2 = 2 x10-6 J
2
in parallel
=2 +3+5=l1JlF Example 74
(i) A 900 pF capacitor is charged by a 100 V battery.

2x11x10-6
q2 = 49.5 J [u q
=2 C
2
] How much electrostatic energy is stored by the
capacitor?
2.52 PHYSICS-XII

(ii) The capacitor is disconnected from the battery and V == Total cha!~ = _Cf.L!"_!lL = CIV] + C2V2
connected to another 900 pF capacitor. What is the Total capacitance C] + C2 C] + C2
electrostatic energy stored by the system ?
V= C]V]
(iii) Where has the remainder of the energy gone ? or
C1 + C2
[NCERT ; CBSE OD 90]
(ii) Energy stored in the capacitors before connection,
Solution. (i) The charge on the capacitor is
1 2
q = CV =900 x 10-]2 Fx 100 V =9 x 10-8 C u, = 2 C]V]
The energy stored by the capacitor is
Total energy after connection,
U = ~ CV 2 = ~ qV = ~ x 9 x 1O-8C x 100 V 1 2
Uf = 2 (C1 + C2)V

R
222
= 4.5 x 10-6 J. 2 2
= ~ (C + C) C1V]
+q -q -.i.+ s. 2 i 2 (C + C2)2
1
2 + -
=( C]C]+ C Ju.
2

Co~ = ~2 C]CiV]2
+
+

SI
+C 2 2 I

.!L+ q
2+ - 2' Clearly, Uf < Ui
+
+ Hence total energy of the combination is less than
the sum of the energy stored in the capacitors before
Fig. 2.111
they are connected.
(ii) In the steady situation, the two capacitors have Example 76. Two capacitors of unknown capacitances C]
their positive plates at the same potential, and their and C2 are connected first in series and then in parallel
IT
negative plates at the same potential. Let the common across a battery of 100 V. If the energy stored in the two
potential difference be V'. The charge on each capacitor combinations is 0.045 J and 0.25 J respectively, determine
is then q = CV'. By charge conservation, q' = q /2. the values of C] and C2• Also calculate the charge on each
.', TotaI energy of the system capacitor in parallel combination. [CBSE D 15]

Solution. For series combination, we have


=2 x~ q V' = q V' = q . £
H
2 C U= 1 C]C2 V2
2 C] + C2
=±. q; =±.qv=~x~qv [-: q'=~and~=V]
0.045 = ~ C]C2 x (100l ... (i)
2 C] + C2
= ~ x 4.5 x 10-6 J = 2.25 x 10-6 J.
O

2 For parallel combination, we have


(iii) There is a transient period before the system 1 2
U =2(C] + C2)V
settles to the situation (ii). During this period, a
transient current flows from the first capacitor to the 0.25 = ~(C] + C2)x (100)2
second. Energy is lost during this time in the form of 2
M

heat and electromagnetic radiation. or C] + C2 = 0.5 x 10-4 ...(ii)


Example 75. A capacitor is charged to potential V]' The C C
power supply is disconnected and the capacitor is connected From (i), 0.045 = ~x ] 2 x (100)2
2 0.5 x 10-4
in parallel to another uncharged capacitor.
8
(i) Derive the expression for the common potential of the or C]C2 = 0.045 x 10-
combination of capacitors. Now (C] -C )2
2
= (C] + C2)2 -4C]C2
(ii) Show that total energy of the combination is less than = (0.5 x 10-4)2 _ 4 x 0.045 x 10-8
the sum of the energy stored in them before they are = (0.25 -0.180) x 10-8 = 0.07 x 10-8
connected. [CBSE OD 15]
4 4
So Iu tiIOn. (.)
I
LCd
et ] an C2 b e th e capac!it ances 0f .. C] -C2 = .J0.07 x 10- =026x 10- ... (iii)
the two capacitors and V be their common potential. On solving (ii) and (ii.i), we get
Then C] = 0.38 x 10-4 F and C2 = 0.12 x 10-4 F
ELECTROSTATIC POTENTIAL AND CAPACITANCE 2.53

Charges on capacitors CI and C2 in parallel also C. The voltage across each capacitor is now V/2.
combination are: The final total energy stored in the two capacitors is
QI = C1 V =0.38x 10-4 x 100C = O.38x 10-2 C U =.! C ( V)2 +.! C ( V)2 =.! cv 2
f 2 2 2 2 4
Q2 = C2V =0.12 x 10-4 x 100C = 0.12x 10-2 C.
=2.5 J
Example 77. A capacitor of capacitance 6 /IF is charged to
a potential of 150 Y Its potential falls to 90 V, when another
capacitor is connected to it. Find the capacitance of the u-: -
Uf _ 2.5 _ 1 _ 1 . 2
5.0 - 2" - . .
second capacitor and the amount of energy lost due to the
connection.

R
~rOblems for Practice
Solution. Here CI = 6 /IF, VI = 150 V, V2 = 0,
1. A capacitor charged from a 50 V d.c. supply is
V =90 V, C2 =? found to have charge of 10/lc. What is the
Common potential, capacitance of the capacitor and how much energy
is stored in it ? [ISeE 93]

SI
V = CI VI + C2 V2
(Ans. 0.2/lF, 2.5 x 10-4 J)
C1 + C2
2. For flash pictures, a photographer uses a capacitor
6
90 V = 6 x 10- x 150 + 0 of 30 /IFand a charger that supplies 3 x 103 V. Find
or
6 x 10-6 + C2 the charge and energy expended in joule for each
flash. (Ans. 9 x 10-2 C, 135 J)
6
C + 6 x 10-6 = 6 x 10- x 150 = 10 x 10-6 3. An electronic flash lamp has 10 capacitors, each 10 /IF,
or 2
90 connected in parallel. The lamp is operated at
100 volt. How much energy will be radiated in the
=4x 10-6 F = 4 /IF.
IT
or C2
flash? (Ans. 0.5 J)
Initial energy stored, 4. Three capacitors of capacitances 10/lF, 20/lF and
1 2 1 -6 2 30 /IFare connected in parallel to a 100 V battery as
Ui = U1 = 2" CI VI = 2" x 6 x 10 x (150) shown in Fig. 2.112. Calculate the energy stored in
the capacitors. [ISeE 94]
= 6.75 x 10-2 J
(Ans. 0.3 J)
H
Final energy stored,
1 2
Uf = 2" (CI + C2) V

= .! (6 + 4) x 10-6 x (90)2 = 4.05 x 10-2 J


O

2
The loss of energy on connecting the two capacitors,
+ -
flU = U - Uf = (6.75 - 4.05) x 10-2
i
'------11------'
= 2.7 x 10-2 J = 0.027 J. lOOV
M

Example 78. A battery of10 V is connected to a capacitor Fig. 2.112


of capacity 0.1 F. The battery is now removed and this
5. A variable capacitor is kept connected to a 10 V
capacitor is connected to a second uncharged capacitor. If the battery. If the capacitance of the capacitor is
charge distributes equally on these two capacitors, find the changed from 7 J.!Fto 3 /IF, what is the change in the
total energtj stored in the two capacitors. Further, compare energy? What happens to this energy? [ISeE 96]
this energy with the initial energy stored in the first
(Ans. 2 x 10-4 J, decrease in energy)
capacitor. [Roorkee 96]
6. The plates of a parallel plate capacitor have an area
Solution. Initial energy stored in the first capacitor is
of 100em 2 each and are separated by 2.5 mm. The
U = .! CV2 =.! x 0.1 x (10)2 = 5.0 J capacitor is charged to 200 V. Calculate the energy
'22 stored in the capacitor. [Punjab 96]
When the first capacitor is connected to the second (Ans. 7.08 x 10-7 J)
uncharged capacitor, the charge distributes equally. 7. A 80 /IF capacitor is charged by a 50 V battery. The
This implies that the capacitance of second capacitor is capacitor is disconnected from the battery and then
2.54 PHYSICS-XII

connected across another unchanged 320 I-lF 13. Two capacitors are in parallel and the energy stored
capacitor. Calculate the charge on the second is 45 J, when the combination is raised to potential
capacitor. .[CBSE D 94 C] of 3000 Y. With the same two capacitors in series,
(Ans. 3.2 xlO-3q the energy stored is 4.05 J for the same potential.
What are their individual capacitances?
8. Find the total energy stored in the capacitors in the
network shown below. [CBSE D 04] (Ans. 91-lF,11-lF)
(Ans. 3.6 x 10-5 J) 14. Find the ratio of the potential differences that must
be applied across the parallel and the series
combination of two capacitors C1. and c; with their
capacitances in the ratio 1 : 3 so that the energy

R
stored in the two cases, becomes the same.
[CBSE F 10]
(Ans . .f3: 4)
HINTS
1. C = !L = 10 I-lC= 0.2 1-lF.

SI
Fig. 2.113 Y 50V
9. A 10 I-lFcapacitor is charged by a 30 V d.c. supply Energy stored,
and then connected across an uncharged 50 I-lF
capacitor. Calculate (i) the final potential difference U = 1 Cy2 = 1 x 0.2 x 10-6 x(50l = 2.5 x10-4 J.
across the combination, and (ii) the initial and final 2. Here C=30I-lF=3xlO-5F, Y=3x103V
energies. How will you account for the difference in 5
Charge, q = CY = 3 x 10- x 3 x 103 C = 9 x 10-2 C
energy? [CBSE OD 04]
Energy, U=1CY 2=1x3xlo-5x9xl06
[Ans. (i) 5V, (ii) Uj = 4.5 x 10-3 J,
IT
U = 0.75 x 10-3 Jl = 135 J.
f
10. Net capacitance of three identical capacitors in 3. Total equivalent capacitance,
series is 1 I-lF.What will be their net capacitance if
C = 10 x 10 f.lF= 100I-lF= 1O-4F
connected in parallel ?
Energy radiated
Find the ratio of energy stored in the two configu-
rations if they are both connected to the same source. = 1 CY 2 = 1 x 10-4 x(100)2 = 0.5 J.
H
[CBSE OD 11] (Ans. 9 I-lF, 1 : 9) 4. C = C1. + c; + C; = 10 + 20 + 30 = 60 I-lF
11. Two capacitors of capacitances 251-lFand 100I-lFare = 60 x 10-6 F
connected in series and are charged by a battery of
120 V. The battery is then removed. The capacitors U = 1 Cy2 = 1 x60 x 10-6 x(100)2 = 0.3 J.
O

are now separated and connected in parallel. Find


(i) p.d. across each capacitor (ii) energy-loss in the 5 Here Cj = 7f.lF= 7x 1O-6F,Y = 10 V
process. (Ans. 38.4 V, 0.05184 J)
u, = 1 cy2 = 1 x7 x 10-6 x(10)2 = 3.5 x 10-4 J
12. Figure 2.114 shews a network of five capacitors
connected to a 100 V supply. Calculate the total Again, C = 31-lF= 3 x 10-6 F, Y = 10 V
f
charge and energy stored in the network.
M

U = 1 C y2 = 1 x3 x 10-6 x(1O)2= 1.5 xlo-4J


[CBSE Sample Paper 08] f f
(Ans. 4 x lO-4C, 0.02 J) Decrease in energy = Uj - U = 2.0 x10-4 J.
f
Energy is lost as heat and electromagnetic
radiation.
6. Here A = 100cm2 = 10-2m2,
d = 2.5 mm = 2.5 x 10-3m, Y = 200 V
U =..! CY 2 =..!. EO A. Y 2
2 2 d
12 2
= ..!x 8.85 x 10- x 10- (200)2
2 2.5 x 10-3
'------IIIr---~
Fig. 2.114 lDDV = 7.08 x10-7 J.
ELECTROSTATIC POTENTIAL AND CAPACITANCE 2.55

8. The two 2!-1Fcapacitors on the right side are in Equivalent capacitance in parallel,
2x2
series, their equivalent capacitance = -- = 1!-IF C' = 25 + 100 = 125 !-IF
2+ 2
Total charge,
This 1!-IFcapacitance is in parallel with the central q' = 2400 + 2400 = 4800 !-IC
1 !-IFcapacitor. Their equivalent capacitance P.D. across each capacitor,
=1+1=2!-1F
This 2!-1F capacitance is in series with the 2 !-IF
V' = 1.. = 4800 !-IC= 38.4 V.
C' 125 !-IF
capacitor at the bottom. Their equivalent capacitance
2x2 (ii) In series,
=-- = l!-1F
2+ 2 cv 2 = .! x 20 x 10-6 x (120)2

R
U = .!
Finally, 1!-IFcapacitance is in parallel with the left 2 2
out 1!-IFcapacitor. The equivalent capacitance is = 0.144 J
C = 1 + 1 = 2 !-IF= 2 x 10-6 F In parallel,
V= 6V U' = V,2 =.! x 125 x 10-6 x (38.4)2 = 0.09216 J
.!C'

SI
2 2
U = .! CV 2 =.! x 2 x 10-6 x (6)2 :. Energy loss
2 2
= 3.6 x 10-5 J. = U - U' = 0.144 - 0.09216 = 0.05184 J.
12. The equivalent circuit diagram for the given
9. Here C1. = 10 !-IF= 10 x 10-6F, ~ =30V,
network is shown below:
~ = 50 !-IF= 50 x 10-6F, V2 =0
(i) Common potential,
C1. VI + ~ V2 10 x 10-6 x 30 + 0
V = = =5 V.
IT
C1. + ~ (10 + 50) x 10-6
(ii) Initial electrostatic energy of 10 !-IFcapacitor,
1 2
Ui = 2" C1. VI
100 V
= .! x 10 x 10-6 x (30)2 = 4.5 x10-3 J '-------iI It-------'
2
H
Final electrostatic energy of the combination, Fig. 2.115

Uf = ~(10 + 50) x 10-6 x(5)2 = 0.75 x ]0-3 J


Two 3!-1F capacitors in paraUel. The equivalent
Loss in energy = Ui - Uf = 3.75 x 10-3J capacitance,
O

The difference in energy is lost in the form of heat C1. = 3 + 3 = 6 !-IF


and electromagnetic radiation as the charge flows The l!-1F capacitor and a 2!-1F capacitor are in
from first capacitor to second capacitor. parallel. Their equivalent capacitance,
C ~ = 1+ 2=3!-1F
10. Here Cs ="3 = 1!-IF
M

Then C1. and ~ form a series combination of equi-


C= 3 !-IF valent capacitance,
Cp = 3C=9!-1F

U
_s
1
= _2__
c, V2
C 1
=~=_=1: 9 This combination is in parallel with the fifth capa-
2Ic p V
2
Up Cp 9 citor of 2 !-IF.
11. (i) Equivalent capacitance in series, :. Net capacitance, C = 2 + 2 = 4 !-IF
Total charge,
C = 25 x 100 = 20 F
25 + 100 !-I q = CV = 4 x 10- 6 x 100 = 4 x 10- 4 C
Total energy stored,
Charge on each capacitor in series,
q = CV = 20 !-IFx 120 V = 2400 !-IC U = .! CV2 =.! x 4 x 10-6 x (100)2 = 0.02 J.
2 2
2.56 PHYSICS-XII

14. Given Eo
Eo
Now Eind

or Conductor

Eo
or
Eo

R
Dielectric

Fig. 2.116 Difference in the behaviour of a conductor and


a dielectric in an external electric field.

In a dielectric, the external field Eo induces dipole

SI
moment by stretching or re-orienting the molecules of
the dielectric. The induced dipole moment sets up an
electric field Eind which opposes Eo but does not
exactly cancel this field. It only reduces it.
Polar and non-polar dielectrics. A dielectric may
consist of either polar or non-polar molecules. A
molecule in which the centre of mass of positive charges
(protons) does not coincide with the centre of mass of
2.27
IT
DiElECTRICS AND THEIR POLARIZATION negative charges (electrons) is called a polar molecule.
43. What are dielectrics ? Explain the difference in the The dielectrics made of polar molecules are called
behaviour of a conductor and a dielectric in the presence polar dielectrics. The polar molecules have unsym-
an external electric field. Distinguish between polar and metrical shapes. They have permanent dipole mo-
non-polar dielectrics. ments of the order of 10- 30 Cm. For example, a water
Dielectrics. In insulators, the electrons remain molecule has a bent shape with its two 0- H bonds
H
attached to the individual atoms or molecules. inclined at an angle of105c as shown in Fig. 2.117. It has
However, these electrons cansuffer small movements a very large dipole moment of 6.1 x 10- 30 Cm. Some
within the atoms or molecules under the influence of other polar molecules are HCI, N~, CO, Cf\0H, etc.
an external electric field. The net effect of these micro-
Non-polar
scopic movements gives rise to some important electric
O

properties to such materials. In view of these electrical


properties, insulators are called dielectrics.
CO,
A dielectric is a substance which does not allow theflow
of charges through it but permits them to exert electrostatic
forces on one another through it. A dielectric is essentially
an insulator which can be polarised through small localised
/
M

Polar p
displacements of its charges.
p
Examples. Glass, wax, water, air, wood, rubber,
stone, plastic, etc. HCl
Difference in the behaviour of a conductor and a Fig. 2.117 Some polar and non-polar molecules.
dielectric in the presence of an external electric field.
Dielectrics have negligibly small number of charge A molecule in which the centre of mass of positive
carriers as compared to conductors. charges coincides with the centre of mass of negative charges
In a conductor, the external field Eo moves the free is called a non-polar molecule. The dielectrics made of
charge carriers inducing field Eind in the opposite non-polar molecules are called non-polar dielectrics.
direction of Eo. The process continues until the two Non-polar molecules have symmetrical shapes. They
fields cancel each other and the net electric field in the have normally zero dipole moment. Examples of
conductor becomes zero. non-polar molecules are ~, N2, 02' CO2, CH4, etc.
ELECTROSTATIC POTENTIAL AND CAPACITANCE 2.57

44. How does a dielectric develop a net dipole moment tend to align with the field. As a result, there is a net
in an external electric field when it has (i) non-polar dipole moment in the direction of the field, as shown in
molecules and (ii) polar molecules? Fig. 2.llS(b)(ii). The extent of polarisation depends on
Polarization of a non-polar dielectric in an external relative values of two opposing energies :
electric field. In the absence of any electric field, the 1. The potential energy of the dipole in the external
centres of positive and negative charges of the field which tends to align the dipole with the field.
molecules of a non-polar dielectric coincide, as shown 2. Thermal energy of agitation which tends to
in Fig. 2.llS(a)(i). The dipole moment of each molecule randomise the alignment of the dipole.
is zero. In the presence of an external electric field Eo '
Hence both polar and non-polar dielectrics develop a
the centres of positive charges are displaced in the
net dipole moment in the presence of an external electric
direction of external field while the centres of negative

R
field. This fact is called polarization of the dielectric.
charges are displaced in the opposite direction. The ~
displacement of the charges stops when the force The polarization P is defined as the dipole moment
exerted on them by the external field is balanced by the per unit volume and its magnitude is usually referred to
restoring force due to the internal fields in the mole- ~
as the polarization density. The direction of P is same
cules. This induces dipole moment in each molecule

SI
~
i.e., each non-polar molecule becomes an induced as that of the external field E o'
dipole. The induced dipole moments of different mole-
45. Explain why the polarization of dielectric reduces the
cules add up giving a net dipole moment to the dielec-
electric field inside the dielectric. Hence define dielectric
tric in the direction of the external field, as shown in
constant.
Fig. 2.11S(a)(ii).
Reduction of electric field by the polarization o~ a
Eo = ° Eo ,,0
dielectric. Consider a rectangular dielectric slab placed
8) ~
~ ~ 8) in a uniform electric field Eo acting parallel to two of its
IT
~ faces, as shown in Fig. 2.119(a). Its molecular dipoles
8)
Dielectric slab
~ ~ ~ 8) 8) 8)
8) +
~ --Eo + $ <:::3) <:::3) $
H

Fig. 2.118 (a)


(i) (ii)

Polarization of a non-polar dielectric


in an external electric field.
+
+
+
8 <:::3)
:
<:::3) e
+
@ <:::3) Eo <:::3) @
Polarization of a polar dielectric in an external .-
O

+
electric field. The molecules of a polar dielectric have
+
@ <:::3) <:::3) @
permanent dipole moments. In the absence of any
external electric field, the dipole moments of different
..-- -- -~- -- - - - - - - - -- - - - ---'
Region of zero
molecules are randomly oriented due to thermal charge density
agitation in the material, as shown in Fig. 2.11S(b)(i). So
M

the total dipole moment is zero. When an external field Fig. 2.119(a) Polarization of a dielectric.
is applied, the dipole moments of different molecules - CJp + CJp

+ +
+
+
+
+
+
+
-r-+E; +
(ii) + +
I~----~------------~I
Fig. 2.118 (b) Polarization of a polar dielectric
Fig. 2.119(b) Reducedfieldin a dielectric,E = Eo - E p'
in an external electric field.
2.58 PHYSICS-XII

-> moment per unit volume or the polarisation density


align themselves in the direction of Eo. This results in
will be
uniform polarization of the dielectric, i.e., every small P = dipole moment of dielectric
volume of the slab has a dipole moment in the
->
volume of dielectric
direction of Eo. The positive charges of the dipoles of o,d
=--=-=(J
o,
first vertical column cancel the negative charges of the Ad A p
dipoles of second column and so on. Thus the volume
Thus the polarisation density may be defined as the charge
charge density in the interior of the slab is zero.
induced per unit surface area.
However, there is a net uncancelled negative charge on
the left face and uncancelled positive charge on the Obviously, a uniformly polarised dielectric with
uniform polarisation density P can be replaced by two

R
right face of the slab.
->
The uncancelled charges are the induced surface surface layers (perpendicular to P) of surface charge
-> densities ± (J P ,and zero charge density in the interior.
charges due to the external field Eo. Since the slab as a
47. Define electric susceptibility. Deduce the relation
whole remains electrically neutral, the magnitude of
between dielectric constant and electric susceptibility.

SI
the positive induced surface charge is equal to that of
->
the negative induced surface charge. Electric susceptibility. If the field E is not large,
->
Thus the polarized dielectric is equivalent to two then the polarisation P is proportional to the resultant
charged surfaces with induced surface charge densities ± (J p' ->
field E existing in the dielectric, i.e.,
Reduced field inside a dielectric and dielectric
constant. In case of a homogeneous and isotropic -> ->
dielectric, the induced surface charges set up an or P = EO X E
->
electric field Ep (field due to polarization) inside the where X (chi) is a proportionality constant called
electric susceptibility. The multiplicative factor
IT
EO is used
dielectric in a direction opposite to that of external to keep X dimensionless. Clearly,
->
field Eo' thus tending to reduce the original field in the ->
P
->
dielectric. The resultant field E in the dielectric will be
x=--
->
-> -> -> EO E
equal to Eo - Ep and directed in the direction of Eo.
Thus the ratio of the polarisation to EO times the electric
H
-> field is called the electric susceptibility of the dielectric. Like
The ratio of the original field Eo and the reduced P, it also describes the electrical behaviour of a dielec-
-> ->
field Eo - Ep in the dielectric is called dielectric constant tric. The dielectrics with constant X are called linear
dielectrics.
(K) or relative permittivity (Er). Thus
Relation between K and x. The net electric field in a
O

-> ->
polarised dielectric is
K=Eo= Eo -> -> ->
-> -> ->
E E-E E = Eo - Ep
o p

46. Define polarisation density. How is it related to But E= (Jp =~


M

the induced surface charge density ? P EO EO

Polarisation density. The induced dipole moment -> -> P


E = Eo--
developed per unit volume of a dielectric when placed in an EO
external electric field is called polarisation density. It is
denoted by P. Suppose a dielectric slab of surface area A
and thickness d acquires a surface charge density ± p (J or
due to its polarisation in the electric field and its two ->
faces acquire charges ± Qp. Then Dividing both sides by E, we get
Q
=---E. E
(J
1=---.1l. -X
p A ->
E
We can consider the whole dielectric slab as a large
dipole having dipole moment equal to Qp d. The dipole or l=K-X or K =1+ X
ELECTROSTATIC POTENTIAL AND CAPACITANCE 2.59

2.28 DiElECTRIC STRENGTH


48. What do you mean by dielectric strength of a
dielectric ?
Dielectric strength. When a dielectric is placed in a
very high electric field, the outer electrons may get
detached from their parent atoms. The dielectric then
behaves like a conductor. This phenomenon is called
dielectric breakdown. Fig. 2.120 A dielectric slab placed in a
The maximum electric field that can exist in a dielectric parallel plate capacitor.
without causing the breakdown of its insulating property is + Qp on the lower surface of the dielectric. These
called dielectric strength of the material. induced charges set up a field Ep inside the dielectric in

R
-+
The unit of dielectric strength is same as that of the opposite direction of Eo. The induced field is given by
electric field i.e., Vm -1. But the more common practical
unit is kV mm-1. E = (J P =R[(J = Q = P, polarisation density]
p EO EO P A
Table 2.1 Dielectric constants and dielectric

SI
The net field inside the dielectric is
strengths of some common dielectrics.

Dielectric Dielectric strength


Dielectric
constant in kV mm 1
Vacuum 1.00000 00 where K is the dielectric constant of the slab. So
between the capacitor plates, the field E exists over a
Air 1.00054 0.8
distance t and field Eo exists over the remaining
Water 81 - distance (d - t). Hence the potential difference
IT
Paper 3.5 14 between the capacitor plates is
Pyrex glass 4.5 13 V = Eo (d - t) + Et = Eo (d - t) + ~o t [.: ~ = K]
Mica 5.4 160
Porcelain 6.5 4 = Eo (d - t + .!) = ~ A (d - t + .!)
K EO K
H
2.29 CAPACITANCE OF A PARALLEl PLATE The capacitance of the capacitor on introduction of
CAPACITOR WITH A DIELECTRIC SLAB dielectric slab becomes

49. Deduce the expression for the capacitance of a C= Q= EOA


parallel plate capacitor when a dielectric slab is inserted V d-t+.!
between its plates. Assume the slab thickness less than the
O

plate separation.
Special Case If the dielectric fills the entire space
Capacitance of a parallel plate capacitor with a between the plates, then t = d, and we get
dielectric slab. The capacitance of a parallel plate E A
capacitor of plate area A and plate separation d with C=-O-.K=KC
O
d
vacuum between its plates is given by
M

Thus the capacitanceof a parallel plate capacitor increases


_ EoA K times when its entire space is filled with a dielectricmaterial.
CO-
d C
Clearly, K =-
Suppose initially the charges on the capacitor plates Co
are ± Q. Then the uniform electric field set up between
Dielectric constant
the capacitor plates is
Capacitance with dielectric between two plates :
(J Q
Eo=-=-- Capacitance with vacuum between two plates
EO AEo
Thus the dielectric constant of a dielectric material
When a dielectric slab of thickness t < d is placed
may be defined as the ratio of the capacitance of a capacitor
between the plates, the field Eopolarises the dielectric.
completely filled with that material to the capacitance of the
This induces charge - Qp on the upper surface and
same capacitor with vacuum between its plates.
2.60 PHYSICS-XII

2.30 CAPACITANCE OF A PARALLEL PLATE 2.31 USES OF CAPACITORS


CAPACITOR WITH A CONDUCTING SLAB 51. Mention some important uses of capacitors.
50. Deduce- the expression for the capacitance of a Uses of capacitors. Capacitors are very useful
parallel plate capacitor when a conducting slab is inserted circuit elements in any of the electric and electronic
between its plates. Assume the slab thickness less than the circuits. Some of their uses are
plate separation. 1. To produce electric fields of desired patterns,
Capacitance of a parallel plate capacitor with a e.g., for Millikan's experiment.
conducting slab. Consider a parallel plate capacitor of 2. In radio circuits for tuning.
plate area A and plate separation d. If the space bet-
3. In power supplies for smoothing the rectified
ween the plates is vacuum, its capacitance is given by
current.
_ EoA

R
CO- 4. For producing rotating magnetic fields in
d
induction motors.
Suppose initially the charges on the capacitor plates
5. In the tank circuit of oscillators.
are ± Q. Then the uniform electric field set up between
the capacitor plates is 6. They store not only charge, but also energy in
Q the electric field between their plates.

SI
cr
Eo=-=--
EO AEo
2.32 EFFECT OF DIELECTRIC ON
where cr is the surface charge density. The potential
VARIOUS PARAMETERS
difference between the capacitor plates will be
Qd 52. A parallel-plate capacitor is charged by a battery
Vo= Eod=- which is then disconnected. A dielectric slab is then
~AEO inserted to fill the space between the plates. Explain the
When a conducting slab of thickness t < d is placed changes, if any, that occur in the values of (i) charge on
between the capacitor plates, free electrons flow inside the plates, (ii) electric field between the plates, (iii) p.d.
IT
it so as to reduce the field to zero inside the slab, as between the plates, (io) capacitance and (v) energy stored
shown in Fig. 2.121. Charges - Q and + Q appear on in the capacitor.
the upper and lower faces of the slab. Now the electric
field exists only in the vacuum regions between the Effect of dielectric when the battery is kept
plates of the capacitor on the either side of the slab, i.e., disconnected from the capacitor. Let C!o ' Co ' Vo ' Eo
the field exists only in thickness d - t, therefore, potential and Uo be the charge, capacitance, potential difference,
H
difference between the plates of the capacitor is electric field and energy stored respectively before the
dielectric slab is inserted. Then
V = Eo (d - t) = ---.fL (d - t)
AEo Vo 1 2
C!o = CoVo ' Eo = d ' Uo =:2 CoVo
I
+ ~ + ~ + ~ Eo ~ + (i) Charge. The charge on the capacitor plates
O

~ +-+Q
remains C!o because the battery has been disconnected
1
d £=0 ~ Conducting before the insertion of the dielectric slab.
I1+ + + + + + +Q slab (ii) Electric field. When the dielectric slab is

1 - ~ - ~ _ ~ £~ ~ - ~---Q
I
inserted between the plates, the induced surface
charge on the dielectric reduces the field to a new value
M

given by
Fiq, 2.121 A conducting slab placed in a
parallel plate capacitor.

:. Capacitance of the capacitor in the presence of (iii) Potential difference. The reduction in the
conducting slab becomes electric field results in the decrease in potential difference.
_ Q _ EoA _ EoA d C_[_d_) C
Ed V
C-------or - V= Ed=_o-=~
- V - (d -t) - d . d - t d- t . 0 K K

(iv) Capacitance. As a result of the decrease in


Clearly, C > Co' Thus the introduction of a conducting
potential difference, the capacitance increases K times.
slab of thickness t in a parallel plate capacitor increases its
capacitance by a factor of _d_ . C = C!o = ---.9L = K C!o = K Co
d- t V VolK Vo
ELECTROSTATIC POTENTIAL AND CAPACITANCE 2.61

(v) Energy stored. The energy stored decreases by a For Your Knowledge
factor of K.

U =..! CV 2 =..! (KC ) ( VO)2 =..! . ..! C 11:2 = Uo . ~ Capacitance of a parallel plate capacitor with
2 2 0 K K 200 K compound dielectric.

53. A parallel plate capacitor is charged by a battery. A. Series typt. ~rang -nen If a capacitor is filled with
When battery remains connected, a dielectric slab is inserted n dielectric slabs of thicknesses t1, t2, ..... , tn' as shown
between the plates. Explain what changes, if any, occur in in Fig. 2.122(a), then this arrangement is equivalent to n
the values of ti) p.d. between the plates, iii) electric field capacitors connected in series.
between the plates, (iii) capacitance, (iu) charge on the With a single dielectric slab,

R
plates and (v) energy stored in the capacitor? eo A
Effect of dielectric when battery remains con- C= t
d - t+-
nected across the capacitor. Let 00 r Co' VO' Eo and Uo K

be the charge, capacitance, potential difference, electric Capacitance with n dielectric slabs will be
field and energy stored respectively, before the

SI
~J
eo A
introduction of the dielectric slab. Then
Vo
00 = CoVo ' Eo = -,
1
Uo = - CoVo
2 C = d _ (~ + t2 +... + tn) + [!L +Kr
!L + ... +
K2 Kn
d 2
But d = ~ + t2 + t3 + .....+ tn
(i) Potential difference. As the battery remains
" connected across the capacitor, so the potential difference eo A
"<remains constant at Vo even after the introduction of C= ~ t2 tn
-+-+ .....+ -
dielectric slab. Kr K2 Kn
IT
(ii) Electric field. As the potential difference
remains unchanged, so the electric field Eobetween the ...-- I_A _
capacitor plates remains unchanged.
V 11:
E=-=~=Eo
d d
~_::_tit3 1<3
(iiz) Capacitance. The capacitanceincreasesfrom Co to C.
H
C=K Co
(iv) Charge. The charge on the capacitor plates increases
from 00 to Q.
___ K,_, It.
Q=CV=KCO·VO=KOo· L
O

(v) Energy stored. The energy stored in the capacitor ig. 2.122(a) Fig.2.122(b)
increases K times.
1 2 1 2 1 2 B. Para'! Is',;e arr; '1 em
'1 The arrangement shown
U="2CV ="2(KCO)VO =K'"2COVO =KUO'
in Fig. 2.122(b) consists of n capacitors in parallel,
having plate areas ~,~, ..... r An ' and plate
M

Table 2.2 Effect of dielectric on various parameters. separation d.


The equivalent capacitance of the parallel arrange-
Battery disconnected Battery kept connected
ment will be
from the capacitor across the capacitor
Q= (1 (constant) Q=K(1
C= C; + C 2 + .....+ Cn
11: V = Vo (constant) Kr eo ~ K2 eo ~ Kn eo An
V=~ = d + d + ... + d
K
eo
E= fu E = fu (constant) or C = d (Kr ~ + K2 ~ + ...+ Kn An)
K
A
C=Kc:;, C=Kc:;, If ~ =~= ..... =An=-;;-,then
U
U =-.!l. U=KUO eo A
K C=---;t;;(Kr + 1<2+ .....+ Kn)
2.62 PHYSICS-XII

isfilled with a medium of dielectric constant 5. Calculate the


value of capacitance of the capacitor in the second case.
[CBSEOD 06]
Solution. Capacitance of the capacitor with air
Formulae Used
between its plates,
1. Capacitance of a parallel plate capacitor filled with a EA
dielectric of dielectric constant K. Co =_0_ =S pF
d
EoKA
C:=KCo=-- When the capacitor is filled with dielectric (K = 5)
d
between its plates and the distance between the plates
2. Capacitance of a parallel plate capacitor with a dielec- is reduced by halt capacitance becomes

R
tric slab of thickness t « d) in between its plates, EOKA EO x 5 x A
C= EoA C=d/2= d/2 =10Co

d -t (1-~) or C = 10 x S = 80 pF.
3. Capacitance of a parallel plate capacitor with a con- Example B1 . Figure 2.123 shows tuio-identical capacitors,

SI
ducting slab of thickness t « d) in between its plates, C1 and C2, each of 1 ~F capacitance connected to a battery of
C= EO A 6 V. Initially switch '5' is closed. After some time '5' is left
d -t
4. Capacitance of spherical capacitor filled with a
dielectric,
ab
C= 41t Eo K. b_ a
Fig. 2.123
5. Capacitance of a cylindrical capacitor filled with a
IT
open and dielectric slabs of dielectric constant K = 3 are
dielectric,
inserted tofill completely the space between the plates of the
C = 21t EO K I
two capacitors. How will the (i) charge and (ii) potential
b
2303log10 - difference between the plates of the capacitors be affected
a
after the slabs are inserted? [CBSE D III
6. Effect of dielectric with battery disconnected from
Solution. With switch 5 closed, VI = V2 = 6 V
the capacitor,
H
:. q1 =q2 =1~Fx6V=6~C
Q = QyrV = Vo , E = Eo , C = K Co' U = Uo
- KKK When dielectric slabs (K =3) are inserted, capaci-
7. Effect of dielectric with battery connected across the tance of each capacitor becomes 3 ~F.
capacitor, P.D. across Cl, V{ = 6 V
O

Q = K(;b , V = Vo ' E = Eo' C = K Co' U = K Uo


Charge, eft =3 ~Fx 6V = 18 ~C
Units Used
With switch 5 open, the p.d. on C2 attains a new
Capacitance C is in farad, charge q in coulomb,
value but charge q2 is still 6 ~C
potential difference V in volt, area A in m 2,
thicknesses d and t in metre. V' = 6~C =2 V.
M

Constant Used 2 3 ~F

Permittivity constant, EO = 8.85 x 1O-12C2N-1m -2.


Example B2. An ebonite plate (K =3), 6 mm thick, is
introduced between the parallel plates of a capacitor of plate
Example 79. In a parallel plate capacitor, the capacitance area 2 x 1O-2~ and plate separation 0.01 m Find the
increases from 4 ~F to SO~F, on introducing a dielectric capacitance.
medium between the plates. What is the dielectric constant of Solution. Here t =6 mm =6x 1O-3m,
the medium? 2 2
A=2x10- m , d =0.01 m, K=3
Solution.
C= EO A = 8.S5 x 10-12 x 2 x 10-2
K = Capacitance with dielectric = SO~F = 20.
Capacitance without dielectric 4 ~F 3
d - t ( 1 - ~) 0.01-6 x 10- (1- ~)
Example BO. A parallel plate capacitor with air between
14
the plates has a capacitance of 8 ~F. The separation between 17.7 x 10: = 29.5 x 10-12 F = 29.5 pF.
the plates is now reduced by half and the space between them 6 x 10-
ELECTROSTATIC POTENTIAL AND CAPACITANCE 2.63
x y
same p.d. between the plates, the plate separation is to be
Example 83. Two parallel PlateL:J increased by 2.4 mm Find the dielectric constant of the slab.
capacitors, X and Y, have the same
area of plates and same separation Solution. Let Eo be the electric field between the
between them. X has air between +- capacitor plates before the introduction of the slab.
the plates while Y contains a 12 V Then, the p.d. between the plates is
dielectric medium of s, = 4. Fig. 2.124
Vo=Eod
(i) Calculate capacitance of each capacitor if equivalent Suppose the separation between the plates is
capacitance of the combination is 4 IlF. increased by d' to maintain the same p.d. after the
(ii) Calculate the potential difference between the plates introduction of the slab of thickness t. Then
of X and Y.

R
(iii) What is the ratio of electrostatic energy stored in X Vo = Eo (d + d' - t) + Eo . t
K
and Y ? [CBSE D 04, 09]
E
Solution. (i) Let Cx = C. Then c, = e,c = 4C Eo (d + d' - t) + -.!1.. t = Eo d
K
Now X and Yare connected in series.
K=_t_= 3.0mm =5.

SI
or
.. C = CXCy = C. 4C t -d' 3.0 mm -2.4 mm
eq Cx+Cy C+4C
Example 86. The area of parallel plates of an air-filled
or 4 IlF = iC or C = 5 IlF capacitor is 0.20 ~ and the distance between them is O.Olm
5 The p.d. across the plates is 3000 V. When a 0.01 m thick
Hence Cx =C=5IlF and c, =4C=4x5=20IlF. dielectric sheet is placed between the plates, the p.d. decreases
to 1000 V. Determine (i) capacitance of the capacitor before
(ii) Let V be the p.d. across X. Then p.d. across Y
placing the sheet (ii) charge on each plate (iii) dielectric
will be V / 4.
IT
constant of the material tio) capacitance of the capacitor after
.. V+V = 12 or V = 9.6 V placing the dielectric (v) permittivity of the dielectric. Given
4 Co=8.85 x 10-12 Fm-1.
Hence Vx = V =9.6 V and Vy = V / 4 =2.4 V. Solution. (I) Capacitance
of air-filled capacitor is
12
(iii) Energy stored in X = ~ C(9.6 )2 = = 4 : 1. i Co = CoA = 8.85 x 10- x 0.20 = 1.77 x 10-10 F.
Energy stored in Y ~4C(2.4)2 1 d 0.01
H
(ii) Charge on each plate,
/ Example 84.An electric field Eo =3 x 10 4 Vm-1 is q = Co Vo = 1.77 x 10-10 x 3000
established between the plates, 0.05 m apart, of a parallel
= 5.31 x 10-7 C.
plate capacitor. After removing the charging battery, an
uncharged metal plate of thickness t = 0.01 m is inserted (iii) Dielectric constant of the material is
O

between the capacitor plates. Find the p.d. across the K- C _ q/V _ Vo _3000_3
capacitor (i) before, (ii) after the introduction of the plate. - Co - q / Vo - V -1000 - .
(iii) What would be the p.d. if a dielectric slab (K = 2) were
introduced in place of metal plate? [Roorkee 91]
(iv) Capacitance after the dielectric sheet is introduced,
Solution. (i) The p.d. across the capacitor plates C= K Co =3 x 1.77 x 10-10 = 5.31 x 10-10 F.
M

before metal plate is inserted, (v) Permittivity of the dielectric is


4
Vo = Eo d =3 x 10 x 0.05 = 1500 V. C= K EO = 3 x 8.85 x 10-12 = 2.65 x 10-11 Fm -1.

(ii) As no electric field exists in metal plate, so the Example 87. The capacitance of a parallel plate capacitor is
p.d. after the introduction of metal plate is 50 pF and the distance between the plates is 4 mm It is
V = Eo (d - t) =3 x 104 x (0.05 -0.01) = 1200 V. charged to 200 V and then the charging battery is removed.
Now a dielectric slab (K = 4) of thickness 2 mm is placed.
(iii) When dielectric slab (K =2) is introduced, the
Determine (i) final charge on each plate (ii) final potential
p.d. becomes
4 difference between the plates (iii) final energy in the
V = Eo(d - t)+ Eo t =1200+ 3x 10 x 0.01 =1350 V. capacitor and (io) energy loss.
. K 2
Solution. Capacitance of air-filled capacitor,
Example 85. A parallel plate capacitor is charged to a
certain potential difference. When a 3.0 mm thick slab is _ Co A
Co --d- ...(1)
slipped between the capacitor plates, then to maintain the
2.64 PHYSICS-XII

Capacitance with dielectric slab of thickness Solution. If Vo is the potential difference when
t « d) is there is no dielectric, then the electric field between the
C= go A ...(2) capacitor plates will be
d-t+t/K V
E-~
0- d
(i) The charge on capacitor plates, when 200 V p.d.
is applied, becomes After the dielectric is inserted, the electric field in
q = Co Vo = 50 x 10- 12
x 200 8
= 10- C the dielectric reduces to
E
Even after the battery is removed, the charge of E=~
K
10- 8 C on the capacitor plates remains the same.
Now the potential difference between the plates

R
(ii) On placing the dielectric slab, suppose the capa-
will be
citance becomes C and potential difference V. Then
q=Co Vo=CV

or
V- Co V _d-t+t/K V
-C 0- d 0

SI
[Using (1) and (2)]
Thus the potential difference decreases by a factor
= 4 - 2 + 2 / 4 x 200 = 125 V.
4 of (K + 3) /4 K, while the free charge qo on the plates
remains same. The capacitance increases to a new
(iii) Final energy in the capacitor is
value given by
U = .! q V = .! x 10-8 x 125 = 6.25 x 10- 7 J. C - qo _ 4K qo _ 4K C
2 2 -Y--K+3'V -K+3 o·
o
IT
(iv) Energy loss
1 Example 90
= uo - u = 2 q (Vo - V)
(a) Find the ratio of the capacitances of a capacitor filled
with two dielectrics of same dimensions but of dielectric
=.! x 10-8 x (200 -125) constants KI and K2 ' respectively.
2
(b) A capacitor is filled with two dielectrics of the same
= 3.75 x 10-7 J.
H
dimensions but of dielectric constants KI = 2 and
Example 88. A parallel plate capacitor is formed by two K2 =3. Find the ratio of capacities in two possible
plates, each of area 100 cd, separated by a distance of 1 mm arrangements. [MNREC 85]
A dielectric of dielectric constant 5 and dielectric strength Solution. (a) The two possible arrangements of the
1.9 x 107 Vm-l is filled between the plates. Find the two dielectrics are shown in Figs. 2.125(a) and (b).
O

maximum charge that can be stored on the capacitor without


causing any dielectric breakdown.
I
Solution. Electric field between capacitor plates is
given by
E=~=-q-
-=E2J=-[J K2
M

go go A
K K

As the electric field should not exceed 1.9 x 107 Vm -I,


I
so the maximum charge that can be stored is Fig. 2.125 (a) (b)

q= K go AE (i) The arrangement (a) can be supposed to be a


= 5 x 8.85 x 10-12 x 100 x 10-4 x 1.9 x 107 parallel combination of two capacitors, each with plate
area A/2 and separation d. Therefore, the total capa-
= 8.4 x 10-6 C. citance is
Example 89. A slab of material of dielectric constant K has
the same area as the plates' of a parallel plate capacitor but
has a thickness 3d / 4, where d is the separation of the plates.
How is the capacitance changed when the slab is inserted
between the plates ? [NCERT]
ELECTROSTATIC POTENTIAL AND CAPACITANCE 2.65

(ii) The arrangement (b) can be supposed to be a citance of the capacitor becomes 2/3rd of its original
series combination of two capacitors, each with plate value. What is the new distance between the
area A and separation d/2. Therefore, the total plates? [CBSE OD OSC] (Ans. 8 mm)
capacitance C ' is given by 7. The distance between the parallel plates of a charged
1 1 1 1 1 capacitor is 5 cm and the intensity of electric field is
- = - + - = --,-- + ---;--
c C'1 C2 _06 __AK 1 60 AK2 300 V em -1. A slab of dielectric constant 5 and thick-
d/2 d/2 ness 1 em is inserted parallel to the plates. Determine

d
= 260A
(1-;s 1)
+ K2
the potential differencebetween the plates, before and
after the slab is inserted ? (Ans. 1500 V, 1260 V)
8. A parallel plate capacitor with plate separation
5 mm is charged by a battery. It is found that on

R
or C' = 260 A ( KlK2 ) introducing a mica sheet 2 mm thick, while keeping
d Kl + K2 the battery connections intact, the capacitor draws
Ratio of the capacitances in the two arrangements is 25% more energy from the battery than before. Find
2 the dielectric constant of mica. (Ans. 2)
C _ 60 A(K1 + K2) d(K1 + K2) _ (K1 + K2)
9. Figure 2.126 shows a parallel plate capacitor of

SI
C,- 2d '26 AK1K2- 4K1K2
0 plate area A and plate separation d. Its entire space
(b) Here Kl =2, K2=3 is filled with three different dielectric slabs of same
thickness. Find the equivalent capacitance of the
C (2 + 3)2 25
arrangement. [Ans. C = 3 eo A Kl K2K3 ]
C 4x 2 x 3 24 d (K1K2+ K2K3+ K3 K1)

~roblems For Practice r- I_A__ ---"l


iT P
r
1. A parallel-plate capacitor having plate area 100cm2 d/3
IT
and separation 1.0 mm holds a charge of 0.12~C +
d/3 Kj K2 K3
when connected to a 120 V battery. Find the +
d/3
dielectric constant of the material filling the gap. .i.

(Ans.11.3)
~. Find the length of the paper used in a capacitor of
bB
H
capacitance 2 ~F, if the dielectric constant of the Fig. 2.126 Fig. 2.127
paper is 2.5 and its width and thickness are 50 mm 10. The space between the plates of a parallel plate
and 0.05 mm, respectively. (Ans. 90 m) capacitor of capacitance C is filled with three
3. A parallel-plate capacitor consists of 26 metal strips, dielectric slabs of equal thickness, as shown in
each of 3 em x 4 em, separated by mica sheets of Fig. 2.127. If the dielectric constants of the three
O

dielectric constant 6 and uniform thickness 0.2 mm. slabs are K1, K2and K3, find the new capacitance.
Find the capacitance. (Ans. 7.97 x 10- 9 F)
[Ans. C=~(KI + K2+ K3)]
4. A parallel-plate capacitor of capacity 0.5 ~F is to be 3
constructed using paper sheets of thickness 0.04mm 11. A slab of material of dielectric constant Khas the
as dielectric. Find how many circular metal foils same area as the plates of a parallel plate capacitor
M

of diameter 0.1 m will have to be used. Take the but has thickness d /2, where d is the separation
dielectric constant of paper used as 4. (Ans. 73) between the plates. Find the expression for the
5. When a slab of insulating material 4 mm thick is capacitance when the slab is inserted between the
introduced between the plates of a parallel plate plates. [CBSE F 10; OD 13] (Ans. ~C )
capacitor, it is found that the distance between the K+l 0
plates has to be increased by 3.2 mm to restore the HINTS
capacitance to the original value. Calculate the
dielectric constant of the material. (Ans. 5) 1. Capacitance,
6. The two plates of a parallel plate capacitor are
If K is the dielectric constant, then
.4 mm apart. A slab of dielectric constant 3 and
thickness 3 mm is introduced between the plates C= K eo A = Kx8.85 x 10-12 xlOOx10-4 = 1O-9F
with its faces parallel to them. The distance d 1.0 x 10-3
between the plates is so adjusted that the capa- :. K =11.3.
2.66 PHYSICS-XII

3. Arrangement of n metal plates separated by dielectric For charge to remain constant,


acts as a parallel combination of (n - 1) capacitors. Cu Vo = CV
C = (n - 1) ICEOA
Cu x 1500 = 25 Cu x V or V = 1260 V.
d 21
25 x 6 x 8.85 x 10-12 x 3 x 4 x 10-4 8. As the battery connections are intact (V = constant)
0.2 x 10-3 and the capacitor draws 25% more charge, so the
capacitance also increases by 25%. That is
= 7.97 x 10-9 F.
C= 125 C =~ r
= (n - 1) ICEOA 100 0 4 '1J
4. As C
d eaA 5 EoA
or
.. 0.5 xlO-6 F d- t ( 1_ ~) = 4 . -d-

R
(n -1) x 4 x 8.85 x 10-12 x3.14 x(0.05)2
0.04 x 10-3 or d - t (1-;) = ~ or t (1-;) = ~
0.5 x 0.04 x 103 7 97
or n - 1= = 1. =: 72 1- ~ = ~ = _5_ = ..! or IC= 2
4 x 8.85 x 3.14 x (0.05)2 or
K 5t 5x2 2

SI
or n= 73. 9: The given arrangement is equivalent to three
EOA capacitors connected in series. Each such capacitor
5. Capacitance without dielectric, C = --
d has plate area A and plate separation d.
K1EOA 31C1EOA
When dielectric is introduced, Cr=~= d

Cz = 3 1C2EOA and c, = 3 K3 EOA


As the capacitance remains same in both cases, so d d
The equivalent capacity C of the given arrangement
IT
EoA EoA is given by
-d- = d'-t (1-;) 1 1 1 1 d (1 1 1J
C=Cr +Cz +c; =3EoA K1 +1(2 +1C3

or d=d'-t (1-;) or d'-d=t (1-;) or C= 3 EOA Kl K2 K3


d (K1 K2 + K2 ~ + K3 K1)
= 3.2 mm, t = 4 mm
H
But d' - d
10. Original capacitance, C = EOA
.. 3.2 = 4(1- ;) The new arrangement
d
is equivalent to three capaci-
tors connected in parallel. Each such capacitor has
or 1 - ~ = 3.2 = 0.8 or ~ = 0.2 or IC= 5.
K 4 IC plate area A /3 and plate separation d. The new
capacitance is
O

6. ~ x Capacitance with air = Capacitance with dielectric C' = Cr + Cz + c;


1C1EOA/3 1C2 EaA/3 K3EOA/3
2 EoA eaA = + + ~--"----
or d d d
3d= d'-t +~ EOA
K = 3d CK1 + K2 + K3)
M

or ~ (d' - 3 + ~) = d = 4 mm or d' = 8 mm
or C'= ~(ICI + Kz + ~).
3
7. P.D. before the dielectric slab is inserted, v:
11. Without dielectric, EU =-.l!
Vo = EU d = 300 V em -1 x 5 em = 1500 V. d
Ea A E A With dielectric, E= EU
Cu = -- = _0_ farad K
d 0.05
or EO A = 0.05 Co
V = EU.!! + E.!! = fu.!! + Eu.!!
2 2 2 1C2
Capacitance with dielectric slab, EUd (K+l) VoCK+1)
=T'-K-= 2K
EOA 0.05 Co 25 Co
C= t 0.01 - C _ qo _ 2Kqo _ 21C
d- t + - 0.05 - 0.01 + _ 21 or
5 - V - V ( K + 1) - IC+ 1 Cu
K o
ELECTROSTATIC POTENTIAL AND CAPACITANCE 2.67

2.33 DISCHARGING ACTION OF SHARP Total potential on the outer sphere,


POINTS : CORONA DISCHARGE VR = Potential due to its own charge Q
54. Briefly explain discharging action of sharp points + Potential due to the charge q on
or corona discharge. the inner sphere
Discharging action of sharp points : Corona
discharge. When a spherical conductor of radius r
carries a charge q, its surface charge density is
= 4n\J~ + ~]
Insulating
suspension
Potential on the inner
G=!L .:s: sphere due to its own charge is
1 q
A . 4n r2 V =---
1 4n eo . r

R
Electric field on the surface is
As the potential at every
point inside a charged sphere
is the same as that on its
surface, so potential on the

SI
inner sphere due to charge Q Fig. 2.129 Small charged
sphere suspended inside a
on outer sphere is
LowE + charged spherical shell.
V =_1_ Q
2 4ne · R
o
Total potential on inner sphere

Vr = 4:eJ; + ~]
IT
Fig. 2.128 Corona discharge.
Hence the potential difference is
The pointed end of a conductor is highly curved
and its radius of curvature r very small. If the con- V - V
r R
= -q-
4ne
o
[!-!]
r R
ductor is given a charge q, then the charge density Gat
the pointed end will be very high. Consequently, the So if q is positive, the potential of the inner sphere
H
electric field near the pointed end will be very high will always be higher than that of the outer sphere.
which may cause the ionisation or electrical break- Now if the two spheres are connected by a conducting
down of the surrounding air. The oppositely charged wire, the charge qwill flow entirely to the outer sphere,
ions neutralise the pointed end while the similarly irrespective of the charge Q already present on the
charged ions are repelled away. Fresh air molecules outer sphere. In fact this is true for conductors of any
O

come near the pointed end and take away its charge, shape.
setting up a kind of electric wind. This process by which
the charge at the pointed end of a conductor gets discharged 2.35 VAN DE GRAAFF GENERATOR *
is called corona discharge. The discharge is often 56. Explain the basic principle, construction and
accompanied by a visible glow near the pointed end. working of Van de Graaff generator.
M

Van de Graaff generator. It is an electrostatic


2.34 COLLECTING ACTION OF A generator capable of building up high potential differences of
HOLLOW CONDUCTOR the order of107 volts.
55. A small sphere of radius r and charge q is enclosed Principle. The working of a Van de Graaff gene-
by a spherical shell of radius R and charge Q. Show that if rator is based on following two electrostatic phenomena:
q is positive, charge q will necessarily flow from the
(i) Discharging action of sharp points (corona
sphere to the shell (when the two are connected by a wire)
discharge) i.e., electric discharge takes place in air or
no matter what the charge Q on the shell is. [NCERT]
gases readily at the pointed ends of conductors.
Collecting action of a hollow sphere. Consider a
(ii) If a charged conductor is brought into internal
small sphere of radius r placed inside a large spherical contact with a hollow conductor, all of its charge
shell of radius R. Let the spheres carry charges q and Q, transfers to the hollow conductor, howsoever high the
respectively. potential of the latter may be.
2.68 PHYSICS-XII

Construction. A large spherical conducting shell (of


few metres radius) is supported at a height several + + + Metal

R
metres above the ground on an insulating column. A shell,S
long narrow belt of insulating material, like rubber or
silk, is wound around two pulleys, P1 at ground level
and P2 at the centre of the shell. +
This belt is kept continuously moving by an electric

SI
+
motor attached to the lower pulley Pl' Near the bottom
and the top of its run, the belt passes close to two +
CoJlector 4-~'--- Ion
sharply pointed brass combs Bl and ~, pointing
comb source
towards the belt. The comb ~, called spray comb is
given a positive potential of 10 kV with respect to the Insulating belt to carry
and deliver charge
earth by means of a battery; while the comb ~, called
collecting comb, is connected to the spherical shell S.
Working. Due to the high electric field at the Insulating pillar

IT
pointed ends of comb ~, the air of the neighbourhood
gets ionised and its positive charge repelled or sprayed
on to the belt, which moves up into the shell S. As it
passes close to comb ~, it induces a negative charge at
the pointed ends of comb ~ and a positive charge on
the shell S. The positive charge spreads uniformly on
Discharge tube

Grounded metal
base
H
the outer surface of the shell S.The high electric field at ..:c..
the pointed ends of comb ~ ionises the air there and T
H.T.B. ,
repels the negative charges on to the belt which
-'- Target
neutralise its positive charge. This process continues.
O
As more and more positive charge is given to the shell,
its potential continues to rise. In this way, a high
potential of 6 to 8 million volts can be built upon the
sphere.
A discharge tube is placed with its upper end Fig. 2.130
M

inside the hollow sphere and lower end earthed. The


ion source is placed at the upper end of the tube. The Use. The high potential difference set up in a
high potential on the sphere repels the charged Van de Graaff generator is used to accelerate charged
particles downward with large acceleration, where particles like protons, deutrons, a-particles, etc. to high
they hit the target atoms to bring about the nuclear energies of about 10 MeV, needed for experiments to
disintegration. probe the small scale structure of matter.
G I DELI N ES To NCERT EXERCISES

2.1. Two charges 5 x 1O-8C and -3 x 10-8 C are located The other possibility is that x may also lie on OA

R
16 cm apart. At what point on the line joining the two charges is produced, as shown in Fig. 2.187.
the electric potential zero ? Take the potential at infinity to be
zero. ql=5xl0
-8
C q2=-3xlO
-8
C

SI
Ans. Zero of electric potential for two charges. As • • I
o A P
shown in Fig. 2.186, suppose the two charges are placed on
X-axis with the positive charge located at the origin 0. 11+-: 0_.1_6_-_-_-~-x--'I'-- x - 0.16 =1
-8 -8
ql = 5x 10 C q2 = - 3 x 10 C Fig. 2.187
·.-----------+I--------------~·
o p A
I--- x ----0·01+1_ --- 0.16- x -----I As V;+ V2 =0

IT
8 8
Fig. 2.186 _1_[ 5 x 10- _ 3x10- ] =0
4TC EO X X - 0.16
Let the potential be zero at the point P and OP = x. For
x < 0 (i.e., to the left of 0), the potentials of the two charges 5 3 =0
or
cannot add up to zero. Clearly, x must be positive. If x lies x x - 0.16
H
between 0 and A, then
or x = 0.40 m = 40 em.
Vl + V2 = 0
Thus the electric potential is zero at 10 em and 40 em
_1_ [ ql + q2 ] - 0 away from the positive charge on the side of the negative
O
4TCEO x 0.16-x charge.
2.2. A regular hexagon of side 10 em has a charge of 5 J.1C at
9 [ 5 x 10- 8 3 x 10- 8 ]
each of its vertices. Calculate the potential at the centre of the
or 9 x 10 - =0
x 0.16 - x hexagon.
M

Ans. Clearly, distance of each charge from the centre


or 5 3 = 0
x 0.16 - x o is
r= IDem =D.1Dm
or x = 0.10 m =10 em,
2.92 PHYSICS-XII

When the capacitor is filled with dielectric (lC = 6)


between its plates and the distance between the plates is
reduced by half, capacitance becomes,
EO lC A EO x 6 x A EoA
C'=--= =12-
d' d/ 2 d
or C' = 12 x 8 = 96 pF.
2.6. Three capacitors each of capacitance 9 pF are connected
in series. (a) What is the total capacitance of the combination?
(b) What is the potential difference across each capacitor when
the combination is connected to a 120 V supply ?

R
Fig. 2.188 Ans. (a) If C is the equivalent capacitance of the series
combination, then
Magnitude of each charge is
111111131
q = 5 IlC = 5x io+c -=-+-+-=-+-+-=-=-
CC1, <; c;99993
:. Potential at the centre 0 is

SI
1 q 6x9 x 109 x 5 x 10-6 or C= 3 pF.
V=6.--.-=------- (b) As all the capacitors have equal capacitance, so
41tEO r 0.10
potential drop t1 V would be same across each capacitor.
6
= 2.7 x10 V. .. V = t1 VI + t1 V2 + t1 V3
2.3. Two charges + 2 IlC and - 2 IlC are placed at points A = t1 V + t1 V + t1 V = 3t1 V
and B, 6 em apart. (i) Identify an equipotential surface of the
t1 V = V = 120 = 40 V.
system (ii) What is the direction of the electric field at every or 3 3
point on the surface ?
2.7. Three capacitors of capacitances 2 pF, 3 pF and 4 pF are
Ans. (i) The equipotential surface will be a plane
IT
connected in parallel. (a) What is the total capacitance of the
normal to AB and passing through its midpoint 0, as
combination? (b) Determine the charge on each capacitor if the
shown in Fig. 2.189. It has zero potential everywhere.
combination is connected to a 100 V supply.
Ans. (a) For the parallel combination, total capacitance
is given by
C = C1, + <; + <; = 2 + 3 + 4 = 9 pF.
H
o (b) When the combination is connected to 100 V
A
supply, charges on the capacitors will be
Equipotential ql = C1,V =2x10-12 x100=2x10-10C
surface q2 = <;V = 3 x 10-12 x 100 = 3 x1().-lOC
q2 = c;V =4x10-12 x100=4x10-10C.
O

Fig. 2.189
2.8. In a parallel plate capacitor with air between the plates,
(ii) The direction of electric field is normal to the plane
each plate has an area of 6 x 10-3 m2 and the distance between
in the direction AB i.e., from positive to negative charge.
the plates is 3 mm. Calculate the capacitance of the capacitor. If
2.4. A spherical conductor of radius 12 cm has a charge of
the capacitor is connected to a 100 V supply, what is the charge
1.6 x 10-7 C distributed uniformly on its surface. What is the
on each plate of the capacitor ?
M

electric field
Ans. Capacitance of capacitor with air between its
(a) inside the sphere (b) just outside the sphere
plates is
(c) at a point 18 cm from the centre of the sphere?
Co = EoA = 8.85 x 10-12 x 6 x 10-3
Ans. Refer to the solution of Example 75 on page 1.60.
d 3x10-3
2.5. A parallel plate capacitor with air between the plates = 1.8 x 10- F = 18 pF.
11
has a capacitance of 8 pF (lpF = 10-12 F). What will be the
capacitance if the distance between the plates be reduced by half, Charge,
the space between them is filled with a substance of dielectric q = Co V = 1.8 x 10-11 x 100 = 1.8 x10-9 C.
constant, lC = 6 ? 2.9. Explain what would happen if in the capacitor given in
Ans. Capacitance of the capacitor with air between its Exercise 2.8, a 3 mm thick mica sheet (of dielectric constant = 6)
plates, were inserted between the plates, (i) while the voltage supply
E A remains connected (ii) after the supply was disconnected.
= -0-=8pF [CBSE Sample Paper 98]
d
ELECTROSTATIC POTENTIAL AND CAPACITANCE 2.93

Ans. From the above question, we have 2.12. A charge of 8 mC is located at the origin. Calculate
CD= 1.8 x 10-11 F = 18 pF, qo = 1.8 x 10-9 C the work done in taking a small charge of - 2 x 10-9 Cfrom
Also, K = 6
a point P (0, 0, 3 em) to a point Q (0, 4 em, 0) via a point R (0,
6 em, 9 em).
(i) When the voltage supply remains connected, the
potential difference between capacitor plates remains Ans. As the work done in taking a charge from one
same i.e., 100 V. point to another is independent of the path followed,
therefore
The capacitance increases K times.
.. C = K CD= 6 x 18 = 108 pF. W = qo [VQ - VpJ = qo[-q- - -q-]
41t EO r2 41t Eo '1
The charge on the capacitor plates will be
q = CV = 108 x 10-12 x 100 = 1.08 x 10-8 C.
4~:J~-I]

R
(ii) After the supply is disconnected, the charge on the =

capacitor plates remains same i.e.,


qo = 18 x 10- 9 C
z R(O, 6 ern, 9 ern)

The capacitance increases K times.

SI
C= K CD=108 pF.
The potential difference between the capacitor plates
becomes
v= Va = 100 =16.6 V.
K 6 P(O, 0, 3 em)
2.10. A 12 pF capacitor is connected to a 50 V battery. How
much electrostatic energy is stored in the capacitor?
Ans. Here C = 12 pF = 12 x 10-12 F, V = 50 V
IT
Energy stored,
q =2 me Q(O, 4 ern, 0)

U = -.!CV2 = -.!x12x10-12 x(50)2 =1.5 x 10-8 J.


2 2 x
2.11. A 600 pF capacitor is charged by a 200 V supply. It is Fig. 2.190
then disconnected from the supply and is connected to another
H
uncharged 600 pF capacitor. How much electrostatic energy is
Here q = 8 mC = 8 x 1O-3C, qo = - 2 x 10-9 C
lost in the process ?
'1 = 3 em = 3 x 10-2 m, r2 = 4 em = 4 x 10-2m
Ans. Here C; = 600 pF, VI = 200 V,
<; = 600 pF, V2 = 0
.. W = - 2 x 10-9 x 8 x 10-3 x 9 x 109
O

Common potential,
12
x[ 4 x ~o-2 - 3 x :0- 2 ]
V = C;VI + <;V2 = 600 x 10- x200+ 0 = 100 V
C; + <; (600+ 600) x 10-12 = 1.2 J.
2.13. A cube of side b has a charge q at each of its vertices.
Initial energy stored,
Determine the potential and electric field due to this charge
M

1 2 1 -12 2 array at the centre of the cube.


Ui = U1 ="2C; VI ="2 x600x10 x(200)
Ans. Length of longest diagonal of the cube
= 12x10-6J = ~b2 + b2 + b2 = J3 b
Final energy stored,
Distance of each charge (placed at vertex) from the
1 2 centre of the cube is
Uf ="2(C; + <;)V
r= J3 b
= -.!(600+ 600) x 10-12 x(100)2 = 6 x 10- 6 J 2
2 :. Potential at the centre of the cube is
Electrostatic energy lost, V = 8._l_.!i=8x_1_.~
6.U = U, - U = 12x10--{;-6 x 10-6 41t Eo r 41t EO J3 b
f
4q
= 6 x10-6 J
2.94 PHYSICS-XII

Electric fields at the centre due to any pair of charges Ans. (a) The charge q placed at the centre of the shell
at the opposite comers will be equal and opposite thus induces a charge - q on the inner surface of the shell and
cancelling out in pairs. Hence resultant electric field at the charge + q on its outer surface.
centre will be zero. .. Surface charge density on the inner surface of the
2.14. Two tiny spheres carrying charges 1.5JlC and 2.5 JlC shell
are located 30 em apart. Find the potential (a) at the midpoint of charge q
---2
the line joining the two charges, and (b) at a point 10 em from surface area 41t1.
this midpoint in a plane normal to the line and passing through
Surface charge density on the outer surface of the shell
the midpoint.
Q+q
Ans. The two situations are shown in Fig. 2.191. 4m:2 .
2

R
C
(b) Even if the shell is not spherical, the entire charge
resides on its outer surface. The net charge on the inner
§f:;&; surface enclosing the cavity is zero. From Gauss's theorem,
6~¥ IDem
~
~ electric field vanishes at all points inside the cavity. For a
\~~/ cavity of arbitrary shape, this is not enough to claim that

SI
electric field inside must be zero. The cavity surface may
A~------...J...------->eB
15 em 0 15 em have positive and negative charges with total charge zero.
q] = 1.5 JlC q2 = 2.5 JlC

Fig. 2.191

(a) Here 1. = r2 = 15em = 0.15 m


:. Potential at the midpoint 0 of the line joining the
P@_R
+
S

Q
-

two charges is
IT
v: - _1_ [ql + q2]
o - 4m;0 1. r2
Fig. 2.192 Electric field vanishes inside a cavity of any shape.
6
= 9 x 109 [ 1.5 x 10- + 2.5 x 10-6] V
0.15 0.15 To overrule this possibility, consider a closed loop
PQRSP, such that part PQR is inside the cavity along a
H
= 9 x 109 x 10-6 [ 10 + 5;] V line of force and the part RSP is inside the conductor.
Since the field inside a conductor is zero, this gives a
network done by the field (in part RSP) in carrying a test
= 9 x 103 x 80 V = 2.4 xl0s V.
3 charge over a closed loop. But this is not possible for a
conservative field like the electrostatic field. Hence there
(b) Here 1. = r2 = ~102 + 152
O

are no lines of force (i.e., no field), and no charge on the


= 55 :::.18em = 0.18 m inner surface of the conductor, whatever be its shape.
.. Potential at point C due to the two charges is 2.16. (a) Show that the normal component of electrostatic
field has a discontinuity from one side of a charged surface to
v: - _1_ [ql + q2] another given by
c - 41tEo 1. r2
cr
M

4 ~ "

9 [ 1.5 x 10-6 2.5 x 10-6] (f2-f1).n =-


= 9xl0 + ---- Eo
0.18 0.18
9 x 109 x 4 x 10-6 5
where n is a unit vector normal to the surface at a point and cris
------=2xl0 V. the surface charge density at that point. (The direction of n is
0.18
from side 1 to side 2)
2.15. A spherical conducting shell of inner radius 1. and
Hence show that just outside a conductor, the electric field
outer radius r2 has a charge Q.
is cr~ / EO'
(a) A charge q is placed at the centre of the shell. What is the
surface charge density on the inner and outer surfaces of (b) Show that the tangential component of electrostatic field
the shell ? is continuous from one side of a charged surface to another.
(b) Is the electric field inside a cavity (with no charge) zero [Hint: For (a), Use Gauss's law. For (b), use the fact
even if the shell is not spherical, but has any irregular that work done by electrostatic field on a closed loop is
shape ? Explain. zero.]
ELECTROSTATIC POTENTIAL AND CAPACITANCE 2.95

Ans. (a) Electric field near a plane sheet of charge is


given by ~
where E{ and Ei are the tangential components of E,. and
E=~ ~
2Eo ~' respectively. Thus,
If n is a unit vector normal to the sheet from side 1 to E{=Ei (',: I.*- 0)
side 2, then electric field on side 2 Hence the tangential component of the electrostatic
~ cr" field is continuous across the surface.
~=-n
2Eo 2.17. A long charged cylinder of linear charged density A. is
surrounded by a hollow co-axial conducting cylinder. What is
in the direction of the outward normal to the side 2.
the electric field in the space between the two cylinders ?

R
Similarly, electric field on side 1 is
Ans. Refer answer to Q. 35 on page 2.30.
~ cr"
E,.=--n 2.18. In a hydrogen atom, the electron and proton are bound
2Eo
at a distance of about 053 A.
in the direction of the outward normal to the side 1.

~J (i) Estimate the potential energy of the system in eY,

SI
(E; - ~). n = .z.-
2Eo
(-
2Eo
=~
EO
taking the zero of potential energy at infinite
separation of the electron from proton.
~ ~ (ii) What is the minimum work required tofree the elec-
As E,. and ~ act in opposite directions, there must be tron, given that its kinetic energy in the orbit is half
discontinuity at the sheet of charge. ow electric field the magnitude of potential energy obtained in (i) ?
vanishes inside a conductor, therefore (iii) What are the answers to (i) and (ii) above if the zero
of potential energy is taken at 1.06A separation ?
Ans. (i) ifI = -1.6 x 10-19 C,
Hence outside the conductor, the electric field is
IT
q2 = + 1.6 x 10-19 C,
r = 0.53 A = 0.53 x 10-10 m
~
(b) Let XY be the charged surface of a dielectric and E,. P.E. of the electron-proton system will be
~ U = _1_. qlq2
and ~ be the electric fields on the two sides of the charged
r
H
41tEo
surface as shown in Fig. 2.193.
(-1.6 x 10-19) x1.6x10-19
=9 x 109 x J
~~ 0.53 x 10-10

d~ : 9x1.6x1.6xlO-19
0.53
J
O

A E; = E1 CDS 91 D
----------------
1+ + + + + + + + + +1
X ---..."-----------...,,,---- Y
9 x 1.6 x 1.6 x 10-19 eV
1 ---------1 0.53 x 1.6 x 10-19
B C
=- 27.2 eV.
' E2 ,
M

~ __ ~2+__ J Dielectric (ii) K.E. of the electron in the orbit


1 1
= - P.E. = - x 27.2eV = 13.6eV
Ei. = E2 CDS 92 2 2
:. Total energy of the electron
Fig. 2.193
= r.E. + K.E.
Consider a rectangular loop ABCD with length I and = (- 27.2 + 13.6)eV
negligibly small breadth. Line integral along the closed
= -13.6eV.
path ABCD will be
As minimum energy of the free electron is zero, so
J E.di=F;.l-~.l =0 minimum work required to free the electron
or E,.I cos 91 - ~I cos 92 = 0 = 0 - ( -13.6)
(E,. cos f\ - ~ cos 92) I = 0 = 13.6 eV.
2.96 PHYSICS-XII

(iii) When the zero of potential energy is not taken at (1.6 x 10-19)2 x 9 x 109 x (- 4)
infinity, the potential energy of the system is 10-10 3 J
U - ql q2 []. _ ~] (1.6 x 10-19)2 x 9 x 109 x 4
=- 19 eV=-19.2eV
4m,0 1. r2 1.6 x 10- x 3

= 9 x 109 x (-1.6 x 10-19) x 1.6 x 10-19 [.: 1 eV = 1.6 x 10-19 JJ

[ 1 1] 2.20. Two charged conducting spheres of radii a and bare


J connected to each other by a wire. What is the ratio of electric
x 0.53 x 10-10 - 1.06 x 10-10
fields at the surfaces of the two spheres? Use the result obtained
9 x 109 x 1.6 x 10-19 x 1.6 x 10-19 [ 1] to explain why charge density on the sharp and pointed ends of a

R
-- 1-- eV conductor is higher than that on its flatter portions.
- . 1.6 x 10-19 x 0.53 x 10-10 2
Ans. The charges will flow between the two spheres
9 x 1.6
= ----eV =-13.6eV till their potentials become equal. Then the charges on the
0.53 x 2 two spheres would be
This indicates that the K.E. of 13.6 eV of case (i) is used Q1 = c; V =.S.

SI
up in increasing the P.E. from -27.2eV to -13.6eV as the Q2 CzV Cz
electron is carried from 0.53 A to 1.06 A position. K.E. in
this situation should be zero. As the total energy in this
But
.S. = ~
case is zero, therefore, minimum work required to free the Cz b
electron
= 0 - (-13.6eV) = 13.6 eV.
2.19. If one of the two electrons of a H2 molecule is removed,
we get a hydrogen molecular ion (H2+). In the ground state of a The ratio of the electric fields at the surface of the two
IT
H; ion, the two protons are separated by roughly 15 Ii, and the spheres will be

electron is roughly 1A from each proton. Determine the 1 Q1


potential energy of the system. Specify your choice of the zero of 11 ~ .7 q b2 a b2 b
potential energy. Ez = _1_ Q2 = Q2 . a2 = b' a2 = -;;.
Ans. The system of charges is shown in Fig. 2.194. 41tSo . b2
!i. =
H
0'1
Also,
Ez 0'2

~=~
0'2 a
O

Thus the surface charge densities are inversely


qz = + qfl--------:---- ...•q3 = + q proportional to the radii of the spheres. Since the flat
Proton rz = 1.5 A Proton portion may be considered as a spherical surface of large
radius and a pointed portion as that of small radius, that is
Fig. 2.194
why, the surface charge density on the sharp and pointed
Charge on an electron,
M

ends of a conductor is much higher than that on its flatter


% = - e = - 1.6 x 10-19 C portion.
Charge on each proton, 2.21. Two charges -q and + q are located at points (O,O,-a)
q2 = q3 = + e = + 1.6 x 10-19 C and (O,O,a) respectively.
If the zero of potential energy is taken at infinity, then (i) What is the electrostatic potential at the points
potential energy of the system is (O,O,z) and (x,y,O) ?
U= U + U + U = _1_ [ ql q2 + q2q3 + ql q3 ] (ii) Obtain the dependence of potential on the distance r
12 23 13 41ts r. t: t: of a point from the origin when r / a » 1
o 1 2 3
(iii) How much work is done in moving a small test
_ 1 [ (-q)q + q.q + (-q)q ] charge from the point (5, 0, 0) to (-7,0,0) along the
- 41tso 1 x 10 10 1.5 x 10 10 1 x 10 10
x-axis?

_ e
- 41ts x 10
o
2

10
[-1+2.-1] 1.5
[q=e] Does the answer change if the path of the test charge
between the same points is not along the x-axis ?
ELECTROSTATIC POTENTIAL AND CAPACITANCE 2.97

Ans. (i) When the point P lies closer to the charge + q (iii) (5, 0, 0) and (- 7, 0, 0) are the points on the X-axis
as shown in Fig. 2.195(a), the potential at this point P i.e., these points lie on the perpendicular bisector of the
will be dipole. Each point is at the same distance from the two
charges. Hence electric potential at each of these points is
1
V = 4n eJ ~ -!] = 4n eJ
1
z ~ a - z - ~- a) ] zero.
Work done in moving the test charge qo from the point
q 2a
(5, 0, 0) to (- 7, 0, 0) is
= 4ne o 'T7
- W = q (1) - V2) = q (0 - 0) = O.
or V=_l p_ [.: p =q x 2a] No, the answer will not change if the path of the test
4n eo . z2 - a2 .
charge between the same two points is not along X-axis.
This is because the work done by the electrostatic field

R
z between two points is independent of the path connecting
the two points.
P (0, 0, z)
2.22. Figure 2.196 below shows a charge array known as an
+ q (0, 0, a) electric quadrupole. For a point on the axis of the quadrupole,
obtain the dependence ojpotential on rfor r» a. Contrast your

SI
y (a)
a result with that due to an electric dipole and an electric
monopole (i.e. a single charge).
-q (0, 0, -a)
I-- a -----..j I-- a ------l
X
+q• -q-q •• +q• •P
I, 'I
Z

Fig. 2.196
IT
Ans. Potential at point P is
+q (0, 0, a)

a
y
(b)
V = 4:eJ r ~ a -; - ; + r 1 a]

- q (0, 0, - a) = _1_. q [r(r + a)-2(r - a)(r + a)+ r(r - a)]


4nea r(r - a)(r + a)
H
X P (0, 0, z)
2 2
= _1_. q [r2 + ar - 2r2 + 2a + r - ar]
Fig. 2.195
4nea r(? - a2)

When the point P lies closer to charge - q, as shown in 1 2q a2 1 Q


O

Fig. 2.195(b), it can be easily seen that = 4neo . r(r2 - a2) = 4ne
o . r(r 2 - a2)
V= __ l p_
4n e . z2 - a2 where Q = 2q a2 is the quadrupole moment of the given
o charge distribution. As r» a, so we can write
Again, any point (x, y, 0) lies in XY-plane which is
V __ 1_ Q
perpendicular bisector of Z-axis. Such a point will be at - 4ne . r3
M

equal distances from the charges - q and + q. Hence


o
potential at point (x, y, 0) will be zero. Hence for large r, quadrupole potential varies as 1/ r3,
whereas dipole potential varies as 1/ r2 and monopole
(ii) If the distance of point P from the or-igin 0 is r, then potential varies as 1/ r.
from the results of part (i), we get
2.23. An electrical technician requires a capacitance of2 IlF
V = + _1_ __P_ [Put z = r] in a circuit across a potential difference of1 kV. A large number
- 4n E • r2 - a2
o of 1.IlF capacitors are available to him each of which can
If r » a, we Canneglect a2 compared to r2, so withstand a potential difference of not more than 400 V.
Suggest a possible arrangement that requires a minimum
V=+_l_ E. number of capacitors.
- 4n ea . r2
Ans. Let this arrangement require n capacitors of 11lF
:. For r» a, the dependence of potential V on ris 1/ r2 each in series and m such series combinations to be
type. connected in parallel.
2.98 PHYSICS-XII

P.O. across each capacitor of a series combination Ans. As c; and c; are in series, their equivalent capa-
= 1000 = 400 or n = 1000 = 2.5 citance C;C; 200 x 200
n 400 = c; + c; = 200 + 200 = 100 pF
But number of capacitors cannot be a fraction, Series combination of C2 and C3 is in parallel with C1'
n= 3 their equivalent capacitance
Equivalent capacitance of the combination is = 100pF + 100pF = 200pF
1
-. m = 2 or m = 2n = 6 The combination of C;, C; and c; is in series with C4'
n equivalent capacitance of the network
:. Total number of capacitors required 200 x 100 F _ 200 F
= 3 x6 = 18 200 + 100 p - p 3

R
So six series combinations, each of three Luf capacitors, Total charge on the network is
should be connected in parallel as shown in Fig. 2.197.
q = CV = 200 x 10-12 x 300 = 2 x 10-8 C
IIlF IIlF IIlF 3
HH This must be equal to charge on C4 and also to the sum

SI
IIlF IIlF IIlF of the charges on the combination of and C;, C; C;.
~HH .. q4 = q = 2
q4 2 x 10-8
x 10-8 C
IIlF IIlF IIlF V4 = - = 12 V = 200 V
~HH P.O. between
C4 100 x 10

points A and B
IIlF IIlF IIlF
~HH = V - V4 = (300 - 200) V = 100 V
V; = 100 V
IIlF IIlF Illf
C; VI = 100 x 10-12 x 100 = 10-8 C
IT
%
~HH =
Also the P'D, across the series combination of C2 and C3
IIlF IIlF IIlF
= 100 V
~HH ow since C; = c;
100
0------ 1kV---.0
V2 = V3 = ""2 = 50 V
H
and q2 = q3 = 200 x 10-12 x 50 = 10-8 C.
Fig. 2.197
2.26. The plates of a parallel plate capacitor have an area of
2.24. What is the area of the plates of a 2 F parallel plate 90 cm2 each and are separated by 2.5 mm The capacitor is
capacitor? Given that the separation between the plates is 05 em charged by connecting it to a 400 V supply.
Ans. Here C = 2F, d = 0.5cm = 5 x 1O-3.m
O

(i) How much energy is stored by the capacitor?


As C=EoA (ii) View this energy stored in the electrostatic field
d between the plates and obtain the energy per unit
Cd 2 x 5 x 10-3 2 volume u. Hence arrive at a relation between u and
A=-= m
EO 8.85 x 10 12 the magnitude of electric field E between the plates.
Ans. (i) Here A = 90 cm2 = 90 x 10-4 m 2 = 9 x 1O-3m2
M

.:::1130 x 106 m2 =1130 km2.


d = 2.5 mm = 2.5 x 10-3m,
2.25. Obtain the equivalent capacitance of the network
EO = 8.85 x lO-12Fm -1,
shown in Fig. 2.198. For a 300 V supply, determine the charge
and voltage across each capacitor. [eBSE OD 081 V=400V'
Capacitance of the parallel plate capacitor is

C= EoA = 8.85 x 10-12 x9x10-3 F


d 2.5 x 10-3

= 31.86 x 10-12 F = 31.86 pF.


Electrostatic energy stored by the capacitor,

100pF U= 1. CV 2 = 1. x 31.86 x 10-12 x (400)2 J


2 2
Fig. 2.198 L-----li
C4
= 254.88 x 10-8 J = 2.55 x 10-6 J.
ELECTROSTATIC POTENTIAL AND CAPACITANCE 2.99

(ii) Energy stored per unit volume or energy density of If u be the energy stored per unit volume or the energy
the capacitor is density of the capacitor, then increase in potential energy
U 2.55 x 10-6 -3 of the capacitor
U = Ad = 9 x 10-3 x 2.5 x 10-3 Jm = u x increase in volume = u. A. t.x
F.t.x = u. A.t.x
= 0.113 Jm -3.
or F = uA = 1so E2. A = 1(soE) A E
The relation between u and E can be arrived at as
follows: = l crA. E = 1qE
U
1
"2 CV
2
1 Eo A V2 1 (V)2 The physical origin of the factor 1in the force formula
U = Ad = ~ ="2 -d-' Ad ="2 Eo d

R
lies in the fact that just inside the capacitor, field is E, and
outside it is zero. So the average value E /2 contributes to
or u= ~ Eo E2.
2 the force.
2.27. A 4 flF capacitor is charged by a 200 V supply. It is 2.29. A spherical capacitor consists of two concentric
then disconnected from the supply and is connected to another spherical conductors, held in position by suitable insulating

SI
uncharged 2 flF capacitor. How much electrostatic energy of the supports (Fig. 2.199). Show that the capacitance of a spherical
first capacitor is lost in the form of heat and electromagnetic capacitor is given by
radiation? [eBSE 00 OSI C = _4_1tS-,0,-t:
.•...
17:-,,-2

Ans. Initial electrostatic energy of the 4 flF capacitor is 1. - r2

U. = ~ CV 2 = ~ x 4 x 10- 6 x (200)2 = 8 x 10-2 J where 1. and r2 are the radii of outer and inner spheres, respectively.
I 2 2
Charge on 4 flF capacitor Charge Q

= CV = 4 x 10-6 x 200 = 8 xlO-4C


IT
When the 4 flF and 2 flF capacitors are connected
together, both attain a common potential V. Thus
V = Total charge = 8 x 10-4 C = 400 V
Total capacitance (4 + 2) x 10-6 F 3
Final electrostatic energy of the combination,
H

Uf = ~ x (4 + 2) x 10-6 x (4~r J = ~6 x 1O-2J Fig. 2.199

Ans. Refer answer to Q. 34 on page 2.30.


= 5.33 x 10-2 J
2.30. A spherical capacitor has an inner sphere of radius
Electrostatic energy of the first capacitor lost in the
O

12 em and an outer sphere of radius 13 em. The outer sphere is


form of heat and electromagnetic radiation is
earthed and the inner sphere is given a charge of 25 flC. The
t.U = U - Uf = (8 - 5.33) x 10-2 J
j space between the co-centric spheres is filled with a liquid of
dielectric constant 32. (a) Determine the capacitance of the
= 2.67 x10-2 J.
capacitor. (b) What is the potential of the inner sphere ?
2.28. Show that the force on each plate of a parallel plate
(c) Compare the capacitance of this capacitor with that of an
M

capacitor has a magnitude equal to ~ qE, where q is the charge isolated sphere of radius 12 em. Explain why the latter is much
2
smaller.
on the capacitor, and E is the magnitude of electric field between
Ans. Here a = 12 em = 12 x 1O-2m ,
the plates. Explain the origin of the factor ~ .
2 b = 13 em = 13 x 1O-2em ,
Ans. Let A be the plate area and cr, the surface charge q = 2.5 flC = 2.5 x 10-6C, K = 32
density of the capacitor. Then (a) Capacitance of the spherical capacitor is
q = crA ab
C = 41tso K.--
b-a
E=~
So 32 12 x 10-2 x 13 x 10-2
---- 9 F
Suppose we increase the separation of the capacitor 9 x 10 ' (13 - 12) x 10 2
plates by small distance t.x against the force F. Then work 32 x 12 x 13
= x 10-11 F = 5.5 x 10-9 F.
done by the external agency = F. Sx 9
2.100 PHYSICS-XII

(b) Potential of the inner sphere is 2.32. A cylindrical capacitor has two co-axial cylinders of
q 2.5 x 10-6 3 2 length 15 em and radii 1.5 em and 1.4em. The outer cylinder is
V =- = 9 V = 0.45 x 10 V = 4.5 x 10 V. earthed and the inner cylinder is given a charge of 35 /lC
C 5.5 x 10-
Determine the capacitance of the system and the potential of the
(c) Capacitance of the isolated sphere of radius 12 em is inner cylinder. Neglect end effects (i.e., bending of field lines at
12 x 10-2 -11 the ends).
C= 41t EO R = 9 F = 1.3 x 10 F.
9 x 10 Ans. Here
L=15 em =0.15 m,
When an earthed conductor is placed near a charged 6 C a = 1.4 cm =0.014 rn,
q =3.5 liC =3.5 x 10-
conductor, the capacitance of the latter increases. The two b =1.5 em =0.015 rn
conductors form a capacitor. But the capacitance of an Capacitance of a cylindrical capacitor is given by
isolated conductor is always small.
21tEo L L

R
2.31. Answer carefully: C=-b-= 1 b
(i) Two large conducting spheres carrying charges Q1and In - 2 ~- 2.303 log -
a 41tEo a
Q2 are brought close to each other. Is the magnitude of
electrostatic force between them exactly given by 0.15 F

q Q22 r where r is the distance between their centres? 2 x 9 x 109 x 2.303 log 0.015

SI
41t EO r 0.014
0.15 x 10-9 F
(ii) If Coulomb's law involved 1Ir3 dependence (instead of
18 x 2.303 x 0.03
1Ir2), would Gauss' law be still true?
(iii) A small test charge is released at rest at a point in an = 0.1206 x 10-9 F = 1.2 x 10-10 F
electrostatic field configuration. Will it travel along the Potential,
line of force passing through that point? _ q _ 3.5 x 10-6 _ 4
V - - - 10 V - 2.9 x10 V.
(iv) What is the work done by the field of a nucleus in a C 1.2 x 10-
complete circular orbit of the electron? What if the orbit
2.33. A parallel plate capacitor is to be designed with a
IT
is elliptical ?
voltage rating 1kV, using a material of dielectric constant 3
(v) We know that electric field is discontinuous across the and dielectric strength about 107 Vm-1. For safety, we would
surface of a charged conductor. Is electric potential also like the field never to exceed say 10% of the dielectric strength.
discontinuous there ? What minimum area of the plates is required to have a
(vi) What meaning would you give to the capacity of a single capacitance of 50 pF ? [CBSEOD 05]
conductor? Ans. Maximum permissible voltage
H
(vii) Guess a possible reason why water has a much greater = lkV = 103 V
dielectric constant (= 80) than say, mica (= 6).
Maximum permissible electric field
Ans. (i) No. When the two spheres are brought close to
= 10%of 107 Vrn-1 = 106 Vm-1
each other, their charge distributions do not remain
uniform and they will not act as point charges. :. Minimum separation d required between the plates
O

is given by
(ii) No. Gauss's law will not hold if Coulomb's law
involved 1/ r3 or any other power of r (except 2). In E= V or d = V = 10: = 10- 3 m
that case the electric flux will depend upon r also. d E 10
(iii) Not necessarily. The small test charge will move Capacitance of a parallel plate capacitor is
along the line of force only if it is a straight line. The C= K EoA
M

line of force gives the direction of acceleration, and not d


that of velocity. Cd 50 x 10-12 x 10-3 2
A=--= m
(iv) Zero. But when the orbit is elliptical, work is done in K EO 3 x 8.85 x 10-12
moving the electron from one point to the other. How-
= 18.8 x 10-4m2 .:::19 cm2.
ever, net work done over a complete cycle is zero.
(v) No, potential is everywhere constant as it is a 2.34. Describe schematically the equipotential surfaces corres-
scalar quantity. ponding to
(vi) A single conductor is a capacitor with one plate at (i) a constant electric field in the Z-direction.
infinity. It also possesses capacitance. (ii) a field that uniformly increases in magnitude but
(vii) Because of its bent shape and the presence of remains in a constant (say, Z) directions.
two highly polar a - H bonds, a water molecule (iii) a single positive charge at the origin.
possesses a large permanent dipole moment about (iv) a uniform grid consisting of long equally spaced
0.6 x 10- 29 Cm. Hence water has a large dielectric
parallel charged wires in a plane.
constant.
ELECTROSTATIC POTENTIAL AND CAPACITANCE 2.101

Ans. (i) For a constant electric field in Z-direction, 2.37. Answer the following:
equipotential surfaces will be planes parallel to XY-planes, (i) The top of the atmosphere is at about 400 kV with respect
as shown in Fig. 2.200. to the surface of the earth, corresponding to an electric field that
iZ decreases with altitude. Near the surface of the earth, the field is
about 100 Vm-1. Why do then we not get an electric shock as we

I 7 step out of our house into the open? (Assume the house to be a
steel cage so there is no field inside.)
E1 I
7)
/.
Equipotentials
(ii) A man fixes outside his house one evening a two metre
high insulating slab carrying on its top a large aluminium sheet
of area 1m2. Will he get an electric shock if he touches the metal
I

R
~-----------------.y sheet next morning?
.: a (iii) The discharging current in the atmosphere due to the
,
,/ small conductivity of air is known to be 1800 A on an average
,
,, over the globe. Why then does the atmosphere not discharge
X~ itself completely in due course and become electrically neutral ?

SI
Fig. 2.200 In other words, what keeps the atmosphere charged?
(iv) What are the forms of energy into which the electric
(ii) In this case also, the equipotential surfaces will be
planes parallel to XY-plane. However, as field increases, energy of the atmosphere is dissipated during a lightning?
such planes will get closer. Ans. (i) Normally the equipotential surfaces are
(iii) For a single positive charge at the origin, the parallel to the surface of the earth as shown in Fig. 2.201.
equipotential surfaces will be concentric spheres having Now our body is a good conductor. So as we step out into
origin as their common centre, as shown in Fig. 2.25. The the open, the original equipotential surfaces of open air
separation between the equipotentials differing by a get modified, but keeping our head and the ground at the
IT
constant potential increases with increase in distance same potential and we do not get any electric shock.
from the origin.
300 V 300 V
(iv) Near the grid the equipotential surfaces will have
varying shapes. At far off distances, the equipotential
200 V 200 V
surfaces will be planes parallel to the grid.
2.35. In a Van de Graaff type generator, a spherical metal shell
lOOV
H
is to be a 15 x 106 V electrode. The dielectric strength of the gas
surrounding the electrode is 5 x 107 Vm-1. What is the mini-
mum radius of the spherical shell required? [CBSE OD 08]
Ans. Maximum permissible potential, V = 1.5 x 106 V
77777777777777 77777777777777
For safety, the maximum permissible electric field is
Ground Ground
O

E = 10% of dielectric strength (a) (b)


= 10% of 5 x 107 Vm -1 = 5 x 106 Vm-1
Fig. 2.201
Now for a spherical shell,
V = _1_.3. (ii) Yes. The aluminium sheet and the ground form a
47t Eo r capacitor with insulating slab as dielectric. The dis-
M

E=_l_ 3...= V charging current in the atmosphere will charge the capa-
47t EO . r2 r citor steadily and raise its voltage. Next morning, if the
man touches the metal sheet, he will receive shock to the
Minimum radius required is
extent depending upon the capacitance of the capacitor
V 1.5 x 106 V 1
r=- = = 3 x 10- m = 30 em. formed.
6
E 5 x 10 Vm 1
(iii) The atmosphere is continuously being charged by
2.36. A small sphere of radius '1 and charge q1 is enclosed by thunder storms and lightning bolts all over globe and
a spherical shell of radius r2 and charge q2' Show that if q1 is maintains an equilibrium with the discharge of the
positive, charge will necessarily flow from the sphere to the shell atmosphere in ordinary weather conditions.
(when the two are connected by a wire) no matter what the (iv) The electrical energy is lost as (i) light energy
charge q2 on the shell is. involved in lightning (ii) heat and sound energy in the
Ans. Refer answer to Q. 55 on page 2.67. accompanying thunder.
2.102 PHYSICS-XII

Text Based Exercises


~YPE A : VERY SHORT ANSWER QUESTIONS (1 mark each)

-4
1. Define electric potential. Is it a scalar or a vector 20. Write an expression for potential at point P (r )
quantity? [Punjab01; CBSEaD 06]
due to two charges 'h and q2 located at positions
2. Define the unit of electric potential. [Punjab02] -4 d-4 . I
r I an r 2 respective y.
3. Write down the relation between electric field and

R
electric potential at a point. 21. Define electron volt. How is it related to joule?
4. Name the physical quantity whose 51,unit is JC-I. 22. How many electron volts make up one joule?
Is it a scalar or a vector quantity? [CBSEaD 2010] [Himachal93]
5. Write the 51unit of potential gradient. 23. Will there be any effect on the potential at a point if

SI
6. Define electric potential difference between two the medium around this point is changed ?
points. Is it scalar or vector? [Punjab01] 24. What work must be done in carrying an a-particle
7. What do you mean by a potential difference of 1 volt? across a potential difference of 1 volt ?
8. Write the dimensional formula of potential 25. What is an equipotential surface? Give an example.
difference. [Punjab2000,02; CBSED 03]
9. 5 J of work is done in moving a positive charge of 26. Why are electric field lines perpendicular at a point
0.5 C between two points. What is the potential on an equipotential surface of a conductor?
difference between these two points? [ISCE9S] [CBSEco ISC]
IT
10. A charge of 2 C moves between two points main- 27. Can you say that the earth is an equipotential
tained at a potential difference of 1 volt. What is the surface?
energy acquired by the charge ? 28. What is the geometrical shape of equipotential
[!SCE94; CBSED 10C] surfaces due to a single isolated charge?
11. In a conductor, a point P is at a higher potential [CBSED13]
than another point Q. In which direction do the 29. What is the shape of the equipotential surfaces for a
H
electrons move? uniform electric field? [Punjab01]
12. Give two examples of conservative forces. 30. How much work is done in moving a 500 IlC charge
[Himachal93] between two points on an equipotential surface?
13. How much is the electric potential of a charge at a [CBSED 02]
O

point at infinity ? 31. A charge of + 1Cis placed at the centre of a spherical


14. What is the nature of symmetry of the potential of a shell of radius 10 cm. What will be the work done in
point charge ? moving a charge of + 11lC on its surface through a
15. What are the points at which electric potential of a distance of 5 cm ?
dipole has maximum value? 32. What is the optical analogue of an equipotential
M

16. What are the points at which electric potential of a surface?


dipole has a minimum value? 33. The middle point of a conductor is earthed and its
17. What is the nature of symmetry of a dipole ends are maintained at a potential difference of
potential? 220 V. What is the potential at the ends and at the
18. What is electrostatic potential energy? Where does middle point?
it reside? 34. Define capacitance of a conductor.
19. What is the value of the angle between the vectors 35. Can there be a potential difference between two
p and E for which the potential energy of an conductors of same volume carrying equal positive
charges?
electric dipole of dipole moment p, kept in an
36. The capacitance of a conductor is 1 farad. What do
-4
external electric field E r has maximum value? you mean by this statement?
[CBSESP IS] 37. What is a capacitor? [Punjab96C]
ELECTROSTATIC POTENTIAL AND CAPACITANCE 2.103

38. Write the physical quantity that has its unit 60. Can we place a parallel plate capacitor of 1 F
coulomb volt -1. Is it a vector or scalar quantity ? capacity in our house?
[CBSE 0 93C, 98] 61. What is the basic difference between a capacitor
39. Define capacitance. Give its 51 unit. and an electric cell ?
[CBSE 0 93C ; ISCE 98] 62. Two capacitors of capacitances c; and c; are
40. Define 51 unit of capacitance. [CBSE 00 94] connected in parallel. A charge q is given to the
41. Write the dimensions of capacitance. combination. What will be the potential difference
42. What is the net charge on a charged capacitor? across each capacitor?
43. On what factors does the capacitance of a capacitor 63. What is the order of capacitances used in a radio
receiver?

R
depend?
44. Write two applications of capacitors in electrical 64. Is there any conductor which can take unlimited
circuits. charge?
45.. In what form is the energy stored in a charged 65. A parallel plate capacitor with air between the
capacitor? plates has a capacitance of 8 pF. What will be the

SI
46. What is the basic purpose of using a capacitor? capacitance if the distance between the plates be
reduced by half and the space between them is
47. Write different expressions for the energy stored in
filled with a substance of dielectric constant K = 6?
a capacitor.
[CBSE 0 05]
48. Write down the expression for the capacitance of a
66. A 500 J.lCcharge is at the centre of a square of side
spherical capacitor.
10 cm. Find the work done in moving a charge of
49. The difference between the radii of the two spheres 10 J.lCbetween two diagonally opposite points on
of a spherical capacitor is increased. Will the capa-
the square. [CBSE 0 08]
IT
citance increase or decrease? [Punjab 2000]
67. The graph of Fig. 2.202, shows the variation of the
50. What is a dielectric ?
total energy (E) stored in a capacitor against the
51. Define dielectric constant in terms of the capa- value of the capacitance (C) itself. Which of the two
citance of a capacitor. [CBSE 0 06] the charge on the capacitor or the potential used to
52. Write down the relation between dielectric constant charge it is kept constant for this graph?
and electric susceptibility.
H
[CBSE Sample Paper 08]
~
53. Write a relation for polarisation P of a dielectric
material in the presence of an external electric field
~
E. [CBSEOO 15]
O

54. Define dielectric strength of a medium. What is its


value for vacuum ?
55. Where is the knowledge of dielectric strength
helpful?
56. What is the effect of temperature on dielectric Fig. 2.202
M

constant?
57. An air capacitor is given a charge of 2 J.lCraising its 68. Define the term 'potential energy' of charge' if at a
potential to 200 V. If on inserting a dielectric distance 'r' in an external electric field.
[CBSEOO 09]
medium, its potential falls to 50 V, what is the
dielectric constant of the medium ? 69. What is the work done in moving a test charge q
through a distance of 1 cm along the equatorial axis
58. An uncharged insulated conductor A is brought
of an electric dipole? [CBSE 00 09]
near a charged insulated conductor B. What
happens to the charge and potential of B? 70. What is the electrostatic potential due to an electric
[CBSE 00 OlC]
dipole at an equatorial point? [CBSE 00 09]

59. For a given potential difference, does a capacitor 71. A metal plate is introduced between the plates of a
store more or less charge with a dielectric than it charged parallel plate capacitor. What is its effect
does without a dielectric ? on the capacitance of the capacitor? [CBSE F 09]
2.104 PHYSICS-XII

72. A hollow metal sphere of radius 5 em is charged 76. What is the amount of work done in moving a
such that the potential on its surface is 10 V: What is charge around a circular arc of radius r at the centre
the potential at the centre of the sphere? of which another point charge is located ?
[CBSEOD11] [CBSEOD13C]
73. In which orientation, a dipole placed in a uniform 77. What is the equivalent capacitance, C, of the five
electric field is in (i) stable, (ii) unstable equili- capacitors, connected as shown in Fig. 2.203 ?
brium ? [CBSEOD 08 ; D 10] [CBSESamplePaper 2011]
74. Write the expression for the work done on an
electric dipole of dipole moment p in turning it
from its position of stable equilibrium to a position
....

R
of unstable equilibrium in a uniform electric field E.
[CBSED13C]
75. Two charges 21lC and -21lC are placed at points A v
and B, 5 cm apart. Depict an equipotential surface

SI
of the system. [CBSED13C] Fig. 2.203

Answers
1. The electric potential at any point in an electric 10. Energy acquired by the charge = qV = 2 C x
••J.
1V = 2
field is defined as the amount of work done in 11. From Q to P.
moving a unit positive charge from infinity to that
12. (i) Electrostatic force, (ii) Gravitational force.
point against the electrostatic force. It is a scalar
IT
quantity. 13. Zero.
2. The SI unit of electric potential is volt. The electric 14. The potential of a point charge is spherically
potential at a point is said to be 1 volt, if 1 joule of symmetric.
work is done in moving a positive charge of 1 15. At axial points, the electric potential of a dipole has
coulomb from infinity to that point against the a maximum positive or negative value.
H
electrostatic force. 16. At equatorial points, the electric potential of a
_ dV .
dipole is zero.
3. E=
dr 17. The dipole potential is cylindrically symmetric.
4. Electric potential or potential difference. It is a 18. The electrostatic potential energy of a system of
scalar quantity. charges may be defined as the work required to be
O

done to bring the various charges to their respective


5. SI unit of potential gradient = Vm -1.
positions from infinity.
6. The potential difference between two points in an
19. P.E. = -pEcos 9. Clearly, P.E. is maximum when
electric field may be defined as the amount of work
cos 9 = -lor 9 = 180°.
done in moving a unit positive charge from one
1
M

point to the other against the electrostatic force. It is


20. V( 17) = _1_ [ ql + q2
a scalar. 4m:o I 17 - 171 I I 17 - 172 I
7. The potential difference between two points is said
to be 1 volt if 1 joule of work is done in moving a 21. Electron volt is the potential energy gained or lost
positive charge of 1 coulomb from one point to the by an electron in moving through a potential
difference of one volt.
other against the electrostatic force.
1electron volt = leV = 1.6 x 10-19 J
.. work done ML2T-2
8. Potential difference = = ---
charge C 22. 1 J = 6.25 x 1018 eV.
23. Yes. If the dielectric constant of the medium is
ML2T-2
=0 = [ML2r3 A-I] increased, the electric potential will decrease.
AT
24. W = qLW = 2e t1V
9. V = W = ~ =10 V.
q O.5C = 3.2 x 10-19 C x 1 V = 3.2 x 10-19 J.
ELECTROSTATIC POTENTIAL AND CAPACITANCE 2.105

25. Any surface which has same electric potential at 42. Zero, because the two plates have equal and
every point is called an equipotential surface. The opposite charges.
surface of a charged conductor is an equipotential 43. The capacitance of a capacitor depends on the
surface.
geometry of the plates, distance between them and
26. If it were not so, the presence of a component of the the nature of the dielectric medium between them.
field along the surface would destroy its equi-
44. (i) Capacitors are used in radio circuits for tuning
potential nature.
purposes.
27. Yes. Earth is a conductor, so its surface is equi- (ii) Capacitors are used in power supplies for
potential. smoothening the rectified current.

R
28. For a point charge, the equipotential surfaces are 45. In a charged capacitor, energy is stored in the form
concentric spherical shells with their centre at the of electrostatic potential energy in the electric field
point charge. between its plates.
29. For a uniform electric field, the equipotential 46. To store charge and electric energy.
surfaces are parallel planes perpendicular to the
1d

SI
1 2 1
direction of the electric field. 47. U = - CV =- - => - QC
2 2 V 2
30. Zero.
48. C = 4m:o . ~ , where a and b are the radii of the
W = q !lV = 500 f,lC x 0 = 0 . b-a
31. Zero. This is because the surface of the spherical inner and outer spheres respectively.
shell will be an equipotential surface.
49. The capacitance will increase.
32. Wavefront.
50. A dielectric is essentially an insulator which allows
33. The potential at the middle point of the conductor electric induction to take place through it but does
IT
is zero and that at the ends + 110 V and - 110 V, so not permit the flow of charges through it.
that the p.d. at ends = 110 - (- 110) = 220 V. 51. The ratio of the capacitance (Cd) of the capacitor
34. The capacitance of a conductor may be defined as completely filled with the dielectric material to the
the charge required to raise its potential by unit capacitance (Cv) of the same capacitor with vacuum
amount. between its plates is called dielectric constant.
35. Yes. Two conductors of same volume but of
H
Cd
different shapes will have different capacitances. K=-
CV
36. A conductor is said to have a capacitance of 1 farad,
if 1 coulomb of charge increases its electric potential 52. K = 1 + X, where K is dielectric constant and X is

through 1 volt. electric susceptibility.


--> -->
O

37. A capacitor is a device to store electric charge. It 53. P = EOXe E.


consists of two conducting plates separated by an
insulating medium. 54. The maximum value of electric field that can exist
inside a dielectric without causing its electrical
38. Capacitance has its unit coulomb volt -1. It is a
breakdown is called its dielectric strength. The
scalar quantity.
dielectric strength for vacuum is infinity.
M

39. The capacitance of a capacitor may be defined as


55. The knowledge of dielectric strength helps in
the charge required to be supplied to either of the
designing a capacitor by determining the
conductors so as to increase the potential difference
maximum potential that can be applied across the
between them by unit amount.
capacitor without causing its electrical breakdown.
40. The S1unit of capacitance is farad (F). A capacitor
56. The value of dielectric constant decreases with the
has a capacitance of 1 F if 1 coulomb of charge is
increase of temperature.
transferred from its one plate to another on
applying a potential difference of 1 volt across the K = Vvacuum = 200 = 4.
57.
two plates. Vdielectric 50
2
41. As 1 F = lC = ~ = lC = 1(As)2 58. The charge on the conductor Bremains unchanged
. 1V IJ / C IJ INm but its potential gets lowered.
A 2T2 [M-1L-2T4A2) 59. A capacitor with a dielectric has a higher capaci-
[Capacitance) = MLT-2L = .
tance and hence stores more charge.
2.106 PHYSICS-XII

60. No. If d = 1 em = 10- 2 m, then area of such a capa- 68. The potential energy of a charge q is the work done
citor would be in bringing charge q from infinity to the position r
2
A = Cd = 1 F x 10- m = 109 m 2 in the external electric field.
Eo 8.85 x 10-12 C2N-1m-2
U (r) = qV(r)
This is a plate about 30 km in length and breadth. 69. As potential at any point on the equatorial axis of
61. A capacitor provides electrical energy stored in it. an electric dipole is zero, so
A cell provides electrical energy by converting
chemical energy into electrical energy. W = q ~ V = q( 0 - 0) = o.
62. In parallel combination, potential difference is same 70. Zero.

R
across each capacitor. 71. The introduction of a metal sheet of thickness t in a
parallel plate capacitor increases its capacitance by
Net capacitance, C= c; + ~
a factor of .s:
d -t
r where d is the plate separation of
:. P.D. across each capacitor, V = !L = --q- .
C c; + ~ the capacitor.

SI
63. In the power supply, it is 1- 10 /-IFand for tuning 72. Potential at the centre
purposes, it is 100/-lF. = Potential at the surface = 10V.

64. Yes, the earth because of its large capacitance can 73. (i) When the dipole moment p is parallel to the the
take unlimited charge. -+
electric field E(8 = 0°), the dipole is in stable
65. With air between the capacitor plates,
E A
equilibrium.
=-"T-=8pF
Co
(ii) When the dipole moment
-+
p is antiparallel to the
IT
With dielectric between the capacitor plates, electric field E(8 = 180°), the dipole is in unstable
C =K EoA =2 KCo = 2 x 6 x 8 = 96 pF. equilibrium.
d/2
74. W = pE(cos~ -cos82) = pE(cosOO-cos1800)
66. The work done in moving a charge of 10 uC
= pE(l+ 1) = 2pE.
between two diagonally opposite points on the
square will be zero because these two points will be 75. See Fig. 2.26 on page 2.15.
H
equipotential. 76. Zero, because all points of the circular arc will be at
the same potential.
67. Energy stored, E = -.! CV2 = -.! Ii
2 2 C 77. C =~, because the combinations of C; and ~ as
1 well as C4 and Cs have been shorted.
When Q is constant, E ex: - , and we get a graph of
C
O

the type given in the question.


Hence the charge Qon the capacitor is kept constant.

"YPE B : SHORT ANSWER QU ESTIONS (2 or 3 marks each)


M

1. Distinguish between electric potential and poten- 5. Derive an expression for the electric potential at a
tial energy and write the relation between them. point along the axis line of the dipole.
[Punjab96C] [CBSE D 2000, 08 ; OD OlC, 02, 13C]
2. Define electric potential. Derive an expression for 6. Show mathematically that the electric potential at
the electric potential at a distance r from a charge q. any equatorial point of an electric dipole is zero.
[Punjab99C] [CBSEOD01]

3. Draw a plot showing the variation of (i) electric 7. Give three differences between the nature of electric
field (E)and (ii) electric potential (V) with distance r potentials of a single point charge and an electric
due to a point charge Q. [CBSE D 12] dipole.
8. Obtain an expression for the electric potential at a
4. Derive an expression for the electric potential due
point due to group of N point charges.
to an electric dipole. [Haryana01]
ELECTROSTATIC POTENTIAL AND CAPACITANCE 2.107

9. Obtain an expression for the potential at a point due 20. (a) Depict the equipotential surfaces for a system
to a continuous charge distribution. [CBSE aD 92C] of two identical positive point charges placed a
10. Show that the electric field at any point is equal to distance'd' apart.
the negative of the potential gradient at that point. (b) Deduce the expression for the potential energy
11. Describe how can we determine the electric potential of a system of two point charges ql and q2
at a point from the knowledge of electric field. brought from infinity to the points ? and ~
12. Two closely spaced equipotential surfaces A and B respectively in the presence of external electric
with potentials V and V + BV, (where 8V is the ->
field E. [CBSE D 10 ; on 15]
change in V), are kept 81 distance apart as shown in
the figure. Deduce the relation between the electric 21. Derive an expression for the potential energy of an

R
->
electric dipole of dipole moment p in an electric
B v+ov
->
field E. [Himachal02; CBSE D OS]
~--7V
22. An electric dipole is held in a uniform electric field
->
E.

SI
(a) Show that the net force acting on it is zero.

(b) The dipole is aligned with its dipole moment p


Fig. 2.204 ->
parallel to the field E . Find:
field and the potential gradient between them. Write
the two important conclusions concerning the (i) the work done in turning the dipole till its
relation between the electric field and electric dipole moment points in the direction
->
potentials. [CBSE D 14C] opposite to E .
IT
13. Show that the amount of work done in moving a
(ii) the orientation of the dipole for which the
test charge over an equipotential surface is zero. torque acting on it becomes maximum.
[Haryana97] [CBSE oo 12, 14C]
14. Show that the direction of the electric field is 23. Using Gauss's law, show that electric field inside a
normal to the equipotential surface at every point. conductor is zero. [CBSE D 2000]
15. Show that the equipotential surfaces are closed
H
24. Just outside a conductor electric field is perpen-
together in the regions of strong field and far apart dicular to the surface. Give reason.
in the region of weak field.
25. Show that the excess charge on a conductor resides
16. Sketch equipotential surfaces for only on its surface.
(i) a positive point charge. [CBSE D 2000]
26. Show that the electric field at the surface of a
O

(ii) a negative point charge. [CBSE DOl]


(iii) two equal and opposite charges separated by a
charged conductor is given by E= ~ ~, where o is
EO
small distance.
the surface charge density and ~ is a unit vector
(iv) two equal and positive charges separated by a
normal to the surface in the outward direction.
small distance. [CBSEOD 10]
M

(v) a uniform electric field. [CBSE D 2000, 01]


Or
17. (a) Draw equipotential surfaces due to a point
Derive an expression for the electric field at the
charge Q> O.
surface of a charged conductor. [CBSE aD 09]
(b) Are these surfaces equidistant from each
other? If not, explain why. [CBSE D HC] 27. Show that electric field is zero in the cavity of
hollow charged conductor.
18. Draw the equipotential surfaces due to an electric
dipole. Locate the points where the potential due 28. What is electrostatic shielding ? Mention its two
to the dipole is zero. [CBSE aD 13] applications.
29. Define electrical capacitance of a conductor. On
19. Two point charges ql and q2 are located at Ti and
what factors does it depend ?
r; respectively in an external electric field E. 30. Show that the capacitance of a spherical conductor
Obtain the expression for the total work done in is proportional to its radius. Hence justify that farad
assembling this configuration. [CBSE D 14C] is a large unit of capacitance. [Himachal96]
2.108 PHYSICS-XII

31. An isolated conductor cannot have a large capa- 44. What is meant by dielectric polarisation ? Hence
citance. Why ? establish the relation: K = 1+ X [Haryana01]
32. Why does the capacitance of a conductor increase, 45. Two dielectric slabs of dielectric constants Kl and K2
when an earth connected conductor is brought near are filled in between the two plates, each of area A,
it ? Briefly explain. of the parallel plate capacitor as shown in Fig. 2.206.
Find the net capacitance of the capacitor.
33. What is a capacitor? Explain its principle.
ICBSE00 05]
[Punjab2000, 02, 03]
34. Derive an expression for the capacitance of a parallel
plate capacitor. [CBSE0 OSC,14; 00 03]

DOl

R
35. (a) Obtain the expression for the energy stored per
unit volume in a charged parallel plate capacitor.
(b) The electric field inside a
parallel plate capacitor is aOb
++++++++++++ I-- 1/2 ------II-- 1/2 ------I

SI
E. Find the amount of Fig. 2.206
work done in moving a
d c 46. A parallel plate capacitor of capacitance C is
charge q over a closed _
rectangular loop a bed a. charged to a potential V. It is then connected to
[CBSE0 14] Fig. 2.205 another uncharged capacitor having the same
capacitance. Find out the ratio of the energy stored
36. Distinguish between polar and non-polar dielec- in the combined system to that stored initially in
trics. Give one example of each. the single capacitor. ICBSE0014]
37. Three capacitors of capacitances ;, C; and C3 are 47. If two similar plates, each of area A having surface
IT
connected in series. Find their equivalent capa- charge densities + cr and - cr are separated by a
citance. [CBSE0 92,93; Haryana 94; Himachal97] distance d in air, write expressions for: (I) The
38. Three capacitors of capacitances q. C; and C3 are electric field between the two plates (iI) The
connected in parallel. Find their equivalent potential difference between the plates (iil) The
capacitance. [CBSE0 92, 94 ; Himachal99] capacitance of the capacitor so formed.
39. Deduce the expression for the electrostatic energy ICBSE OD 07]
H
stored in a capacitor of capacitance 'C' and having 48. Explain, using suitable diagrams, the difference in
charge 'Q'. the behaviour of a (i) conductor and (ir) dielectric in
the presence of external electric field. Define the
How will the (i) energy stored and (ii) the electric
terms polarization of a dielectric and write its
field inside the capacitor be affected when it is
relation with susceptibility. ICBSE0 15]
completely filled with a dielectric material of
O

49. A capacitor is charged with a battery and then its


dielectric constant 'K ? [CBSE00 08,12]
plate separation is increased without disconnecting
40. If two charged conductors are touched mutually the battery. What will be the change in
and then separated, prove that the charges on them
(a) charge stored in the capacitor?
will be divided in the ratio of their capacitances.
(b) energy stored in the capacitor?
41. Two capacitors with capacity; and C; are charged
M

(c) potential difference across the plates of the


to potential V; and V2 respectively and then
capacitor?
connected in parallel. Calculate the common
(d) electric field between the plates of the
potential. across the combination, the charge on
capacitor? [CBSESamplePaper 2011]
each capacitor, the electrostatic energy stored in the
system and the change in the electrostatic energy 50. The charges ql = 3 flF,
from its initial value. [CBSESamplePaper 08] q2 and q3 = - 7 flF
= 4 flF
are placed on the
42. Explain why the polarization of a dielectric reduces circumference of a circle of
the electric field inside the dielectric. Hence define radius 1.0 m, as shown in
dielectric constant. [CBSE0 99] Fig. 2.207. What is the
43. Define' dielectric .constant' of a medium. Briefly value of charge q4 placed
explain why the capacitance of a parallel plate capa- on the same circle if the
citor increases, on introducing a dielectric medium potential at centre, Vc = O? Fig. 2.207
between the plates. [CBSE00 06C] IISCE03]
ELECTROSTATIC POTENTIAL AND CAPACITANCE 2.109
51. Two thin concentric shells of radii 1. and r2 (r2 > 1.) (b) How would the charge between the two flow, if
have charges ql and q2. Write the expression for the they are connected by a conducting wire? Name
potential at the surface of inner and outer shells. the device which works on this fact. [CBSE 00 09]
[CBSE 00 13 C]
53. Briefly describe discharging action of sharp points
52. (a) A charge + Q is placed on a large spherical (or corona discharge).
conducting shell of radius R Another small con-
54. Draw a labelled schematic diagram of a Van-de-
ducting sphere of radius r carrying charge 'if is
Graaff generator. State its working principle.
introduced inside the large shell and is placed at its
Describe briefly how it is used to generate high
centre. Find the potential difference between two
voltages. [CBSE 0 13 C]
points, one lying on the sphere and the other on the
shell.

R
Answers
••
1. Refer to points 3 and 11 of Glimpses. (b) Refer answer. to Q. 20 on page 2.17.

SI
2. Refer answer to Q. 2 on page 2.2. 21. Refer answer to Q. 22 on page 2.18.
3. See Fig. 2.3 on page 2.2. 22. (a) Refer answer to Q. 40 on page 1.41 of chapter 1.
n n
4. Refer answer to Q. 5 on page 2.3. (b) (i) W = J ,d8 = J pEsin 8 = pE[-cos era = -2pE.
5. Refer answer to Q. 3 on page 2.2. a a
6. Refer answer to Q. 4 on page 2.3. (ii) As , = pEsin 8, so , is maximum when 8 = 90°.
7. Refer answer to Q. 6 on page 2.3. 23. Refer answer to Q. 25 on page 2.24.
8. Refer answer to Q. 7 on page 2.4. 24. Refer answer to Q. 25 on page 2.24.
IT
9. Refer answer to Q. 8 on page 2.4. 25. Refer answer to Q. 25 on page 2.24.
10. Refer answer to Q. 10 on page 2.11. 26. Refer answer to Q. 25 on page 2.24.
11. Refer answer to Q. 11 on page 2.12. 27. Refer answer to Q. 25 on page 2.24.
12. Work done in moving a unit positive charge 28. Refer answer to Q. 26 on page 2.25.
through distance ol,
29. Refer answer to Q. 27 on page 2.26.
H
Exol =VA - VB = V -(V + OV) =-OV
30. Refer answer to Q. 29 on page 2.26.
E=- OV 31. Refer answer to Q. 30 on page 2.28.
01
32. Refer answer to Q. 31 on page 2.28.
For conclusions, refer answer to Q. 10 on page 2.12.
33. Refer answer to Q. 31 on page 2.28.
O

13. Refer answer to Q. 14 on page 2.14.


34. Refer answer to Q. 33 on page 2.29.
14. Refer answer to Q. 14 on page 2.14.
35. (a) Refer answe~ to Q. 40 on page 2.49.
15. Refer answer to Q. 14 on page 2.14.
(b) E.Lab and E.L dc, so Wab = 0 and Wed = O.
16. Refer answer to Q. 15 on page 2.15.
Also, Wbc = - Wda
17. (a) See Fig. 2.25 on page 2.15.
M

Total work done in moving charge q over the closed


(b) As E = -
dV
dr or dr = -r: dV loop abcda,
W = Wab + Wbc :+- Wcd + Wda
1
:. For constant dV, dr oc - oc r2 =
0- Wda + 0+ Wda = O.
E
36. Refer answer to Q. 43 on page 2.56.
Hence the spacing between the equipotential
surface will increase with the increase in distance 37. Refer answer to Q. 36 on page 2.33.
from the point charge. 38. Refer answer to Q. 37 on page 2.33.
18. See Fig. 2.26 on page 2.15. The electric potential is . 39. Refer answer to Q. 38 page 2.48.
.zero at the equatorial points of the dipole. When the capacitor is completely filled with a
19. Refer answer to Q. 20 on page 2.17. dielectric material and for constant charge Q,
20. (a) See Fig. 2.27 on page 2.16. C = KCU and V = Va / K
2.110 PHYSICS-XII

(i) U=!:CV2 =!:(KCu)(VO]2=.!..!CuV02 = Uo placed in an external electric field is called


2 2 K K2 K polarisation. For linear isotropic dielectrics,
~ ~
(ii) E = fu P = EOXe E
~
where Xe is the electric susceptibility of the
40. Refer answer to Q. 41 on page 2.49.
dielectric medium.
41. Refer answer to Q. 42 on page 2.49.
49. C=EoA.
42. Refer answer to Q. 45 on page 2.57. d
43. Refer to point 35 of Glimpses on page 2.116. When d is increased, C decreases.
44. Refer answer to Q. 47 on page 2.58. (a) q = CV decreases due to the decrease in the

R
45. The given arrangement is equivalent to a parallel value of C.
combination of two capacitors each with area A I 2 (b) U = !:CV2 decreases due to the decrease in the
and plate separation d. Hence the net capacitance of 2
the resulting capacitor is value of C.

SI
(c) V remains unchanged because' the battery
C=c;+c;
remains connected.
=
EO (AI 2) K} EO (AI 2) K2
+~---..=. (d) E = V I d decreases due to the increase in the
d d value of d.
46. Initial energy stored in the single capacitor SO. As Vc = 0

= !:CV2 =!: q2 :. _1_ [ !!1.+ q2 + q3 + q4 ] = 0


2 2C 4m,0 r r r r
Capacitance of the combined (parallel) system
IT
or Ih + q2 + q3 + q4 = 0
=
C+ C=2C or 3 + 4 - 7 + q4 = 0
As the total charge q remains the same, so
or q4 = O.
Final energy stored in the combined system = !:i.... 51. Potential at the surface of inner shell,
22C
Final energy =!: = 1 : 2. "t = Potential due to its own charge q}
H
Initial energy 2 + Potential due to charge q2 on outer shell

47. (i) Electric field at points between the two plates,


E=~+~=~.
- 4:EO ( :: + :: J

r
2ea 2Eo EO
Potential at the surface of outer shell
(ii) Potential difference between the plates,
O

V2 = Potential due to charge q} on inner shell


ad
V= Ed=-.
EO _ 4:J~n:~: to charge q, on outer shell
(iii) Capacitance of the capacitor so formed,
C=..i=~= EoA.
52. Refer answer to Q. 55 on page 2.67.
M

V ad I EO d
53. Refer answer to Q. 54 on page 2.67.
48. Refer answer to Q. 43 on page 2.56.
54. Refer answer to Q. 56 on page 2.67.
Polarisation of a dielectric. The induced dipole
moment set up per unit volume of a dielectric when

~YPE C : LONG ANSWER QUESTIONS (5 marks each)

1. Find the expression for the electric field intensity 2. Define electrostatic potential energy of a charge
and the electric potential, due to a dipole at a point system. Derive an expression for the potential energy
on the equatorial line. Would the electric field be of a system of three point charges. Generalise the
necessarily zero at a point where the electric result for a system of N point charges.
potential is zero ? Give an example to illustrate
your answer. ICBSE Sample Paper 2011]
ELECTROSTATIC POTENTIAL AND CAPACITANCE 2.111

3. Define the term electric dipole moment. Derive an 10. Find the expression for the capacitance of a parallel
expression for the total work done in rotating the plate capacitor of area A and plate separation d if
dipole through an angle e in uniform electric field (i) a dielectric slab of thickness t and (ii) a metallic
..... slab of thickness t, where (t < d) are introduced one
E. [CBSE OD 93, 95 ; D 96C) by one between the plates of the capacitor. In which
4. Derive an expression for the potential energy of an case would the capacitance be more and why?
electric dipole placed in a uniform electric field. [CBSE Sample Paper 2011]
Hence discuss the conditions of its stable and 11. What is a dielectric? A dielectric slab of thickness t
unstable equilibrium. is kept between the plates of a parallel plate
5. Explain the principle of a capacitor. Derive an capacitor separated by distance d. Derive the

R
expression for the capacitance of a parallel plate expression for the capacitance of the capacitor for
capacitor. [CBSE D 92, 94] t «d. [Himachal 02 ; CBSE D 93 ; OD 0lC)
6. Obtain the expression for the capacitance of a 12. (a) Show that in a parallel plate capacitor, if the
parallel plate capacitor. medium between the plates of a capacitor is filled
Three capacitors of capacitances C1, Cz and c; are with an insulating substance of dielectric constant

SI
connected (i) in series, (ii) in parallel. Show that the K. its capacitance increases. (b) Deduce the
energy stored in the series combination is the same expression for the energy stored in a capacitor of
as that in the parallel combination. [CBSE OD 03] capacitance C with charge Q. [CBSE D 09C)

7. Deduce an expression for the total energy stored in 13. (a) A small sphere, of radius 'o', carrying a positive
a parallel plate capacitor and relate it to the electric charge q, is placed concentrically inside a larger
field. [CBSE F 94C) hollow conducting shell of radius b(b> a). This
8. (a) Derive the expression for the energy stored in a outer shell has a charge Q on it. Show that if these
parallel plate capacitor. Hence obtain the spheres are connected by a conducting wire, charge
IT
expression for the energy density of the electric will always flow from the inner sphere to the outer
field. sphere, irrespective of the magnitude of the two
charges. [CBSE F 15]
(b) A fully charged parallel plate capacitor is
connected across an uncharged identical (b) Name the machine which makes use of this
capacitor. Show that the energy stored in the principle. Draw a simple labelled line diagram of
combination is less than that stored initially in this machine. What 'practical difficulty' puts on
H
the single capacitor. [CBSE OD 15] upper limit on the maximum potential difference
which this machine can built up ?
9. Define the terms (i) capacitance of a capacitor
[CBSE D 09C ; OD 14]
.(ii) dielectric strength of a dielectric.When a dielectric
is inserted between the plates of a charged parallel 14. Explain the principle of a device that can build up
plate capacitor, fully occupying the intervening high voltages of the order of a few million volts.
O

region, how does the polarization of the dielectric Draw a schematic diagram and explain the working
medium affect the net electric field ? For linear of this device.
dielectrics, show that the introduction of a Is there any restriction on the upper limit of the
dielectric increases its capacitance by a factor K. high voltages set up in this machine? Explain.
characteristic of the dielectric. [CBSE D OSC) [CBSE D 12]
M

Answers
1. For derivation of electric field intensity at No, the electric field may not be necessarily zero at

equatorial point of a dipole, refer answer to Q. 38 a point where the electric potential is zero. For
on page 1.40 of chapter 1. example, the electric potential at an equatorial point of
1 P a dipole is zero, while electric field is not zero.
El,qua = 41tEo (r2 + a2 )3/2 2. Refer answer to Q. 18 on page 2.16.
For derivation of electric potential at an equatorial 3. Refer answer to Q. 22 on page 2.18.
point of a dipole, refer answer to Q. 4 on page 2.3 of 4. Refer answer to Q. 22 on page 2.18.
chapter 2. 5. Refer answer to Q. 31 on page 2.28 and Q. 33 on
Vequa = 0 page 2.29.
2.112 PHYSICS-XII

6. Refer answer to Q. 33 on page 2.29 and Q. 39 on 11. Refer answer to Q. 49 on page 2.59.
page 2.48. 12. (a) Refer answer to Q. 49 on page 2.59.
7. Refer answer to Q. 38 on page 2.48 and Q. 40 on
(b) Refer answer to Q. 38 on page 2.48.
page 2.48.
8. (a) Refer answer to Q. 38 on page 2.48 and Q. 40 on 13. (a) Refer answer to Q. 55 on page 2.67
page 2.48. (b) A Van-de-Graaff generator works on this prin-
(b) Refer to the solution of Example 75 on page 2.52. ciple. See Fig. 2.130. The potential on the outer
9. Refer answer to Q. 45 on page 2.57 and Q. 49 on surface of its metallic shell cannot exceed the
page 2.59. breakdown field of air (=::3x106Ym-l)
because then the charges begin to leak into air.
10. For derivation, refer answers to Q. 49 on page 2.59
This puts the limit on the potential difference

R
and Q. 50 on page 2.60.
which the machine can built up.
EoA
Cdielectric = t 14. The device is Van de Graff generator. For its principle
d=t +- and working, refer answer to Q. 56 on page 2.67.
K

C = EOA Yes, there is a restriction on the upper limit of the


[For metal, K = 00 1

SI
metal d=t high voltages set up in the Van de Graaff generator.
The high voltages can be built up only upto the
Clearly, Cmetal > Cdielectric breakdown field of the surrounding medium .

.l'rYPE 0 : VALUE BASED QUESTIONS (4 marks each)


1. Immediately after school hour, as Birnla with her 2. One evening, Pankaj outside his house fixed a two
friends carne out, they noticed that there was a metre high insulating slab and attached a large
IT
sudden thunderstorm accompanied by the aluminium sheet of area 1m 2 over its top. To his
lightning. They could not find any suitable place for surprise, next morning when he incidently touched
shelter. Dr. Kapoor who was passing thereby in his the aluminium sheet, he received an electric shock.
car noticed these children and offered them to corne He got afraid. He narrated the incident to his
in his car. He even took care to drop them to the Physics teacher in the school who explained him
locality where they were staying. Birnla's parents, the reason behind it.
H
who were waiting, saw this and expressed their Answer the following questions based on the above
gratitude to Dr. Kapoor. [CBSE00 lSC] information:
(a) What values did Dr. Kapoor and Birnla's (a) What are the values being displayed by
parents display? Pankaj?
(b) Why is it considered safe to be inside a car (b) What may be the reason behind the electric
O

especially during lightning and thunder- shock received by Pankaj ?


storm ?
(c) Define the term' dielectric strength'. What does
this term signify ?
M

Answers
1. (a) Dr. Kapoor displayed helpfulness, empathy and its insulating property is called its dielectric

scientific temper. strength. It signifies the maximum electric field
Bimla parents displayed gratefulness and upto which the dielectric can safely play its role.
indebtedness. 2. (a) Keen observer and curiosity.
(b) It is safer to sit inside a car during a thunder- (b) The aluminium sheet and the ground form a
storm because the metallic body of the car capacitor alongwith the insulating slab. The
becomes an electrostatic shielding from discharging current of the atmosphere charges
lightning. the capacitor steadily and raises its voltage. So,
(c) The maximum electric field that a dielectric when Pankaj touches the aluminium sheet, he
medium can withstand without break-down of receives an electric shock.
Electrostatic Potential and Capacitance

R
GLIMPSES

SI
1. Potential difference. The potential difference 5. Electric potential due to a point charge. The
between two points is defined as the amount of electric potential of a point charge q at distance r
work done in bringing a unit positive charge from it is given by
from one point to another against the electro- 1 q 1
static forces. V=--.- i.e -., Voc-
41t 1:0 . r r
.. Work done
Potential difference = ---- It is spherically symmetric.
Charge
6. Electric potential due to a dipole. Electric potential
IT
_ _ WAB
or VAB-VB-VA ---
q
at a point having position vector r, due to a

2. 51 unit of potential difference is volt (V). The


dipole of moment p at the origin is given by

potential difference between two points in an --+ --+ 8


V- 1 p.r_ 1 pcos
electric field is said to be 1 volt if 1joule of work - 41tl:o ·7- 41tl:o -,:;.--
H
has to be done in moving a positive charge of 1
coulomb from one point to the other against the At points on the axial line of the dipole
electrostatic forces. (8 =Ooor 180°),

1 V = 1 JCI = 1 Nm CI V. = +_1_ E
axial - 41t I: .,:;.
a
O

3. Electric potential. It is defined as the amount of


At points on the equatorial line of the dipole
work done in bringing a unit positive charge
(8 =90°),
from infinity to the observation point against
the electrostatic forces. Vequa =0.

Work done
M

7. Electric potential due to a group of N point


Electric potential
Charge charges. If rl, r2, r3 ... rN are the distances of N
point charges from the observation point, then
or V=W
q V = _1_ [ qi + q2 + q3 + ....+ qN ]
Electric potential is a scalar quantity. 41tl:o 't ~ ~ rN

4. 51 unit of electric potential is volt. The electric


potential at a point in an electric field is said to be
1 volt if one joule of work has to be done in
moving a positive charge of 1 coulomb from 8. Determination of electric field from electric
infinity to that point against the electrostatic potential. The rate of change of potential with
forces. distance is called potential gradient. Electric
(2.113)
2.114 PHYSICS-XII

field at any point is equal to the negative of the 13. Potential energy of a system of three point
potential gradient at that point charges. It is given by

E=- dV U = _1_ [ q1 q2 + q2 q3 + q3 q1 ]
dr 41t So '12 r23 r31

SI unit of electric field = Vm-1 14. Potential energy of N point charges. It is given
-7
by
The direction of E is in the direction of steepest
decrease of potential. U=_l_ L qj qj
41t So All pairs ';j
9. Determination of electric potential from electric

R
field. The electric potential at a point having
position vector t is given by

r -7 -7

V =- f E . dr 15. Potential energy of a dipole in a uniform electric

SI
00 field. It is equal to the amount of work done in
turning the dipole from orientation 91 to 92 in
10. Equipotential surface. Any surface that has the field E.
same electric potential at every point on it is
U = - pE (cos 92 - cos 91)
called an equipotential surface. The surface of a
charged conductor is an equipotential surface. If initially the dipole is perpendicular to the
Some of the important properties of equi- field E, 91 =90° and 92 = 9 (say), then
potential surface are as follows : -7 -7
U = - pE cos 9 = - P . E
IT
(i) No work is done in moving a test charge
over an equipotential surface. When 9 = 0°, U = - pE i.e., the potential energy
of the dipole is minimum. The dipole is in stable
(ii) Electricfield is always normal to the
equilibrium.
equipotential surface at every point.
When 9 =90°, U =0
(iii) Equipotential surfaces are close together in
H
the regions of strong field and farther apart When 9 =180°, U =+ pE
in the regions of weak field. i.e., the potential energy of the dipole is
(iv) No two equipotential surfaces can intersect maximum. The dipole is in unstable equilibrium.
each other. 16. Conductors and insulators. Conductors are the
O

11. Electric potential energy. The electric potential substances which allow large scale physical
energy of a system of point charges is defined as movement of electric charges through them
the amount of work done in assembling the when an external electric field is applied. They
charges at their locations by bringing them in, contain a large number of free electrons.
from infinity. Insulators are the substances which do not
M

allow physical movement of electric charges


P.E. of a charge = Charge x Electric potential
through them when an external electric field is
at the given point
applied. They contain a negligibly small
It is measured in joule G) or electron volt (eV). number of free charge carriers.
1 eV = 1.6 x 10-19 J 17. Electrostatic properties of a conductor. When
12. Potential energy of a system of two point placed in an electrostatic field, a conductor
charges. If two point charges q1 and q2 are shows the following properties:
separated by distance r12 ' then their potential (i) Net electrostatic field is zero in the interior
energy is of a conductor.
U=_l_. q1 q2 (ii) Just outside the surface of a conductor,
41t So r12 electric field is normal to the surface.
ELECTROSTATIC POTENTIAL AND CAPACITANCE (Competition Section) 2.115

(iii) The net charge in the interior of a conductor 24. Parallel plate capacitor. It consists of two large
is zero and any excess charge resides on its parallel conducting plates, each of area A, and
surface. separated by a small distance d. Its capacitance
(iv) Potential is constant within and on the is
surface of a conductor.
(v) Electric field at the surface of a charged
conductor is proportional to the surface 25. Spherical capacitor. It consists of two concentric
charge density. spherical conducting shells of inner and outer
(vi) Electric field is zero in the cavity of a hollow radii a and b.

R
charged conductor. 4 1t EO ab
C=--"--
18. Electrostatic shielding. The phenomenon of b-a
making a region free from any electric field is
26. Cylindrical capacitor. It consists of two coaxial
called electrostatic shielding. It is based on the
conducting cylinders of inner and outer radii

SI
fact that electric field vanishes inside the cavity
a and b and of common length 1.
of a hollow conductor.
19. Capacitance of a conductor. It is the charge I 1
C =2 1t EO --b =2 rt EO b
required to increase the potential of a conductor loge - 2303 loglO -
by unit amount. a a
27. Capacitors in series. The equivalent capa-
. Charge
Capacitance = ---=- citance C5 of number of capacitors connected in
Potential
series is given by
IT
or C =.!i
1 1 1 1
V -=-+-+-+ ...
C5 C1 C2 C3
20. Capacitance of a spherical conductor. It is pro-
portional to the radius R of the spherical In a series combination of capacitors, the charge
conductor. on each capacitor is same but the potential
H
C = 41t1,0 R difference across any capacitor is inversely
proportional to its capacitance.
21. Capacitor. It is an arrangement of two con-
ductors separated by an insulating medium 28. Capacitors in parallel. The equivalent capa-
that is used to store electric charge and electric citance of a number of capacitors connected in
energy. parallel is given by
O

22. Capacitance of a capacitor. The capacitance of a Cp =C1 +C2 +C3 +...


capacitor is the charge required to be supplied
In a parallel combination of capacitors, the
to one of its conductors so as to increase the
potential difference across each capacitor is
potential difference between two conductors by
same but the charge on each capacitor is
M

unit amount.
q proportional to its capacitance.
c=-V 29. Energy stored in a capacitor. The energy stored
in a capacitor of capacitance C and charge q with
23. Farad. It is the SI unit of capacitance. The
voltage V is
capacitance of a capacitor is 1 farad (F) if 1
coulomb of charge is transferred from its one U =.! CV 2 = .!. Q2 =.! QV
plate to another on applying a potential dif- 2 2 C 2
ference of 1 volt across the two plates. .
30. Energy density. The electrical energy stored per
1 farad = 1 coulomb or 1F = 1 C unit volume or energy density in a region with
1 volt 1V electric field E is
1 2
1mF = 10-3 F,1 J.l F = 10-6F, 1pF = 10-12 F. U=2" EO E
2.116 PHYSICS-XII

31. Common potential. If a number of conductors of dielectric as the medium to its capacitance (Co)
capacitances CI, C2, C3, .... , at potentials VI' V2, when conductors are in vacuum.
V3, ..... , having charges ql' q2' Q3' .... respectively C
are placed in contact, their common potential V is K=-
CO
given by
V = Total charge = Ql + Q2 + Q3 + . It is also equal to the ratio of the applied electric
Total capacitance CI + C2 + C3 + . field (Eo) to the reduced value of electric field
(E) on inserting the dielectric slab between the
= CIVI + C2V2+ C3V3+···· plates of the capacitor.
CI+C2+C3+···· E E

R
K=~= 0
32. Loss of energy on sharing charges. If two E Eo - E'
conductors of capacitances CI and C2 at Here E' is the field set up due to polarisation of
potentials VI and V2 respectively are connected the dielectric in the opposite direction of Eo.
together, a loss of energy takes place which is

SI
37. Capacitance of a parallel plate capacitor filled
given by
with a dielectric.
su =.!. CIC2 (V _ V )2.
2C+C I 2 C =K C = EO KA
I 2 o d
33. Dielectric. A dielectric is a substance which does 38. Capacitance of a parallel plate capacitor with a
not allow the flow of charges through it but dielectric slab between its plates. If t is the
permits them to exert electrostatic forces on one thickness of the dielectric slab and t < d, then
another. It is essentially an insulator which can
_ EoA
IT
be polarised through small localised displace-
ments of its charges. C-d-t(l-~r
34. Polar and non-polar dielectrics. The dielectrics
39. Capacitance of a parallel plate capacitor with
made of polar molecules (such as HCl, N~,
conducting slab between its plates. For t < d,
~O, C~OH, etc.) are called polar dielectrics. In
c-1_d) EO A =(_d )
H
a polar molecule, the centre of mass of positive C
charges does not coincide with the centre of l.d-t d d-t o·
mass of negative charges.
40. Capacitance of a spherical capacitor filled with a
The dielectrics made of non-polar molecules are dielectric.
called non-polar dielectrics. In a non-polar ab
O

molecule, the centre of mass of positive charges


C = 4 1t EO K -- .
b-a
coincides with the centre of mass of negative
41. Capacitance of a cylindrical capacitor filled with
charges e.g., ~, 02' CO2, CH4, etc.
a dielectric
35. Polarisation of dielectric. If the medium
= 2 1t EO K I
M

between the plates of a capacitor is filled with a C



dielectric, the electric field due to the charged 2.303 10gl0 -
a
plates induces a net dipole moment in the
dielectric. This effect is called polarisation which 42. Van de Graaff generator. It is an electrostatic
induces a field in the opposite direction. The net generator capable of building up high potential
electric field inside the dielectric and hence the differences of the order of 107 volts, It is based
potential difference between the plates are on the principle that when a charged conductor
is brought into internal contact with a hollow
reduced. Consequently, the capacitance C increases
conductor, it transfers whole of its charge to the
from its value Co when there is vacuum.
hollow conductor, howsoever high the potential
C='KCO' of the latter may be. Also, it uses discharging
36. Dielectric constant. It is the ratio of the action of sharp points. It is used for accelerating
capacitance (C) of the capacitor with the charged particles.

You might also like